2021 AIME I Discussion
Go back to the Math Jam ArchiveAoPS Instructors will discuss all 15 problems on the 2021 AIME I.
Copyright © 2025 AoPS Incorporated. This page is copyrighted material. You can view and print this page for your own use, but you cannot share the contents of this file with others.
Facilitator: AoPS Staff
MaryM
2021-03-12 18:23:25
The Math Jam will start at 6:30 PM Eastern / 3:30 PM Pacific.
The Math Jam will start at 6:30 PM Eastern / 3:30 PM Pacific.
MaryM
2021-03-12 18:23:32
The classroom is moderated, meaning that you can type into the classroom, but these comments will not go directly into the room.
The classroom is moderated, meaning that you can type into the classroom, but these comments will not go directly into the room.
MaryM
2021-03-12 18:23:47
Please do not ask about administrative aspects of the contests or ask me to speculate about the results. We do not know what the index will be for the USAJMO or the USAMO.
Please do not ask about administrative aspects of the contests or ask me to speculate about the results. We do not know what the index will be for the USAJMO or the USAMO.
MaryM
2021-03-12 18:31:18
Welcome to the 2021 AIME I Math Jam!
Welcome to the 2021 AIME I Math Jam!
MathWizard10
2021-03-12 18:31:30
hello
hello
Lilavigne
2021-03-12 18:31:30
Hi !
Hi !
rjeagle2019
2021-03-12 18:31:30
yay
yay
jessiewang28
2021-03-12 18:31:30
hello!
hello!
MaryM
2021-03-12 18:31:33
I'm Maria Mendes, and I'll be leading our discussion today. Here's a little bit about me:
I'm Maria Mendes, and I'll be leading our discussion today. Here's a little bit about me:
MaryM
2021-03-12 18:31:41
I realized math was fun as a teenager when I saw a problem about spider webs and multiples of 8. I continued to participate in Mathematical Olympiads during high school and eventually made it onto the Brazilian IMO team. Later, I graduated in Applied Mathematics and got a Master’s degree in probability theory. I joined AoPS in 2017, as a grader, finding a passion for teaching. When not working, I am likely to be found studying art, concert chasing, or playing with my two beloved kittens.
I realized math was fun as a teenager when I saw a problem about spider webs and multiples of 8. I continued to participate in Mathematical Olympiads during high school and eventually made it onto the Brazilian IMO team. Later, I graduated in Applied Mathematics and got a Master’s degree in probability theory. I joined AoPS in 2017, as a grader, finding a passion for teaching. When not working, I am likely to be found studying art, concert chasing, or playing with my two beloved kittens.
MaryM
2021-03-12 18:31:51
Before we get started I would like to take a moment to explain our virtual classroom to those who have not previously participated in a Math Jam or one of our online classes.
Before we get started I would like to take a moment to explain our virtual classroom to those who have not previously participated in a Math Jam or one of our online classes.
MaryM
2021-03-12 18:31:58
The classroom is moderated, meaning that students can type into the classroom, but these comments will not go directly into the room. These comments go to the instructors, who may choose to share your comments with the room.
The classroom is moderated, meaning that students can type into the classroom, but these comments will not go directly into the room. These comments go to the instructors, who may choose to share your comments with the room.
MaryM
2021-03-12 18:32:04
This helps keep the class organized and on track. This also means that only well-written comments will be dropped into the classroom, so please take time writing responses that are complete and easy to read.
This helps keep the class organized and on track. This also means that only well-written comments will be dropped into the classroom, so please take time writing responses that are complete and easy to read.
MaryM
2021-03-12 18:32:11
Notice that this is a lot like one of our classes except there are a lot more of you and the same number of me. I won't be able to post all of your comments all of the time.
Notice that this is a lot like one of our classes except there are a lot more of you and the same number of me. I won't be able to post all of your comments all of the time.
MaryM
2021-03-12 18:32:28
Also, we won't be going through the math quite as thoroughly as we do in our classes -- I can't teach all the necessary material for every problem as we go. Another difference between tonight and our regular online classes is that it is very unlikely that we'll be able to answer every single question you ask. We always to try do so in our regular online classes, but we have a large number of students tonight! So, please go ahead and ask questions, but also please understand if we aren't able to answer them all!
Also, we won't be going through the math quite as thoroughly as we do in our classes -- I can't teach all the necessary material for every problem as we go. Another difference between tonight and our regular online classes is that it is very unlikely that we'll be able to answer every single question you ask. We always to try do so in our regular online classes, but we have a large number of students tonight! So, please go ahead and ask questions, but also please understand if we aren't able to answer them all!
BeefChickling34
2021-03-12 18:32:40
got that
got that
GabeW1234
2021-03-12 18:32:40
LOL same number of you
LOL same number of you
MaryM
2021-03-12 18:32:43
This jam will probably be quite long, so we will take a 5 minutes break after solving problem 9.
This jam will probably be quite long, so we will take a 5 minutes break after solving problem 9.
sugar_rush
2021-03-12 18:32:54
okay
okay
Blueclay
2021-03-12 18:32:54
ok
ok
MaryM
2021-03-12 18:33:10
Helping out today are our two assistants Eli Brottman (goveganddomath) and Detelin Dosev (OlympiadPro2000).
Helping out today are our two assistants Eli Brottman (goveganddomath) and Detelin Dosev (OlympiadPro2000).
MaryM
2021-03-12 18:33:19
Eli has always had a passion for math. He just graduated from Northern Illinois University, with a mathematics major and minors in computer science and statistics. In the future, he hopes to pursue a PhD in computer science, applied mathematics, or a related field, in order to fulfill his dream of becoming a computational cancer researcher. Eli has participated in MATHCOUNTS, USAMTS, AMC 10/12, and AIME. In his spare time, Eli enjoys listening to music, volunteering in his community, and using math to make the world a better place.
Eli has always had a passion for math. He just graduated from Northern Illinois University, with a mathematics major and minors in computer science and statistics. In the future, he hopes to pursue a PhD in computer science, applied mathematics, or a related field, in order to fulfill his dream of becoming a computational cancer researcher. Eli has participated in MATHCOUNTS, USAMTS, AMC 10/12, and AIME. In his spare time, Eli enjoys listening to music, volunteering in his community, and using math to make the world a better place.
MaryM
2021-03-12 18:33:25
Detelin won a silver medal in the 1995 IMO and a gold medal in the 1995 Balkan Math Olympiad. He received his M.S. in Mathematics from Sofia University and his Ph.D. in Mathematics from Texas A&M.
Detelin won a silver medal in the 1995 IMO and a gold medal in the 1995 Balkan Math Olympiad. He received his M.S. in Mathematics from Sofia University and his Ph.D. in Mathematics from Texas A&M.
Blueclay
2021-03-12 18:33:36
Hi.
D
Hi.

Blueclay
2021-03-12 18:33:36
Nice to meet you both!
Nice to meet you both!
goveganddomath
2021-03-12 18:33:38
Hi everyone! I look forward to working with you tonight!
Hi everyone! I look forward to working with you tonight!
OlympiadPro2000
2021-03-12 18:34:03
Hello everyone!
Hello everyone!
MaryM
2021-03-12 18:34:06
They can answer questions by whispering messages to you that will show up on your screen. However, due to the large size of the session tonight, they may not be able to get to you right away (or at all). Repeating your question over and over will not get it answered faster (in fact, it is less likely to get answered), so please, just ask your question once, be patient, and please understand that we may not be able to answer all the questions tonight.
They can answer questions by whispering messages to you that will show up on your screen. However, due to the large size of the session tonight, they may not be able to get to you right away (or at all). Repeating your question over and over will not get it answered faster (in fact, it is less likely to get answered), so please, just ask your question once, be patient, and please understand that we may not be able to answer all the questions tonight.
MaryM
2021-03-12 18:34:13
Also remember that the purpose of this Math Jam is to work through the solutions to AIME problems, and not to merely present the answers. "Working through the solutions" includes discussing problem-solving tactics. Also on occasion we may stop to prove things that you wouldn't necessary need to prove while doing the contest. So please, when a question is posted, do not simply respond with the final answer. That's not why we're here. We're going to work through the problems step-by-step, and comments that skip key steps or jump ahead in the problem, without providing explanation or motivation, won't be acknowledged.
Also remember that the purpose of this Math Jam is to work through the solutions to AIME problems, and not to merely present the answers. "Working through the solutions" includes discussing problem-solving tactics. Also on occasion we may stop to prove things that you wouldn't necessary need to prove while doing the contest. So please, when a question is posted, do not simply respond with the final answer. That's not why we're here. We're going to work through the problems step-by-step, and comments that skip key steps or jump ahead in the problem, without providing explanation or motivation, won't be acknowledged.
MaryM
2021-03-12 18:34:34
Let's get started! We're going to work through all 15 problems from the 2021 AIME I, in order.
Let's get started! We're going to work through all 15 problems from the 2021 AIME I, in order.
Poki
2021-03-12 18:34:58
yeayy!
yeayy!
mathisverynice
2021-03-12 18:34:58
alright!!
alright!!
sugar_rush
2021-03-12 18:34:58
lesgo
lesgo
MaryM
2021-03-12 18:35:02
1. Zou and Chou are practicing their 100-meter sprints by running 6 races against each other. Zou wins the first race, and after that, the probability that one of them wins a race is 23 if they won the previous race but only 13 if they lost the previous race. The probability that Zou will win exactly 5 of the 6 races is mn, where m and n are relatively prime positive integers. Find m+n.
1. Zou and Chou are practicing their 100-meter sprints by running 6 races against each other. Zou wins the first race, and after that, the probability that one of them wins a race is 23 if they won the previous race but only 13 if they lost the previous race. The probability that Zou will win exactly 5 of the 6 races is mn, where m and n are relatively prime positive integers. Find m+n.
MaryM
2021-03-12 18:35:18
What are the possibilities for how the six races can go?
What are the possibilities for how the six races can go?
themathboi101
2021-03-12 18:36:14
Win or lose
Win or lose
Hershey806
2021-03-12 18:36:14
zou will lose one of the next 5 games
zou will lose one of the next 5 games
Lilavigne
2021-03-12 18:36:14
he can lose game 2, 3, 4, 5, or 6
he can lose game 2, 3, 4, 5, or 6
Irving1004
2021-03-12 18:36:14
win 1, win 2, win 3, win 4, win 5, lose 6
win 1, win 2, win 3, win 4, win 5, lose 6
MaryM
2021-03-12 18:36:24
Z has to win 5 and C has to win 1.
Z has to win 5 and C has to win 1.
MaryM
2021-03-12 18:36:30
But we already know that Z won the first race.
But we already know that Z won the first race.
MaryM
2021-03-12 18:36:38
So C has to win exactly one of the 2nd through 6th races.
So C has to win exactly one of the 2nd through 6th races.
MaryM
2021-03-12 18:36:41
In how many ways can this happen?
In how many ways can this happen?
Bananaman27
2021-03-12 18:36:56
5
5
peace09
2021-03-12 18:36:56
5 Ways
5 Ways
jxwis2010
2021-03-12 18:36:56
5
5
rockyrockrock
2021-03-12 18:36:56
5
5
math-Passion
2021-03-12 18:36:56
5 WAYS
5 WAYS
helloworld_
2021-03-12 18:36:56
5
5
GabeW1234
2021-03-12 18:37:00
5
5
Z_Lu
2021-03-12 18:37:00
5 ways
5 ways
MaryM
2021-03-12 18:37:03
Since there are 5 choices for which race C wins, this can happen in 5 different ways.
Since there are 5 choices for which race C wins, this can happen in 5 different ways.
MaryM
2021-03-12 18:37:14
And do these 5 different ways all have equal probability?
And do these 5 different ways all have equal probability?
coolotter
2021-03-12 18:37:32
nope
nope
ryanfu2008
2021-03-12 18:37:32
NO
NO
Lilavigne
2021-03-12 18:37:32
no
no
functionalmath
2021-03-12 18:37:32
no
no
Benranger
2021-03-12 18:37:32
yes
yes
yayy
2021-03-12 18:37:32
no
no
jxwis2010
2021-03-12 18:37:32
no
no
Blueclay
2021-03-12 18:37:32
no
no
cowsrule
2021-03-12 18:37:32
no
no
peace09
2021-03-12 18:37:32
No!
No!
Irving1004
2021-03-12 18:37:32
No
No
math-Passion
2021-03-12 18:37:32
No
No
GabeW1234
2021-03-12 18:37:32
no
no
MaryM
2021-03-12 18:37:35
No. It depends on how many times the winner "flips" from Z to C or from C to Z.
No. It depends on how many times the winner "flips" from Z to C or from C to Z.
MaryM
2021-03-12 18:37:39
How many times does this flip happen?
How many times does this flip happen?
Irving1004
2021-03-12 18:38:20
2 or 1
2 or 1
yayy
2021-03-12 18:38:20
1 or 2
1 or 2
peace09
2021-03-12 18:38:20
Either Once or Twice
Either Once or Twice
jxwis2010
2021-03-12 18:38:20
once for all, twice for when C wins the last race
once for all, twice for when C wins the last race
math-Passion
2021-03-12 18:38:20
Once or Twice
Once or Twice
dbasu1
2021-03-12 18:38:20
twice for C-win = 2nd, 3rd, 4th, or 5th, and once for C-win = 6th
twice for C-win = 2nd, 3rd, 4th, or 5th, and once for C-win = 6th
cowsrule
2021-03-12 18:38:20
1 or 2?
1 or 2?
helloworld_
2021-03-12 18:38:20
1 or 2
1 or 2
MaryM
2021-03-12 18:38:23
If C wins one of the 4 middle races, such as ZZCZZZ, then a flip happens twice: once from Z to C and then once from C back to Z.
If C wins one of the 4 middle races, such as ZZCZZZ, then a flip happens twice: once from Z to C and then once from C back to Z.
MaryM
2021-03-12 18:38:29
Now that the probability is the same for any of these sequences of races; that is P(ZCZZZZ)=P(ZZCZZZ)=P(ZZZCZZ)=P(ZZZZCZ).
Now that the probability is the same for any of these sequences of races; that is P(ZCZZZZ)=P(ZZCZZZ)=P(ZZZCZZ)=P(ZZZZCZ).
MaryM
2021-03-12 18:38:33
But if C wins the last race, as in ZZZZZC, then a flip only happens once: from Z to C for the last race (and it doesn't flip back, because the races are over).
But if C wins the last race, as in ZZZZZC, then a flip only happens once: from Z to C for the last race (and it doesn't flip back, because the races are over).
MaryM
2021-03-12 18:38:41
So our total probability is 4P(ZCZZZZ)+P(ZZZZZC).
So our total probability is 4P(ZCZZZZ)+P(ZZZZZC).
MaryM
2021-03-12 18:38:46
Now we have to compute these probabilities.
Now we have to compute these probabilities.
MaryM
2021-03-12 18:38:53
What is P(ZCZZZZ)?
What is P(ZCZZZZ)?
functionalmath
2021-03-12 18:39:49
8/243 sorry
8/243 sorry
gorefeebuddie
2021-03-12 18:39:49
8/243
8/243
peace09
2021-03-12 18:39:49
1⋅(13)2⋅(23)3=8243.
1⋅(13)2⋅(23)3=8243.
dbasu1
2021-03-12 18:39:49
=1/3 * 1/3 * 2/3 * 2/3 * 2/3
= 8/243
=1/3 * 1/3 * 2/3 * 2/3 * 2/3
= 8/243
jxwis2010
2021-03-12 18:39:49
8/243
8/243
coolotter
2021-03-12 18:39:49
8/243
8/243
Z_Lu
2021-03-12 18:39:49
8/243
8/243
RedFireTruck
2021-03-12 18:39:49
8/243
8/243
Bananaman27
2021-03-12 18:39:49
(2/3)^3(1/3)^2
(2/3)^3(1/3)^2
MaryM
2021-03-12 18:39:56
We have 2 flips and 3 non-flips.
We have 2 flips and 3 non-flips.
MaryM
2021-03-12 18:40:01
So the probability is (13)2⋅(23)3.
So the probability is (13)2⋅(23)3.
MaryM
2021-03-12 18:40:09
And the other case? What is P(ZZZZZC)?
And the other case? What is P(ZZZZZC)?
sugar_rush
2021-03-12 18:40:56
16243
16243
Irving1004
2021-03-12 18:40:56
2 times that, which is 16/243
2 times that, which is 16/243
jxwis2010
2021-03-12 18:40:56
16/243
16/243
Kesav
2021-03-12 18:40:56
16/243
16/243
rockyrockrock
2021-03-12 18:40:56
(2/3)^4*1/3
(2/3)^4*1/3
dbasu1
2021-03-12 18:40:56
(2/3)^4 * (1/3) for 4 non-flips and 1 flip
=16/243
(2/3)^4 * (1/3) for 4 non-flips and 1 flip
=16/243
Hershey806
2021-03-12 18:40:56
234⋅13
234⋅13
Bananaman27
2021-03-12 18:40:56
(2/3)^4(1/3)^1
(2/3)^4(1/3)^1
yayy
2021-03-12 18:40:56
(23)4∗(13)
(23)4∗(13)
gracefulharmony
2021-03-12 18:40:56
16/243
16/243
MaryM
2021-03-12 18:40:58
We have 1 flip and 4 non-flips.
We have 1 flip and 4 non-flips.
MaryM
2021-03-12 18:41:03
So the probability is (13)⋅(23)4.
So the probability is (13)⋅(23)4.
MaryM
2021-03-12 18:41:06
Therefore, what's the overall probability?
Therefore, what's the overall probability?
donguri
2021-03-12 18:42:03
16243
16243
ryanfu2008
2021-03-12 18:42:03
16/81
16/81
gorefeebuddie
2021-03-12 18:42:03
48/243 or 16/81
48/243 or 16/81
peace09
2021-03-12 18:42:03
4⋅8+16243=48243=1681.
4⋅8+16243=48243=1681.
Lilavigne
2021-03-12 18:42:03
16/81
16/81
gracefulharmony
2021-03-12 18:42:03
48/243=16/81
48/243=16/81
rjeagle2019
2021-03-12 18:42:03
sorry 48/243
sorry 48/243
MaryM
2021-03-12 18:42:08
It's 4⋅(13)2⋅(23)3+(13)⋅(23)4.
It's 4⋅(13)2⋅(23)3+(13)⋅(23)4.
MaryM
2021-03-12 18:42:12
As a single fraction, it's 4⋅23+2435.
As a single fraction, it's 4⋅23+2435.
MaryM
2021-03-12 18:42:16
This is 4835.
This is 4835.
MaryM
2021-03-12 18:42:22
We can cancel a 3 to get 1634.
We can cancel a 3 to get 1634.
MaryM
2021-03-12 18:42:27
This won't simplify any further, so we have 1634=1681 as the fraction in lowest terms.
This won't simplify any further, so we have 1634=1681 as the fraction in lowest terms.
MaryM
2021-03-12 18:42:33
And the answer is...
And the answer is...
Kesav
2021-03-12 18:42:54
the answer is 16 + 81 = 97 =D
the answer is 16 + 81 = 97 =D
Math192
2021-03-12 18:42:54
97
97
coolotter
2021-03-12 18:42:54
097
097
sl808
2021-03-12 18:42:54
97
97
superstar627
2021-03-12 18:42:54
97
97
Hershey806
2021-03-12 18:42:54
097
097
awesomeandy
2021-03-12 18:42:54
97
97
yayy
2021-03-12 18:42:54
97
97
ryanfu2008
2021-03-12 18:42:54
97
97
s-sawen-07
2021-03-12 18:42:54
97
97
uneducatedpotato
2021-03-12 18:42:57
097
097
Argonauts16
2021-03-12 18:42:57
97
97
MaryM
2021-03-12 18:42:58
Our final answer is 16+81=097.
Our final answer is 16+81=097.
MaryM
2021-03-12 18:43:07
2. In the diagram below, ABCD is a rectangle with side lengths AB=3 and BC=11, and AECF is a rectangle with side lengths AF=7 and FC=9, as shown. The area of the shaded region common to the interiors of both rectangles is mn, where m and n are relatively prime positive integers. Find m+n.
2. In the diagram below, ABCD is a rectangle with side lengths AB=3 and BC=11, and AECF is a rectangle with side lengths AF=7 and FC=9, as shown. The area of the shaded region common to the interiors of both rectangles is mn, where m and n are relatively prime positive integers. Find m+n.
MaryM
2021-03-12 18:43:15

MaryM
2021-03-12 18:43:27
Before we dive in, let's get some more info into the diagram. I generally think it's a good idea to neatly add as much info the the diagram as we reasonably can.
Before we dive in, let's get some more info into the diagram. I generally think it's a good idea to neatly add as much info the the diagram as we reasonably can.
MaryM
2021-03-12 18:43:33
Specifically, let's label the other two points, and label some lengths. I'll also add variables for some of the unknown lengths.
Specifically, let's label the other two points, and label some lengths. I'll also add variables for some of the unknown lengths.
MaryM
2021-03-12 18:43:40

MaryM
2021-03-12 18:43:50
Now what?
Now what?
functionalmath
2021-03-12 18:44:51
use triangle similarity
use triangle similarity
coolotter
2021-03-12 18:44:51
SIMILAR TRIANGLES
SIMILAR TRIANGLES
peace09
2021-03-12 18:44:51
Similar Triangles!
Similar Triangles!
RedFireTruck
2021-03-12 18:44:51
similar triangles?
similar triangles?
helloworld_
2021-03-12 18:44:51
Use similar triangles
Use similar triangles
awesomeandy
2021-03-12 18:44:51
similar triangles
similar triangles
MeepMurp5
2021-03-12 18:44:51
Triangle PCE is similar to triangle PAB by Angle-Angle Similarity.
Triangle PCE is similar to triangle PAB by Angle-Angle Similarity.
yayy
2021-03-12 18:44:51
use similar triangles
use similar triangles
rockyrockrock
2021-03-12 18:44:51
ABP and CEP are similar ration 3:7
ABP and CEP are similar ration 3:7
peace09
2021-03-12 18:44:51
△ABP∼△CEP by AA Similarity.
△ABP∼△CEP by AA Similarity.
MaryM
2021-03-12 18:44:53
I think I see similar triangles!
I think I see similar triangles!
MaryM
2021-03-12 18:44:57
Indeed, we have ∠APB=∠CPE because they're vertical angles.
Indeed, we have ∠APB=∠CPE because they're vertical angles.
MaryM
2021-03-12 18:45:00
So △APB∼△CPE are similar right triangles.
So △APB∼△CPE are similar right triangles.
MaryM
2021-03-12 18:45:04
How does that help?
How does that help?
RedFireTruck
2021-03-12 18:46:00
write ratios!
write ratios!
happyhari
2021-03-12 18:46:00
use ratios
use ratios
Lilavigne
2021-03-12 18:46:00
we can write an equation
we can write an equation
yayy
2021-03-12 18:46:00
make equations
make equations
XGHou
2021-03-12 18:46:00
Ratio of side lengths is known to be 3/7
Ratio of side lengths is known to be 3/7
ryanfu2008
2021-03-12 18:46:00
we can write their ratios
we can write their ratios
Irving1004
2021-03-12 18:46:00
Well we can use ratios
Well we can use ratios
functionalmath
2021-03-12 18:46:00
use AB/CE
use AB/CE
rockyrockrock
2021-03-12 18:46:00
9−yx=73
9−yx=73
Irving1004
2021-03-12 18:46:00
PE/PB = 7/3
PE/PB = 7/3
happyhari
2021-03-12 18:46:00
we could use ratios
we could use ratios
peace09
2021-03-12 18:46:00
x9−y=37=y11−x.
x9−y=37=y11−x.
MaryM
2021-03-12 18:46:02
We have ABCE=BPEP=APCP.
We have ABCE=BPEP=APCP.
MaryM
2021-03-12 18:46:08
We can fill out the lengths that we've labeled.
We can fill out the lengths that we've labeled.
MaryM
2021-03-12 18:46:12
We get 37=x9−y=y11−x.
We get 37=x9−y=y11−x.
MaryM
2021-03-12 18:46:19
How do we work with this?
How do we work with this?
peace09
2021-03-12 18:46:44
Split it into two equations!
Split it into two equations!
MaryM
2021-03-12 18:46:47
Setting 37 equal to the other two fractions gives us two equations in two variables: 37=x9−y,37=y11−x.
Setting 37 equal to the other two fractions gives us two equations in two variables: 37=x9−y,37=y11−x.
MaryM
2021-03-12 18:46:52
How we can solve this?
How we can solve this?
awesomeandy
2021-03-12 18:47:22
cross multiply
cross multiply
Moonshine-Dragonwing
2021-03-12 18:47:22
cross multiplication and then systems
cross multiplication and then systems
Lilavigne
2021-03-12 18:47:22
cross multiply
cross multiply
superstar627
2021-03-12 18:47:22
crossmultiply
crossmultiply
coolotter
2021-03-12 18:47:22
cross-multiply
cross-multiply
yayy
2021-03-12 18:47:22
cross multiply
cross multiply
gracefulharmony
2021-03-12 18:47:22
Multiply out
Multiply out
math-Passion
2021-03-12 18:47:22
Cross multiply
Cross multiply
cowsrule
2021-03-12 18:47:22
27-3y=7x and 33-3x=7y by cross multiplication
27-3y=7x and 33-3x=7y by cross multiplication
rzlng
2021-03-12 18:47:22
27-37 = 7x, 33-3x = 7y by clearing denominators.
27-37 = 7x, 33-3x = 7y by clearing denominators.
MaryM
2021-03-12 18:47:24
Let's clear out the denominators: 3(9−y)=7x,3(11−x)=7y.
Let's clear out the denominators: 3(9−y)=7x,3(11−x)=7y.
MaryM
2021-03-12 18:47:30
This looks like two linear equations in two variables. So we can definitely solve it!
This looks like two linear equations in two variables. So we can definitely solve it!
MaryM
2021-03-12 18:47:37
Let's clean it up a little more: 27=7x+3y,33=3x+7y.
Let's clean it up a little more: 27=7x+3y,33=3x+7y.
MaryM
2021-03-12 18:47:41
What's the most efficient way to proceed?
What's the most efficient way to proceed?
MaryM
2021-03-12 18:47:48
Which variable is more important for us to know, x or y?
Which variable is more important for us to know, x or y?
awesomeandy
2021-03-12 18:48:27
x
x
uneducatedpotato
2021-03-12 18:48:27
x
x
helloworld_
2021-03-12 18:48:27
x
x
gracefulharmony
2021-03-12 18:48:27
x
x
cowsrule
2021-03-12 18:48:27
x
x
gorefeebuddie
2021-03-12 18:48:27
x
x
peace09
2021-03-12 18:48:27
x.
x.
Lilavigne
2021-03-12 18:48:27
x
x
coolotter
2021-03-12 18:48:27
wait actually x
wait actually x
rzlng
2021-03-12 18:48:27
x
x
MaryM
2021-03-12 18:48:31
Look back at what we're trying to find.
Look back at what we're trying to find.
MaryM
2021-03-12 18:48:44
We want the gray area [APCQ]. What is [APCQ]?
We want the gray area [APCQ]. What is [APCQ]?
yayy
2021-03-12 18:49:44
(11−x)∗3
(11−x)∗3
rzlng
2021-03-12 18:49:44
(11-x) * 3
(11-x) * 3
cowsrule
2021-03-12 18:49:44
(11-x)*3
(11-x)*3
Asher51
2021-03-12 18:49:44
area of a parallelogram with base 11-x and height 3
area of a parallelogram with base 11-x and height 3
dolphindash
2021-03-12 18:49:44
33-3x
33-3x
ryanfu2008
2021-03-12 18:49:44
33-3x
33-3x
MaryM
2021-03-12 18:49:47
Quadrilateral APCQ is a parallelogram with base PC=11−x and height AB=3, so its area is 3(11−x).
Quadrilateral APCQ is a parallelogram with base PC=11−x and height AB=3, so its area is 3(11−x).
MaryM
2021-03-12 18:49:54
(We can also take AP=y as the base and CE=7 as the height.)
(We can also take AP=y as the base and CE=7 as the height.)
MaryM
2021-03-12 18:49:58
So finding either variable will be helpful.
So finding either variable will be helpful.
MaryM
2021-03-12 18:50:03
Most of you want to find x, so let’s do that.
Most of you want to find x, so let’s do that.
MaryM
2021-03-12 18:50:15
How do we efficiently solve our system for x?
How do we efficiently solve our system for x?
yayy
2021-03-12 18:51:03
eliminate y
eliminate y
Lilavigne
2021-03-12 18:51:03
eliminate the y's
eliminate the y's
Asher51
2021-03-12 18:51:03
eliminate y
eliminate y
cowsrule
2021-03-12 18:51:03
multiply 2nd equation by 7 and multiply 1st equation by 3
multiply 2nd equation by 7 and multiply 1st equation by 3
gracefulharmony
2021-03-12 18:51:03
cancel out y
cancel out y
MaryM
2021-03-12 18:51:07
We can eliminate y.
We can eliminate y.
MaryM
2021-03-12 18:51:12
For instance, we can multiply the top equation by 7, the bottom equation by 3, and then subtract.
For instance, we can multiply the top equation by 7, the bottom equation by 3, and then subtract.
MaryM
2021-03-12 18:51:15
189=49x+21y,99=9x+21y.
189=49x+21y,99=9x+21y.
MaryM
2021-03-12 18:51:19
What do we get when we subtract?
What do we get when we subtract?
math9990
2021-03-12 18:51:52
90 = 40x
90 = 40x
rzlng
2021-03-12 18:51:52
90 = 40x
90 = 40x
Kesav
2021-03-12 18:51:52
90 = 40x
90 = 40x
coolotter
2021-03-12 18:51:52
90 = 40x
90 = 40x
cowsrule
2021-03-12 18:51:52
90=40x
90=40x
happyhari
2021-03-12 18:51:52
90 = 40x
90 = 40x
awesomeandy
2021-03-12 18:51:52
90 = 40x
90 = 40x
jxwis2010
2021-03-12 18:51:52
40x=90
40x=90
happyhari
2021-03-12 18:51:52
x = 9/4 = 2.25
x = 9/4 = 2.25
futbol3.14159
2021-03-12 18:51:52
90 = 40x, x = 9/4
90 = 40x, x = 9/4
skyleristhecoolest
2021-03-12 18:51:52
40x = 90
40x = 90
MaryM
2021-03-12 18:51:55
The y's cancel, and we're left with 90=40x.
The y's cancel, and we're left with 90=40x.
MaryM
2021-03-12 18:51:59
So x=94.
So x=94.
MaryM
2021-03-12 18:52:03
So what is the area of parallelogram APCQ?
So what is the area of parallelogram APCQ?
Kesav
2021-03-12 18:52:52
26.25 or 105/4
26.25 or 105/4
peace09
2021-03-12 18:52:52
[APCQ]=1054.
[APCQ]=1054.
rockyrockrock
2021-03-12 18:52:52
33-27/4
33-27/4
rzlng
2021-03-12 18:52:52
(11 - 9/4) * 3 = (35/4) * 3 = 105/4
(11 - 9/4) * 3 = (35/4) * 3 = 105/4
rockyrockrock
2021-03-12 18:52:52
105/4
105/4
awesomeandy
2021-03-12 18:52:52
3(11-9/4)
3(11-9/4)
functionalmath
2021-03-12 18:52:52
33-27/4=105/4
33-27/4=105/4
rockyrockrock
2021-03-12 18:52:52
1054
1054
coolotter
2021-03-12 18:52:52
105/4
105/4
Irving1004
2021-03-12 18:52:52
105/4
105/4
MaryM
2021-03-12 18:52:54
The area of parallelogram APCQ is 3(11−x)=3(11−94)=1054.
The area of parallelogram APCQ is 3(11−x)=3(11−94)=1054.
MaryM
2021-03-12 18:52:58
So what's our final answer?
So what's our final answer?
sugar_rush
2021-03-12 18:53:17
109
109
Z_Lu
2021-03-12 18:53:17
109
109
donguri
2021-03-12 18:53:17
109
109
dolphindash
2021-03-12 18:53:17
109
109
ryanfu2008
2021-03-12 18:53:17
109
109
cowsrule
2021-03-12 18:53:17
109
109
justin6688
2021-03-12 18:53:17
109
109
happyhari
2021-03-12 18:53:17
109
109
bookworm_2019
2021-03-12 18:53:17
109
109
rzlng
2021-03-12 18:53:17
105 + 4 = 109
105 + 4 = 109
geometry6
2021-03-12 18:53:17
109
109
MaryM
2021-03-12 18:53:20
We get 105+4=109.
We get 105+4=109.
MaryM
2021-03-12 18:53:35
3. Find the number of positive integers less than 1000 that can be expressed as the difference of two integral powers of 2.
3. Find the number of positive integers less than 1000 that can be expressed as the difference of two integral powers of 2.
MaryM
2021-03-12 18:53:43
Just so we have some easy terminology, let's call a positive number n good if we can write it as n=2a−2b for some nonnegative integers a and b.
Just so we have some easy terminology, let's call a positive number n good if we can write it as n=2a−2b for some nonnegative integers a and b.
MaryM
2021-03-12 18:53:50
So, we want to count all the good n that are less than 1000.
So, we want to count all the good n that are less than 1000.
MaryM
2021-03-12 18:54:00
Any ideas?
Any ideas?
MaryM
2021-03-12 18:54:03
We're working with powers of 2, so what number-theory tactic does that possibly suggest?
We're working with powers of 2, so what number-theory tactic does that possibly suggest?
bambithenambi
2021-03-12 18:55:03
binary!
binary!
CircleInvert
2021-03-12 18:55:03
Binary?
Binary?
peace09
2021-03-12 18:55:03
Binary?
Binary?
Z_Lu
2021-03-12 18:55:03
binary??
binary??
happyhari
2021-03-12 18:55:03
binary
binary
sugar_rush
2021-03-12 18:55:03
binary representation?
binary representation?
MaryM
2021-03-12 18:55:09
Maybe thinking about this in base 2 will help!
Maybe thinking about this in base 2 will help!
MaryM
2021-03-12 18:55:13
What's 2a in base 2?
What's 2a in base 2?
ryanfu2008
2021-03-12 18:55:52
1 followed by a 0s
1 followed by a 0s
rockyrockrock
2021-03-12 18:55:52
100000 with a 0's
100000 with a 0's
awesomeguy856
2021-03-12 18:55:52
1000... with a 0's
1000... with a 0's
helloworld_
2021-03-12 18:55:52
1 followed by a zeros
1 followed by a zeros
yuanyuanC
2021-03-12 18:55:52
1 followed by a zeros
1 followed by a zeros
happyhari
2021-03-12 18:55:52
1 and a 0s after the 1
1 and a 0s after the 1
gorefeebuddie
2021-03-12 18:55:52
1 and then a 0's trailing
1 and then a 0's trailing
peace09
2021-03-12 18:55:52
1000\hdots0⏟a zeroes.
1000\hdots0⏟a zeroes.
bambithenambi
2021-03-12 18:55:52
1 with a zeroes
1 with a zeroes
MaryM
2021-03-12 18:55:55
It's a 1 followed by a zeros.
It's a 1 followed by a zeros.
MaryM
2021-03-12 18:56:01
(That's why thinking in base k when you're working with powers of k is often helpful: the powers kn in base k have a nice, easy-to-work-with form.)
(That's why thinking in base k when you're working with powers of k is often helpful: the powers kn in base k have a nice, easy-to-work-with form.)
MaryM
2021-03-12 18:56:07
So our good number n in base 2 looks like: 10…0⏟a 0's−10…0⏟b 0's.
So our good number n in base 2 looks like: 10…0⏟a 0's−10…0⏟b 0's.
MaryM
2021-03-12 18:56:11
What does it look like when we perform this subtraction?
What does it look like when we perform this subtraction?
MaryM
2021-03-12 18:56:36
Maybe it helps to write out an example. Here's a=8 and b=5:
Maybe it helps to write out an example. Here's a=8 and b=5:
MaryM
2021-03-12 18:56:44
100000000−100000
100000000−100000
MaryM
2021-03-12 18:56:55
What do we get in this particular case?
What do we get in this particular case?
rockyrockrock
2021-03-12 18:57:42
11100000
11100000
Irving1004
2021-03-12 18:57:42
11100000
11100000
RedFireTruck
2021-03-12 18:57:42
11100000
11100000
peace09
2021-03-12 18:57:42
11100000.
11100000.
BillXu21
2021-03-12 18:57:42
11100000
11100000
yayy
2021-03-12 18:57:42
11100000
11100000
ryanfu2008
2021-03-12 18:57:42
11100000
11100000
awesomeguy856
2021-03-12 18:57:45
11100000
11100000
MaryM
2021-03-12 18:57:48
100000000−10000011100000
100000000−10000011100000
MaryM
2021-03-12 18:57:52
Do you see how this generalizes?
Do you see how this generalizes?
gorefeebuddie
2021-03-12 18:58:46
"a-b" 1111111 and then "b" 0's after
"a-b" 1111111 and then "b" 0's after
ryanfu2008
2021-03-12 18:58:46
11100000
11100000
happyhari
2021-03-12 18:58:46
a - b 1s in the beginning and b 0s in the end
a - b 1s in the beginning and b 0s in the end
dolphindash
2021-03-12 18:58:46
a-b 1's and b 0's
a-b 1's and b 0's
ryanfu2008
2021-03-12 18:58:46
2a−2b in base 2 is [a-b] 1s followed by b 0s
2a−2b in base 2 is [a-b] 1s followed by b 0s
yayy
2021-03-12 18:58:46
111...10000...0 with a−b 1's and b 0's
111...10000...0 with a−b 1's and b 0's
peace09
2021-03-12 18:58:46
2a−2b in Base 2 is Composed of a−b Leading 1's and b Leading 0's?
2a−2b in Base 2 is Composed of a−b Leading 1's and b Leading 0's?
Lilavigne
2021-03-12 18:58:46
b-a ones followed by b zeros
b-a ones followed by b zeros
jxwis2010
2021-03-12 18:58:46
1....10...0, a-b 1s, b 0's
1....10...0, a-b 1s, b 0's
MaryM
2021-03-12 18:58:49
We get that a good number, written in base 2, has one or more 1's followed by zero or more 0's. Specifically, a−b is the number of 1's, and b is the number of 0's.
We get that a good number, written in base 2, has one or more 1's followed by zero or more 0's. Specifically, a−b is the number of 1's, and b is the number of 0's.
MaryM
2021-03-12 18:58:55
You could prove this algebraically with the identity 2a−2b=2a−1+2a−2+⋯+2b. But I think the example we did is sufficiently convincing.
You could prove this algebraically with the identity 2a−2b=2a−1+2a−2+⋯+2b. But I think the example we did is sufficiently convincing.
MaryM
2021-03-12 18:59:05
So how do we count the number of these less than 1000?
So how do we count the number of these less than 1000?
MaryM
2021-03-12 18:59:09
What do we have to decide to construct such a number?
What do we have to decide to construct such a number?
peace09
2021-03-12 19:01:03
How Many 1's and How Many 0's
How Many 1's and How Many 0's
yayy
2021-03-12 19:01:03
the number of 1's and 0's
the number of 1's and 0's
jxwis2010
2021-03-12 19:01:03
how many 0s, how many 1s
how many 0s, how many 1s
awesomeguy856
2021-03-12 19:01:03
the amount of 1s and amount of 0s
the amount of 1s and amount of 0s
MaryM
2021-03-12 19:01:08
We have to decide how many 1’s and how many 0’s we have. This is the same thing as deciding in which position the first 1 will go, and how many 1’s we will have.
We have to decide how many 1’s and how many 0’s we have. This is the same thing as deciding in which position the first 1 will go, and how many 1’s we will have.
MaryM
2021-03-12 19:01:20
For example, if the first 1 is in the 20 spot, what are the choices for how many 1's we can have?
For example, if the first 1 is in the 20 spot, what are the choices for how many 1's we can have?
MaryM
2021-03-12 19:01:40
By first I mean leftmost, by the way.
By first I mean leftmost, by the way.
MaryM
2021-03-12 19:03:13
Careful, we want the leftmost one to be in the units digits.
Careful, we want the leftmost one to be in the units digits.
peace09
2021-03-12 19:03:47
Only 1 1!
Only 1 1!
Lilavigne
2021-03-12 19:03:47
1
1
MT-Math
2021-03-12 19:03:47
only one 1 possible
only one 1 possible
MaryM
2021-03-12 19:03:49
We must have 1 one.
We must have 1 one.
MaryM
2021-03-12 19:03:58
If the leftmost 1 is in the 21 spot, how many choices?
If the leftmost 1 is in the 21 spot, how many choices?
functionalmath
2021-03-12 19:04:23
2
2
Irving1004
2021-03-12 19:04:23
2
2
rockyrockrock
2021-03-12 19:04:23
2
2
peace09
2021-03-12 19:04:23
One 1 and One 0, or Two 1s.
One 1 and One 0, or Two 1s.
Lilavigne
2021-03-12 19:04:23
2
2
donguri
2021-03-12 19:04:23
2
2
BillXu21
2021-03-12 19:04:23
2
2
MaryM
2021-03-12 19:04:26
We can have 1 or 2 ones, so we have 2 choices.
We can have 1 or 2 ones, so we have 2 choices.
MaryM
2021-03-12 19:04:31
How does this generalize?
How does this generalize?
MaryM
2021-03-12 19:04:35
If the first 1 is in the 2k spot, how many ones can we have?
If the first 1 is in the 2k spot, how many ones can we have?
Lilavigne
2021-03-12 19:05:33
k+1
k+1
Z_Lu
2021-03-12 19:05:33
k+1
k+1
BillXu21
2021-03-12 19:05:33
k+1
k+1
peace09
2021-03-12 19:05:33
k+1 1s.
k+1 1s.
Irving1004
2021-03-12 19:05:33
k+1
k+1
happyhari
2021-03-12 19:05:33
k + 1
k + 1
MT-Math
2021-03-12 19:05:33
k+1
k+1
MaryM
2021-03-12 19:05:35
We can have any number from 1 to k+1 ones. So there are k+1 choices.
We can have any number from 1 to k+1 ones. So there are k+1 choices.
MaryM
2021-03-12 19:05:39
But what else do we need to consider?
But what else do we need to consider?
ryanfu2008
2021-03-12 19:06:35
<1000
<1000
Lilavigne
2021-03-12 19:06:35
the maximum number of 1s
the maximum number of 1s
yayy
2021-03-12 19:06:35
it has to be less than 1000
it has to be less than 1000
Z_Lu
2021-03-12 19:06:35
if it will be greater than 1000
if it will be greater than 1000
ancientwarrior
2021-03-12 19:06:35
if the difference is >=1000
if the difference is >=1000
MT-Math
2021-03-12 19:06:35
what is the farthest left position we can place the first 1
what is the farthest left position we can place the first 1
MaryM
2021-03-12 19:06:38
We need the number to be less than 1000.
We need the number to be less than 1000.
MaryM
2021-03-12 19:06:48
What does that mean in terms of where the first 1 is?
What does that mean in terms of where the first 1 is?
Lilavigne
2021-03-12 19:07:23
the largest power of 2 smaller than 1000
the largest power of 2 smaller than 1000
ryanfu2008
2021-03-12 19:07:23
at most 2^9 position, as 2^10-1024>1000
at most 2^9 position, as 2^10-1024>1000
yayy
2021-03-12 19:07:23
before the 10th spot
before the 10th spot
peace09
2021-03-12 19:07:23
...Since the positive integer must be less than 1000, the number may not have more than 10 digits.
...Since the positive integer must be less than 1000, the number may not have more than 10 digits.
MaryM
2021-03-12 19:07:26
If the first 1 is in the 210=1024 position or more, then we'll never be less than 1000, so that's always too big.
If the first 1 is in the 210=1024 position or more, then we'll never be less than 1000, so that's always too big.
MaryM
2021-03-12 19:07:36
On the other hand, if the first 1 is in the 28=256 position or less, then we'll never be more than 29=512, so we're OK.
On the other hand, if the first 1 is in the 28=256 position or less, then we'll never be more than 29=512, so we're OK.
MaryM
2021-03-12 19:07:43
What happens if the first 1 is in the 29=512 position?
What happens if the first 1 is in the 29=512 position?
MaryM
2021-03-12 19:07:50
How far to the right can we go without going over 1000?
How far to the right can we go without going over 1000?
sugar_rush
2021-03-12 19:09:22
5
5
peace09
2021-03-12 19:09:22
We may have at most five 1s.
We may have at most five 1s.
yayy
2021-03-12 19:09:22
5 places
5 places
Z_Lu
2021-03-12 19:09:22
2^5
2^5
Lilavigne
2021-03-12 19:09:22
at most 5 ones
at most 5 ones
happyhari
2021-03-12 19:09:22
up to 2^5?
up to 2^5?
MaryM
2021-03-12 19:09:28
29=512 is OK.
29=512 is OK.
MaryM
2021-03-12 19:09:31
29+28=512+256=768 is OK.
29+28=512+256=768 is OK.
MaryM
2021-03-12 19:09:35
29+28+27=512+256+128=896 is OK.
29+28+27=512+256+128=896 is OK.
MaryM
2021-03-12 19:09:39
29+⋯+26=896+64=960 is OK.
29+⋯+26=896+64=960 is OK.
MaryM
2021-03-12 19:09:45
29+⋯+25=960+32=992 is OK.
29+⋯+25=960+32=992 is OK.
MaryM
2021-03-12 19:09:49
29+⋯+24=992+16=1008 is too big!
29+⋯+24=992+16=1008 is too big!
MaryM
2021-03-12 19:09:54
So there are 5 good numbers with the first one at 29 that are less than 1000.
So there are 5 good numbers with the first one at 29 that are less than 1000.
MaryM
2021-03-12 19:09:58
How do we sum all this up to get our final answer?
How do we sum all this up to get our final answer?
peace09
2021-03-12 19:11:29
5+9+8+7+\hdots+1=5+45=50.
5+9+8+7+\hdots+1=5+45=50.
Irving1004
2021-03-12 19:11:29
5 + 9(9+1)/2
5 + 9(9+1)/2
donguri
2021-03-12 19:11:29
50
50
Z_Lu
2021-03-12 19:11:29
1+2+3+4+5+6+7+8+9+5=50
1+2+3+4+5+6+7+8+9+5=50
Lilavigne
2021-03-12 19:11:29
5+(9*10/2)
5+(9*10/2)
sugar_rush
2021-03-12 19:11:29
we do 5+\dbinom{10}{2}
we do 5+\dbinom{10}{2}
BillXu21
2021-03-12 19:11:29
1+2+3+4+5+6+7+8+9+5
1+2+3+4+5+6+7+8+9+5
MaryM
2021-03-12 19:11:34
We had 1 + 2 + \cdots + 9 good numbers starting at 2^0 through 2^8, plus another 5 starting at 2^9.
We had 1 + 2 + \cdots + 9 good numbers starting at 2^0 through 2^8, plus another 5 starting at 2^9.
MaryM
2021-03-12 19:11:42
So the number of good numbers less than 1000 is (1+2+\cdots+9) + 5.
So the number of good numbers less than 1000 is (1+2+\cdots+9) + 5.
MaryM
2021-03-12 19:11:46
The first term is (1+2+\cdots+9) = \dfrac{9(10)}{2} = 45.
The first term is (1+2+\cdots+9) = \dfrac{9(10)}{2} = 45.
MaryM
2021-03-12 19:11:51
So our total is 45 + 5 = \boxed{050}.
So our total is 45 + 5 = \boxed{050}.
BillXu21
2021-03-12 19:12:46
yay
yay
MaryM
2021-03-12 19:12:51
4. Find the number of ways 66 identical coins can be separated into three nonempty piles so that there are fewer coins in the first pile than in the second pile and fewer coins in the second pile than in the third pile.
4. Find the number of ways 66 identical coins can be separated into three nonempty piles so that there are fewer coins in the first pile than in the second pile and fewer coins in the second pile than in the third pile.
MaryM
2021-03-12 19:13:00
Can we write an equation that we're trying to count solutions to?
Can we write an equation that we're trying to count solutions to?
Lilavigne
2021-03-12 19:14:06
x+y+z=66, such that x < y < z
x+y+z=66, such that x < y < z
ryanfu2008
2021-03-12 19:14:06
a+b+c = 66, a<b<c
a+b+c = 66, a<b<c
BorealBear
2021-03-12 19:14:06
a+b+c=66 for a<b<c
a+b+c=66 for a<b<c
BillXu21
2021-03-12 19:14:06
x+y+z=66 x<y<z x,y,and z are positive integers
x+y+z=66 x<y<z x,y,and z are positive integers
peace09
2021-03-12 19:14:06
a_1+a_2+a_3=66, where 0<a_1<a_2<a_3, and a_i denotes the number of coins in the ith pile.
a_1+a_2+a_3=66, where 0<a_1<a_2<a_3, and a_i denotes the number of coins in the ith pile.
sugar_rush
2021-03-12 19:14:06
a+b+c=66 and 0<a<b<c
a+b+c=66 and 0<a<b<c
RedFireTruck
2021-03-12 19:14:06
a+b+c=66, 0<a<b<c
a+b+c=66, 0<a<b<c
yayy
2021-03-12 19:14:06
a + b + c = 66, a < b < c
a + b + c = 66, a < b < c
ryanfu2008
2021-03-12 19:14:06
a+b+c=60
a<b<c
a+b+c=60
a<b<c
Z_Lu
2021-03-12 19:14:06
set x,y,z as piles 1,2 and 3, respectively. Then x+y+z=66, and x<y<z
set x,y,z as piles 1,2 and 3, respectively. Then x+y+z=66, and x<y<z
MeepMurp5
2021-03-12 19:14:06
x+y+z=66, x<y<z, x,y,z\in positive integers
x+y+z=66, x<y<z, x,y,z\in positive integers
MaryM
2021-03-12 19:14:09
We want solutions to x + y + z = 66 with 0 < x < y < z.
We want solutions to x + y + z = 66 with 0 < x < y < z.
MaryM
2021-03-12 19:14:14
Any ideas?
Any ideas?
MaryM
2021-03-12 19:14:51
There is a long bookkeeping solution where you perform casework on x and make a chart. But I'll present a more interesting solution.
There is a long bookkeeping solution where you perform casework on x and make a chart. But I'll present a more interesting solution.
MaryM
2021-03-12 19:15:01
One good problem-solving tactic is to try to solve a simpler problem.
One good problem-solving tactic is to try to solve a simpler problem.
MaryM
2021-03-12 19:15:05
What's a simpler problem?
What's a simpler problem?
MaryM
2021-03-12 19:15:43
Hint: can we remove one condition from the problem?
Hint: can we remove one condition from the problem?
XGHou
2021-03-12 19:16:15
If we remove the restriction that x<y<z
If we remove the restriction that x<y<z
coolotter
2021-03-12 19:16:15
remove the 0<x<y<z constraint
remove the 0<x<y<z constraint
CircleInvert
2021-03-12 19:16:15
Remove a<b<c
Remove a<b<c
yalex999
2021-03-12 19:16:15
x < y < z
x < y < z
RedFireTruck
2021-03-12 19:16:15
0<x<y<z
0<x<y<z
yayy
2021-03-12 19:16:15
a b and c don't have to be ordered
a b and c don't have to be ordered
b20081
2021-03-12 19:16:15
0<x<y<z, we can count the unordered and place them in order
0<x<y<z, we can count the unordered and place them in order
MaryM
2021-03-12 19:16:18
Let's just count positive integer solutions to x + y + z = 66 without any restrictions.
Let's just count positive integer solutions to x + y + z = 66 without any restrictions.
MaryM
2021-03-12 19:16:33
This is simpler because there's a commonly-used technique to solve this.
This is simpler because there's a commonly-used technique to solve this.
sugar_rush
2021-03-12 19:16:49
stars & bars?
stars & bars?
peace09
2021-03-12 19:16:49
Stars-and-Bars!
Stars-and-Bars!
volcanogobbler
2021-03-12 19:16:49
stars and bars
stars and bars
BillXu21
2021-03-12 19:16:51
stars and bars
stars and bars
MaryM
2021-03-12 19:16:53
Imaging placing the 66 coins in a row.
Imaging placing the 66 coins in a row.
MaryM
2021-03-12 19:17:11
Then to separate them into three piles, what do we need to do?
Then to separate them into three piles, what do we need to do?
yayy
2021-03-12 19:17:51
put dividers between them
put dividers between them
Lilavigne
2021-03-12 19:17:51
insert 2 bars
insert 2 bars
happyhari
2021-03-12 19:17:51
make two bars
make two bars
functionalmath
2021-03-12 19:17:51
draw "bars" to distinguish between a b and c
draw "bars" to distinguish between a b and c
mm999aops
2021-03-12 19:17:51
place two bars except 'stars' might be equal
place two bars except 'stars' might be equal
peace09
2021-03-12 19:17:51
Place 2 Separators Among the 66 Coins!
Place 2 Separators Among the 66 Coins!
floradaisy136
2021-03-12 19:17:51
put in two bars
put in two bars
MaryM
2021-03-12 19:17:55
We need to choose two of the 65 "slots" between coins, to separate the coins into three piles.
We need to choose two of the 65 "slots" between coins, to separate the coins into three piles.
MaryM
2021-03-12 19:18:01
And in how many ways can we do this?
And in how many ways can we do this?
skyleristhecoolest
2021-03-12 19:18:18
65 choose 2
65 choose 2
coolotter
2021-03-12 19:18:18
65 choose 2 = 2080
65 choose 2 = 2080
yayy
2021-03-12 19:18:18
65 choose 2
65 choose 2
BillXu21
2021-03-12 19:18:18
65 pick 2
65 pick 2
sugar_rush
2021-03-12 19:18:18
\dbinom{65}{2}=2080
\dbinom{65}{2}=2080
MT-Math
2021-03-12 19:18:18
65 choose 2
65 choose 2
mm999aops
2021-03-12 19:18:18
65*64/2
65*64/2
functionalmath
2021-03-12 19:18:18
65C2
65C2
rockyrockrock
2021-03-12 19:18:18
65 choose 2
65 choose 2
CircleInvert
2021-03-12 19:18:20
\binom{65}{2}
\binom{65}{2}
nathanqiu
2021-03-12 19:18:20
65 choose 2
65 choose 2
MaryM
2021-03-12 19:18:22
We can do this in \dbinom{65}{2} ways.
We can do this in \dbinom{65}{2} ways.
MaryM
2021-03-12 19:18:27
So there are \dbinom{65}{2} solutions to x+y+z = 66, where all we require is that x,y,z are positive integers.
So there are \dbinom{65}{2} solutions to x+y+z = 66, where all we require is that x,y,z are positive integers.
MaryM
2021-03-12 19:18:35
Now, which of these solutions are not solutions to our original problem?
Now, which of these solutions are not solutions to our original problem?
sugar_rush
2021-03-12 19:19:31
when x=y or x=z or y=z
when x=y or x=z or y=z
Irving1004
2021-03-12 19:19:31
When at least two groups have the same numebr of coins
When at least two groups have the same numebr of coins
yayy
2021-03-12 19:19:31
when x=y or x=z or y=z
when x=y or x=z or y=z
nathanqiu
2021-03-12 19:19:31
the ones where there are two equal piles, or 3 equal piles
the ones where there are two equal piles, or 3 equal piles
Montclarion6
2021-03-12 19:19:31
Any time two of the piles are equal.
Any time two of the piles are equal.
uneducatedpotato
2021-03-12 19:19:31
when two or more of these are equal
when two or more of these are equal
MaryM
2021-03-12 19:19:34
One problem we might have is that two (or all three) of the piles might have the same number of coins.
One problem we might have is that two (or all three) of the piles might have the same number of coins.
MaryM
2021-03-12 19:19:40
Can we count these?
Can we count these?
mm999aops
2021-03-12 19:20:24
yes
yes
happyhari
2021-03-12 19:20:24
yes
yes
Irving1004
2021-03-12 19:20:24
Yes
Yes
Benranger
2021-03-12 19:20:24
yes
yes
sugar_rush
2021-03-12 19:20:24
yes!
yes!
mayowl
2021-03-12 19:20:24
yes
yes
peace09
2021-03-12 19:20:24
Yes!
Yes!
MaryM
2021-03-12 19:20:26
I think so!
I think so!
MaryM
2021-03-12 19:20:32
Let's try it.
Let's try it.
MaryM
2021-03-12 19:20:51
Let's start with solutions where x=y.
Let's start with solutions where x=y.
MaryM
2021-03-12 19:20:58
What equation do we get in this case?
What equation do we get in this case?
yayy
2021-03-12 19:21:28
2x + z = 66
2x + z = 66
peace09
2021-03-12 19:21:28
2x+z=66.
2x+z=66.
happyhari
2021-03-12 19:21:28
2x + z = 66
2x + z = 66
BillXu21
2021-03-12 19:21:28
2x+z=66
2x+z=66
Lilavigne
2021-03-12 19:21:28
2x+z=66
2x+z=66
Z_Lu
2021-03-12 19:21:28
2x+z=66
2x+z=66
mm999aops
2021-03-12 19:21:28
2x+z=66
2x+z=66
MaryM
2021-03-12 19:21:38
If x=y, then we have x + x + z = 66, or 2x + z = 66.
If x=y, then we have x + x + z = 66, or 2x + z = 66.
MaryM
2021-03-12 19:21:39
How many solutions are there for positive x and z?
How many solutions are there for positive x and z?
Kesav
2021-03-12 19:22:11
32
32
Montclarion6
2021-03-12 19:22:11
32
32
functionalmath
2021-03-12 19:22:11
32
32
416554
2021-03-12 19:22:11
32
32
mm999aops
2021-03-12 19:22:11
as long as z is even- so z=2,z=4 to z=64
as long as z is even- so z=2,z=4 to z=64
yayy
2021-03-12 19:22:11
32
32
sugar_rush
2021-03-12 19:22:11
32 right?
32 right?
foodisgood
2021-03-12 19:22:11
32
32
b20081
2021-03-12 19:22:11
32
32
MaryM
2021-03-12 19:22:14
We must have 0 < 2x < 66.
We must have 0 < 2x < 66.
MaryM
2021-03-12 19:22:17
So we must have 0 < x < 33.
So we must have 0 < x < 33.
MaryM
2021-03-12 19:22:21
Does every such x give us a solution?
Does every such x give us a solution?
sugar_rush
2021-03-12 19:22:45
yep
yep
BillXu21
2021-03-12 19:22:45
yes
yes
coolotter
2021-03-12 19:22:45
yes
yes
peace09
2021-03-12 19:22:45
Yes!
Yes!
happyhari
2021-03-12 19:22:45
yes
yes
Lilavigne
2021-03-12 19:22:45
yes
yes
yayy
2021-03-12 19:22:45
yes
yes
b20081
2021-03-12 19:22:45
yes
yes
RohanQV
2021-03-12 19:22:45
Yes!
Yes!
t_ameya
2021-03-12 19:22:45
yes
yes
MaryM
2021-03-12 19:22:49
Yes: once we have 0 < x < 33, we set z = 66 - 2x.
Yes: once we have 0 < x < 33, we set z = 66 - 2x.
MaryM
2021-03-12 19:22:59
There are 32 integers x with 0 < x < 33, so there are 32 solutions with x=y.
There are 32 integers x with 0 < x < 33, so there are 32 solutions with x=y.
MaryM
2021-03-12 19:23:05
How many solutions with x=z?
How many solutions with x=z?
MaryM
2021-03-12 19:23:55
(I mean, we are fixing x=z, but this time y might not the equal to x.)
(I mean, we are fixing x=z, but this time y might not the equal to x.)
t_ameya
2021-03-12 19:24:05
32
32
416554
2021-03-12 19:24:05
32
32
happyhari
2021-03-12 19:24:05
32
32
sugar_rush
2021-03-12 19:24:05
same
same
peace09
2021-03-12 19:24:05
32?
32?
coolotter
2021-03-12 19:24:05
32 again
32 again
RohanQV
2021-03-12 19:24:05
32 also!
32 also!
BillXu21
2021-03-12 19:24:05
32
32
s-sawen-07
2021-03-12 19:24:05
same?
same?
MT-Math
2021-03-12 19:24:05
32
32
happyhari
2021-03-12 19:24:05
32, as well
32, as well
volcanogobbler
2021-03-12 19:24:05
32?
32?
ancientwarrior
2021-03-12 19:24:05
the same
the same
MaryM
2021-03-12 19:24:07
By symmetry, it's exactly the same!
By symmetry, it's exactly the same!
MaryM
2021-03-12 19:24:11
So there are 32 solutions with x=z.
So there are 32 solutions with x=z.
MaryM
2021-03-12 19:24:15
And how many solutions with y=z?
And how many solutions with y=z?
Kesav
2021-03-12 19:24:28
32
32
Lilavigne
2021-03-12 19:24:28
32
32
happyhari
2021-03-12 19:24:28
32
32
kante314
2021-03-12 19:24:28
32
32
PiMasterMC
2021-03-12 19:24:28
32
32
awesomeandy
2021-03-12 19:24:28
32
32
peace09
2021-03-12 19:24:28
32, As Well!
32, As Well!
RohanQV
2021-03-12 19:24:28
32 also!
32 also!
s-sawen-07
2021-03-12 19:24:28
again .. same?
again .. same?
Z_Lu
2021-03-12 19:24:28
32
32
Irving1004
2021-03-12 19:24:28
32
32
MaryM
2021-03-12 19:24:31
Same: 32.
Same: 32.
MaryM
2021-03-12 19:24:34
So that gives us 32+32+32 = 96 solutions with two piles the same.
So that gives us 32+32+32 = 96 solutions with two piles the same.
MaryM
2021-03-12 19:24:37
Right?
Right?
Professor-Mom
2021-03-12 19:25:06
nppe
nppe
yayy
2021-03-12 19:25:06
overcount
overcount
peace09
2021-03-12 19:25:06
Wait! Some Cases Overlap!
Wait! Some Cases Overlap!
iiRishabii
2021-03-12 19:25:06
no!
no!
Kesav
2021-03-12 19:25:06
no we are overcounting
no we are overcounting
happyhari
2021-03-12 19:25:06
no, we are overcounting, what if they are all the same
no, we are overcounting, what if they are all the same
Professor-Mom
2021-03-12 19:25:06
No
No
Lilavigne
2021-03-12 19:25:06
no overcount
no overcount
MT-Math
2021-03-12 19:25:06
we overcounted kinda
we overcounted kinda
Irving1004
2021-03-12 19:25:06
no
no
Z_Lu
2021-03-12 19:25:06
no, overcounting
no, overcounting
MaryM
2021-03-12 19:25:09
Not quite.
Not quite.
MaryM
2021-03-12 19:25:13
What about the solutions with all three piles the same?
What about the solutions with all three piles the same?
MaryM
2021-03-12 19:25:17
Can this happen?
Can this happen?
mm999aops
2021-03-12 19:25:51
yes-22+22+22
yes-22+22+22
peace09
2021-03-12 19:25:51
Yes... x=y=z=22.
Yes... x=y=z=22.
coolotter
2021-03-12 19:25:51
yes, when all are 22
yes, when all are 22
yayy
2021-03-12 19:25:51
yes, (all 22)
yes, (all 22)
happyhari
2021-03-12 19:25:51
yes, if x = y = z = 22
yes, if x = y = z = 22
Arrowhead575
2021-03-12 19:25:51
22,22,22
22,22,22
sugar_rush
2021-03-12 19:25:51
when all three have 22
when all three have 22
Lilavigne
2021-03-12 19:25:51
yes when x=y=z=22
yes when x=y=z=22
PiMasterMC
2021-03-12 19:25:51
all are eq to 22
all are eq to 22
skyleristhecoolest
2021-03-12 19:25:51
22 22 22
22 22 22
Irving1004
2021-03-12 19:25:51
yes
yes
MT-Math
2021-03-12 19:25:51
yes, if x, y, and z all have 22
yes, if x, y, and z all have 22
MaryM
2021-03-12 19:25:53
Yes, but only in one way: all three piles must have 22 coins.
Yes, but only in one way: all three piles must have 22 coins.
MaryM
2021-03-12 19:25:59
But how many times have we counted this solution in our count of 96?
But how many times have we counted this solution in our count of 96?
PiMasterMC
2021-03-12 19:26:24
3
3
coolotter
2021-03-12 19:26:24
3 times
3 times
t_ameya
2021-03-12 19:26:24
3?
3?
volcanogobbler
2021-03-12 19:26:24
3 times!
3 times!
Arrowhead575
2021-03-12 19:26:24
3
3
happyhari
2021-03-12 19:26:24
3 times
3 times
s-sawen-07
2021-03-12 19:26:24
3
3
peace09
2021-03-12 19:26:24
3 Times
3 Times
Irving1004
2021-03-12 19:26:24
3 times
3 times
BillXu21
2021-03-12 19:26:24
3 times
3 times
MaryM
2021-03-12 19:26:28
We've counted it three times: once for each pair of piles.
We've counted it three times: once for each pair of piles.
MaryM
2021-03-12 19:26:31
So what do we need to do?
So what do we need to do?
s-sawen-07
2021-03-12 19:27:05
so we must get rid of 2 times
so we must get rid of 2 times
mm999aops
2021-03-12 19:27:05
minues two times
minues two times
Summer2019
2021-03-12 19:27:05
subtract
subtract
peace09
2021-03-12 19:27:05
Subtract 2 Pairs
Subtract 2 Pairs
happyhari
2021-03-12 19:27:05
subtract 2(1) = 2
subtract 2(1) = 2
Lilavigne
2021-03-12 19:27:05
subtract it twice
subtract it twice
awesomeandy
2021-03-12 19:27:05
subtract 2
subtract 2
happyhari
2021-03-12 19:27:05
subtract 2
subtract 2
coolotter
2021-03-12 19:27:05
add 2 back!
add 2 back!
functionalmath
2021-03-12 19:27:05
subtract 2
subtract 2
PiMasterMC
2021-03-12 19:27:05
subtract 2 from the total
subtract 2 from the total
dolphindash
2021-03-12 19:27:05
subtract 2
subtract 2
happyhari
2021-03-12 19:27:05
96 - 2 = 94
96 - 2 = 94
MaryM
2021-03-12 19:27:07
We need to subtract 2 from our count of 96, to account for the fact that the (22,22,22) solution was counted three times, and we only want to count it once.
We need to subtract 2 from our count of 96, to account for the fact that the (22,22,22) solution was counted three times, and we only want to count it once.
MaryM
2021-03-12 19:27:11
Therefore, there are 96 - 2 = 94 solutions with two (or all three) piles the same.
Therefore, there are 96 - 2 = 94 solutions with two (or all three) piles the same.
MaryM
2021-03-12 19:27:15
How does that help?
How does that help?
Arrowhead575
2021-03-12 19:28:36
Subtract from original amount
Subtract from original amount
mm999aops
2021-03-12 19:28:36
we can subtract that from the total number
we can subtract that from the total number
Kesav
2021-03-12 19:28:36
we do 2080 - 94 = 1986!
we do 2080 - 94 = 1986!
sugar_rush
2021-03-12 19:28:36
subtract from 2080
subtract from 2080
mm999aops
2021-03-12 19:28:36
the total is 65*32, so we want 65*32-94?
the total is 65*32, so we want 65*32-94?
functionalmath
2021-03-12 19:28:36
subtract 94 illegal cases from the 2080 legal ones
subtract 94 illegal cases from the 2080 legal ones
t_ameya
2021-03-12 19:28:36
this is the amount of cases that don't work
this is the amount of cases that don't work
RohanQV
2021-03-12 19:28:36
Helps remove overcounting cases
Helps remove overcounting cases
Montclarion6
2021-03-12 19:28:36
2080-94
2080-94
MaryM
2021-03-12 19:28:44
That means there are \dbinom{65}{2} - 94 solutions with all three piles different.
That means there are \dbinom{65}{2} - 94 solutions with all three piles different.
MaryM
2021-03-12 19:28:51
And now what?
And now what?
MaryM
2021-03-12 19:28:55
How many of these will have x < y < z?
How many of these will have x < y < z?
yayy
2021-03-12 19:29:27
the 6 ways of ordering x, y, z are symetrical, so we can divide by 6
the 6 ways of ordering x, y, z are symetrical, so we can divide by 6
nathanqiu
2021-03-12 19:29:27
then divide by 6
then divide by 6
nathanqiu
2021-03-12 19:29:27
divide by 6 to account for overcount
divide by 6 to account for overcount
biketoday
2021-03-12 19:29:27
1 out of every 6
1 out of every 6
rockyrockrock
2021-03-12 19:29:27
we subtract and divide by 6 !
we subtract and divide by 6 !
Lilavigne
2021-03-12 19:29:27
1/6 of them
1/6 of them
coolotter
2021-03-12 19:29:27
WE ARE NOT DONE, divide by 6 now since 3! = six
WE ARE NOT DONE, divide by 6 now since 3! = six
PiMasterMC
2021-03-12 19:29:27
the large number divide by 6
the large number divide by 6
functionalmath
2021-03-12 19:29:27
divide by 3!=6 because ordering
divide by 3!=6 because ordering
yuanyuanC
2021-03-12 19:29:27
1/6 of them?
1/6 of them?
MaryM
2021-03-12 19:29:30
Here there's a clever observation.
Here there's a clever observation.
MaryM
2021-03-12 19:29:49
If we have a solution with all three piles different, x,y,z could be in any order. And all the orders are equally likely!
If we have a solution with all three piles different, x,y,z could be in any order. And all the orders are equally likely!
MaryM
2021-03-12 19:29:58
That is, if we pick a random solution, we could have x,y,z in any of the 3! = 6 possible orders of smallest, medium, largest.
That is, if we pick a random solution, we could have x,y,z in any of the 3! = 6 possible orders of smallest, medium, largest.
MaryM
2021-03-12 19:30:04
So the likelihood that x < y < z (the order we want) is the order we get is \dfrac16.
So the likelihood that x < y < z (the order we want) is the order we get is \dfrac16.
MaryM
2021-03-12 19:30:11
Therefore, exactly \dfrac16 of our "all three piles different" solutions have x smallest, y middle, and z largest.
Therefore, exactly \dfrac16 of our "all three piles different" solutions have x smallest, y middle, and z largest.
MaryM
2021-03-12 19:30:19
So our final count is \dfrac{\dbinom{65}{2}-94}{6}.
So our final count is \dfrac{\dbinom{65}{2}-94}{6}.
MaryM
2021-03-12 19:30:22
And now we compute!
And now we compute!
MaryM
2021-03-12 19:30:26
What is \dbinom{65}{2}?
What is \dbinom{65}{2}?
functionalmath
2021-03-12 19:30:53
2080
2080
kante314
2021-03-12 19:30:53
2008
2008
ancientwarrior
2021-03-12 19:30:53
2080
2080
b20081
2021-03-12 19:30:53
2080
2080
skyleristhecoolest
2021-03-12 19:30:53
2080
2080
Arrowhead575
2021-03-12 19:30:53
2080
2080
jxwis2010
2021-03-12 19:30:53
2080
2080
GabeW1234
2021-03-12 19:30:53
65*32=2080
65*32=2080
yuanyuanC
2021-03-12 19:30:53
2080
2080
PiMasterMC
2021-03-12 19:30:53
2080
2080
MaryM
2021-03-12 19:30:55
It's \dfrac{65 \cdot 64}{2} = 65 \cdot 32 = 2080.
It's \dfrac{65 \cdot 64}{2} = 65 \cdot 32 = 2080.
MaryM
2021-03-12 19:31:00
So our answer is \dfrac{2080-94}{6}.
So our answer is \dfrac{2080-94}{6}.
MaryM
2021-03-12 19:31:09
What is this equal to?
What is this equal to?
coolotter
2021-03-12 19:31:32
so our final solution is (2080-96+2)/6 = 1986/6 = \boxed{331}!
so our final solution is (2080-96+2)/6 = 1986/6 = \boxed{331}!
Kesav
2021-03-12 19:31:32
331
331
yuanyuanC
2021-03-12 19:31:32
331
331
happyhari
2021-03-12 19:31:32
331
331
sugar_rush
2021-03-12 19:31:32
\boxed{331} is our final answer!
\boxed{331} is our final answer!
PiMasterMC
2021-03-12 19:31:32
331
331
b20081
2021-03-12 19:31:32
331
331
GabeW1234
2021-03-12 19:31:32
331
331
mm999aops
2021-03-12 19:31:32
1986/6 = 331
1986/6 = 331

uneducatedpotato
2021-03-12 19:31:32
331?
331?
MaryM
2021-03-12 19:31:34
This simplifies to \dfrac{1986}{6} = \boxed{331}.
This simplifies to \dfrac{1986}{6} = \boxed{331}.
MaryM
2021-03-12 19:31:43
5. Call a three-term strictly increasing arithmetic sequence of integers special if the sum of the squares of the three terms equals the product of the middle term and the square of the common difference. Find the sum of the third terms of all special sequences.
5. Call a three-term strictly increasing arithmetic sequence of integers special if the sum of the squares of the three terms equals the product of the middle term and the square of the common difference. Find the sum of the third terms of all special sequences.
MaryM
2021-03-12 19:32:00
You might be inclined to write a 3-term arithmetic sequence as a, a+d, a+2d for initial term a and common difference d.
You might be inclined to write a 3-term arithmetic sequence as a, a+d, a+2d for initial term a and common difference d.
MaryM
2021-03-12 19:32:03
Does anybody have another suggestion?
Does anybody have another suggestion?
Mathaddict3825
2021-03-12 19:32:25
a-d, a, a+d
a-d, a, a+d
floradaisy136
2021-03-12 19:32:25
a-d, a, a+d
a-d, a, a+d
helloworld_
2021-03-12 19:32:25
a - d, a, a + d
a - d, a, a + d
sugar_rush
2021-03-12 19:32:25
x-d, x, x+d
x-d, x, x+d
yayy
2021-03-12 19:32:25
a-r, a, a+r
a-r, a, a+r
mathicorn
2021-03-12 19:32:25
write as a-d, a, a+d?
write as a-d, a, a+d?
Asher51
2021-03-12 19:32:25
b-d, b, and b+d
b-d, b, and b+d
kante314
2021-03-12 19:32:25
Let the middle number be a
Let the middle number be a
PiMasterMC
2021-03-12 19:32:25
a-d, a, a + d
a-d, a, a + d
CircleInvert
2021-03-12 19:32:27
Use a-d, a, and a+d
Use a-d, a, and a+d
MaryM
2021-03-12 19:32:29
Yes, I find that when we're working with arithmetic sequences, especially of odd length, we can take advantage of some symmetry by focusing on the middle term.
Yes, I find that when we're working with arithmetic sequences, especially of odd length, we can take advantage of some symmetry by focusing on the middle term.
MaryM
2021-03-12 19:32:32
That is, let's write our special sequence as a-d, a, a+d with middle term a and common difference d.
That is, let's write our special sequence as a-d, a, a+d with middle term a and common difference d.
MaryM
2021-03-12 19:32:39
Hopefully this will provide some advantage to us later.
Hopefully this will provide some advantage to us later.
MaryM
2021-03-12 19:32:45
Let's write down the condition for this sequence to be special.
Let's write down the condition for this sequence to be special.
MaryM
2021-03-12 19:32:50
In words, we have \text{sum of squares of all three terms} = (\text{middle term}) \cdot (\text{square of the common difference}).
In words, we have \text{sum of squares of all three terms} = (\text{middle term}) \cdot (\text{square of the common difference}).
MaryM
2021-03-12 19:32:56
What does this give us?
What does this give us?
pinkpig
2021-03-12 19:33:38
ad^2 = 3a^2+2d^2
ad^2 = 3a^2+2d^2
peace09
2021-03-12 19:33:38
(a-d)^2+a^2+(a+d)^2=3a^2+2d^2=ad^2.
(a-d)^2+a^2+(a+d)^2=3a^2+2d^2=ad^2.
CircleInvert
2021-03-12 19:33:38
3a^2+2d^2=ad^2
3a^2+2d^2=ad^2
Irving1004
2021-03-12 19:33:38
well (a-b)^2 + (a+b)^2 = 2a^2 + 2b^2
well (a-b)^2 + (a+b)^2 = 2a^2 + 2b^2
Summer2019
2021-03-12 19:33:38
3a^2+2d^2=ad^2
3a^2+2d^2=ad^2
kante314
2021-03-12 19:33:38
(a-d)^2 + a^2 + (a+d)^2 = a x d^2
(a-d)^2 + a^2 + (a+d)^2 = a x d^2
t_ameya
2021-03-12 19:33:38
a^2+(a-d)^2+(a+d)^2=a\cdotd^2
a^2+(a-d)^2+(a+d)^2=a\cdotd^2
Kesav
2021-03-12 19:33:38
3a^2 + 2d^2 = ad
3a^2 + 2d^2 = ad
Asher51
2021-03-12 19:33:38
a^2 + (a-d)^2 + (a+d)^2 = ad^2
a^2 + (a-d)^2 + (a+d)^2 = ad^2
pinkpig
2021-03-12 19:33:38
ad^2=3a^2+2d^2
ad^2=3a^2+2d^2
MaryM
2021-03-12 19:33:41
We get (a-d)^2 + a^2 + (a+d)^2 = ad^2.
We get (a-d)^2 + a^2 + (a+d)^2 = ad^2.
MaryM
2021-03-12 19:34:06
Which simplifies to 3a^2 + 2d^2 = ad^2.
Which simplifies to 3a^2 + 2d^2 = ad^2.
MaryM
2021-03-12 19:34:10
(Note that the +2ad from the first square cancels with the -2ad from the third square! This is why writing the sequence symmetrically is useful.)
(Note that the +2ad from the first square cancels with the -2ad from the third square! This is why writing the sequence symmetrically is useful.)
MaryM
2021-03-12 19:34:15
Now what?
Now what?
peace09
2021-03-12 19:35:05
3a^2=ad^2-2d^2, Implying d^2=\tfrac{3a^2}{a-2}.
3a^2=ad^2-2d^2, Implying d^2=\tfrac{3a^2}{a-2}.
iiRishabii
2021-03-12 19:35:05
3a^2=ad^2-2d^2
3a^2=d(ad-2d)
3a^2=ad^2-2d^2
3a^2=d(ad-2d)
sugar_rush
2021-03-12 19:35:05
subtract off 2d^{2} from each side
subtract off 2d^{2} from each side
mm999aops
2021-03-12 19:35:05
\frac{3a^2}{a-2}=d^2
\frac{3a^2}{a-2}=d^2
Lilavigne
2021-03-12 19:35:09
isolate d^2
isolate d^2
MaryM
2021-03-12 19:35:12
We can try isolating the variable d.
We can try isolating the variable d.
MaryM
2021-03-12 19:35:16
We can write 3a^2 = ad^2 - 2d^2, so
(a - 2) d^2 = 3a^2.
We can write 3a^2 = ad^2 - 2d^2, so
(a - 2) d^2 = 3a^2.
MaryM
2021-03-12 19:35:24
What does this equation say about a?
What does this equation say about a?
MaryM
2021-03-12 19:35:51
Hint: think about divisibility.
Hint: think about divisibility.
sugar_rush
2021-03-12 19:36:40
a-2 divides 3a^{2}?
a-2 divides 3a^{2}?
pinkpig
2021-03-12 19:36:40
3/(a-2) is a perfect square because a^2 is already a perfect square
3/(a-2) is a perfect square because a^2 is already a perfect square
Irving1004
2021-03-12 19:36:40
a-2 divides into 3a^2 and this is a perfect square
a-2 divides into 3a^2 and this is a perfect square
pinkpig
2021-03-12 19:36:40
a-2 divides 3a^2
a-2 divides 3a^2
Unevenlogic
2021-03-12 19:36:40
(a-2) | 3a^2
(a-2) | 3a^2
MaryM
2021-03-12 19:36:43
This equation says that a - 2 divides 3a^2. That seems like it will narrow down the possible values of a.
This equation says that a - 2 divides 3a^2. That seems like it will narrow down the possible values of a.
MaryM
2021-03-12 19:36:48
We could try dividing 3a^2 by a - 2 using long division, but a faster way is to use a substitution.
We could try dividing 3a^2 by a - 2 using long division, but a faster way is to use a substitution.
MaryM
2021-03-12 19:36:54
Let b = a - 2. Then a = b + 2, so
bd^2 = 3(b + 2)^2 = 3(b^2 + 4b + 4) = 3b^2 + 12b + 12.
Let b = a - 2. Then a = b + 2, so
bd^2 = 3(b + 2)^2 = 3(b^2 + 4b + 4) = 3b^2 + 12b + 12.
MaryM
2021-03-12 19:37:19
What does this equation say about b?
What does this equation say about b?
MaryM
2021-03-12 19:37:58
Do you know a number that is a multiple of b by looking at that equation?
Do you know a number that is a multiple of b by looking at that equation?
math9990
2021-03-12 19:38:17
b divides 12
b divides 12
pinkpig
2021-03-12 19:38:17
b divides 12
b divides 12
Unevenlogic
2021-03-12 19:38:17
b | 3b^2 + 12b + 12
b | 3b^2 + 12b + 12
sugar_rush
2021-03-12 19:38:17
b divides 12 because it divides 3b^{2}+12b+12
b divides 12 because it divides 3b^{2}+12b+12
pinkpig
2021-03-12 19:38:17
After you simplify it, you get that b is divisible by 12
After you simplify it, you get that b is divisible by 12
PiMasterMC
2021-03-12 19:38:17
12?
12?
MaryM
2021-03-12 19:38:20
Since b divides the left-hand side, b divides the right-hand side. Also, b divides 3b^2 and 12b, so b must divide 12.
Since b divides the left-hand side, b divides the right-hand side. Also, b divides 3b^2 and 12b, so b must divide 12.
MaryM
2021-03-12 19:38:27
And from the equation (a - 2) d^2 = 3a^2, b = a - 2 must be positive.
And from the equation (a - 2) d^2 = 3a^2, b = a - 2 must be positive.
MaryM
2021-03-12 19:38:31
At this point, we could go through the divisors of 12, plug them into the equation
d^2 = \frac{3(b + 2)^2}{b},
and see which values lead to squares.
At this point, we could go through the divisors of 12, plug them into the equation
d^2 = \frac{3(b + 2)^2}{b},
and see which values lead to squares.
MaryM
2021-03-12 19:38:43
But we can narrow down the possible values even further, by writing the equation bd^2 = 3(b + 2)^2 as
3bd^2 = 9(b + 2)^2.
But we can narrow down the possible values even further, by writing the equation bd^2 = 3(b + 2)^2 as
3bd^2 = 9(b + 2)^2.
MaryM
2021-03-12 19:38:48
What does this equation say about b?
What does this equation say about b?
MaryM
2021-03-12 19:39:13
Hint: look at all those perfect squares!
Hint: look at all those perfect squares!
GabeW1234
2021-03-12 19:40:11
3b is a square
3b is a square
Montclarion6
2021-03-12 19:40:11
b=3k^2
b=3k^2
pinkpig
2021-03-12 19:40:11
b is three times a perfect square
b is three times a perfect square
math9990
2021-03-12 19:40:11
3b is a perfect square
3b is a perfect square
Rubikscube3.1415
2021-03-12 19:40:11
3b is a perfect square
3b is a perfect square
Lilavigne
2021-03-12 19:40:11
b=3*perfect square
b=3*perfect square
MaryM
2021-03-12 19:40:17
Since d^2 and 9(b + 2)^2 are perfect squares, 3b must be a perfect square. ( This is exactly the same as saying that b is three times a perfect square).
Since d^2 and 9(b + 2)^2 are perfect squares, 3b must be a perfect square. ( This is exactly the same as saying that b is three times a perfect square).
MaryM
2021-03-12 19:40:24
Which divisors b of 12 have the property that 3b is a perfect square?
Which divisors b of 12 have the property that 3b is a perfect square?
pinkpig
2021-03-12 19:41:25
3 and 12
3 and 12
sugar_rush
2021-03-12 19:41:25
3 and 12 itself
3 and 12 itself
foodisgood
2021-03-12 19:41:25
3,12
3,12
Summer2019
2021-03-12 19:41:25
b=3, 12
b=3, 12
peace09
2021-03-12 19:41:25
3 and 12.
3 and 12.
Arrowhead575
2021-03-12 19:41:25
3,12
3,12
yayy
2021-03-12 19:41:25
3, 12
3, 12
mm999aops
2021-03-12 19:41:25
3 12
3 12
Unevenlogic
2021-03-12 19:41:25
3 and 12
3 and 12
MaryM
2021-03-12 19:41:27
Only b = 3 and b = 12 have this property.
Only b = 3 and b = 12 have this property.
MaryM
2021-03-12 19:41:31
So, a = 5 or a = 14.
So, a = 5 or a = 14.
MaryM
2021-03-12 19:41:36
If a = 5, then d^2 = \dfrac{3 \cdot 5^2}{3} = 25. Then what is d?
If a = 5, then d^2 = \dfrac{3 \cdot 5^2}{3} = 25. Then what is d?
BillXu21
2021-03-12 19:42:00
5
5

peace09
2021-03-12 19:42:00
d=5.
d=5.
GabeW1234
2021-03-12 19:42:00
5
5
uneducatedpotato
2021-03-12 19:42:00
5
5
mm999aops
2021-03-12 19:42:00
d is 5 because it's incresaing
d is 5 because it's incresaing
Euler1728
2021-03-12 19:42:00
5
5
coolotter
2021-03-12 19:42:00
d = 5
d = 5
foodisgood
2021-03-12 19:42:00
5
5
MaryM
2021-03-12 19:42:02
If d^2 = 25, then d = 5. The value of d must be positive because the sequence is strictly increasing.
If d^2 = 25, then d = 5. The value of d must be positive because the sequence is strictly increasing.
MaryM
2021-03-12 19:42:07
Similarly, if a = 14, then d^2 = \dfrac{3 \cdot 14^2}{12} = 49, so d = 7.
Similarly, if a = 14, then d^2 = \dfrac{3 \cdot 14^2}{12} = 49, so d = 7.
MaryM
2021-03-12 19:42:11
So what is the sum of all possible third terms?
So what is the sum of all possible third terms?
pinkpig
2021-03-12 19:42:52
031
031
iiRishabii
2021-03-12 19:42:52
31
31
Unevenlogic
2021-03-12 19:42:52
10 + 21 = 31
10 + 21 = 31
pinkpig
2021-03-12 19:42:52
10+21=31
10+21=31
Lilavigne
2021-03-12 19:42:52
31
31
sugar_rush
2021-03-12 19:42:52
10+21=\boxed{31}!
10+21=\boxed{31}!
foodisgood
2021-03-12 19:42:52
31
31
coolotter
2021-03-12 19:42:52
we can have 0 5 10 or 7 14 21 so we get 10 + 21 = \boxed{31}!
we can have 0 5 10 or 7 14 21 so we get 10 + 21 = \boxed{31}!
Kesav
2021-03-12 19:42:52
31
31
functionalmath
2021-03-12 19:42:52
31
31
Z_Lu
2021-03-12 19:42:52
31
31
MaryM
2021-03-12 19:42:54
The sum of all possible third terms is (5 + 5) + (14 + 7) = 10 + 21 = \boxed{031}.
The sum of all possible third terms is (5 + 5) + (14 + 7) = 10 + 21 = \boxed{031}.
BillXu21
2021-03-12 19:43:02

BillXu21
2021-03-12 19:43:02
yay
yay
MaryM
2021-03-12 19:43:05
6. Segments \overline{AB},\, \overline{AC},\, and \overline{AD} are edges of a cube and segment \overline{AG} is a diagonal through the center of the cube. Point P satisfies BP=60\sqrt{10},\, CP=60\sqrt5,\, DP=120\sqrt2,\, and GP=36\sqrt7. Find AP.
6. Segments \overline{AB},\, \overline{AC},\, and \overline{AD} are edges of a cube and segment \overline{AG} is a diagonal through the center of the cube. Point P satisfies BP=60\sqrt{10},\, CP=60\sqrt5,\, DP=120\sqrt2,\, and GP=36\sqrt7. Find AP.
MaryM
2021-03-12 19:43:15
Any ideas?
Any ideas?
Lilavigne
2021-03-12 19:44:03
coordinates?
coordinates?
Summer2019
2021-03-12 19:44:03
coordinates
coordinates
functionalmath
2021-03-12 19:44:03
put it on the coordinate plane
put it on the coordinate plane
BillXu21
2021-03-12 19:44:03
coordinate
coordinate
bbmmall
2021-03-12 19:44:03
coordinate bash
coordinate bash
b20081
2021-03-12 19:44:03
coordinate bash
coordinate bash
math9990
2021-03-12 19:44:03
coordinates, then calculate BP CP DP GP in terms of the coordinates, then you can use that systems of equations to find AP
coordinates, then calculate BP CP DP GP in terms of the coordinates, then you can use that systems of equations to find AP
MaryM
2021-03-12 19:44:05
We want to compute a bunch of distances, so it may be helpful to work with coordinates.
We want to compute a bunch of distances, so it may be helpful to work with coordinates.
MaryM
2021-03-12 19:44:11
How can we set the coordinates up?
How can we set the coordinates up?
GabeW1234
2021-03-12 19:44:54
a= (0,0,0)?
a= (0,0,0)?
BillXu21
2021-03-12 19:44:54
a is the origin\
a is the origin\
ancientwarrior
2021-03-12 19:44:54
Let A be the origin?
Let A be the origin?
sugar_rush
2021-03-12 19:44:54
A=\text{origin}
A=\text{origin}
functionalmath
2021-03-12 19:44:54
A is the origin, then set up the axis such that 3 edges are the axis
A is the origin, then set up the axis such that 3 edges are the axis
dbasu1
2021-03-12 19:44:54
A is the origin
A is the origin
pinkpig
2021-03-12 19:44:54
we could let a be the origin
we could let a be the origin
MaryM
2021-03-12 19:44:57
I'd probably make A = (0,0,0) be the origin of 3-D space.
I'd probably make A = (0,0,0) be the origin of 3-D space.
MaryM
2021-03-12 19:45:06
We're not told the side length of the cube.
We're not told the side length of the cube.
MaryM
2021-03-12 19:45:10
So let's call it s.
So let's call it s.
MaryM
2021-03-12 19:45:14
Then what are good choices for B,C,D?
Then what are good choices for B,C,D?
math9990
2021-03-12 19:46:08
(s,0,0) (0,s,0) (0,0,x)
(s,0,0) (0,s,0) (0,0,x)
GabeW1234
2021-03-12 19:46:08
(0,0,s),(0,s,0),(s,0,0)
(0,0,s),(0,s,0),(s,0,0)
yayy
2021-03-12 19:46:08
(s, 0, 0), (0, s, 0), (0, 0, s)
(s, 0, 0), (0, s, 0), (0, 0, s)
ancientwarrior
2021-03-12 19:46:08
(s,0,0) (0,s,0) (0,0,s)
(s,0,0) (0,s,0) (0,0,s)
functionalmath
2021-03-12 19:46:08
(s,0,0), (0,s,0) and (0,0,s)
(s,0,0), (0,s,0) and (0,0,s)
pinkpig
2021-03-12 19:46:08
B would be (s,0,0), C would be (0,s,0,) and D would be (0,0,s)
B would be (s,0,0), C would be (0,s,0,) and D would be (0,0,s)
BillXu21
2021-03-12 19:46:08
b = (x,0,0) c = (0,s,0) d = (0,0,s) g = (s,s,s)
b = (x,0,0) c = (0,s,0) d = (0,0,s) g = (s,s,s)
regulareagle
2021-03-12 19:46:08
(s,0,0) (0,s,0) and (0,0,s)
(s,0,0) (0,s,0) and (0,0,s)
sugar_rush
2021-03-12 19:46:08
B=(s, 0, 0), C=(0, s, 0), D=(0, 0, s)
B=(s, 0, 0), C=(0, s, 0), D=(0, 0, s)
foodisgood
2021-03-12 19:46:08
(0,s,0), (s,0,0) and (0,0,s)?
(0,s,0), (s,0,0) and (0,0,s)?
MaryM
2021-03-12 19:46:15
We can set B = (s,0,0), C = (0,s,0), and D = (0,0,s).
We can set B = (s,0,0), C = (0,s,0), and D = (0,0,s).
MaryM
2021-03-12 19:46:22
Then what is G?
Then what is G?
sugar_rush
2021-03-12 19:46:51
G=(s, s, s)
G=(s, s, s)
pinkpig
2021-03-12 19:46:51
G would be (s,s,s)
G would be (s,s,s)
BillXu21
2021-03-12 19:46:51
(s,s,s)
(s,s,s)
ancientwarrior
2021-03-12 19:46:51
(s, s, s)
(s, s, s)
Irving1004
2021-03-12 19:46:51
(s, s, s)
(s, s, s)
GabeW1234
2021-03-12 19:46:51
(s,s,s)
(s,s,s)
Lilavigne
2021-03-12 19:46:51
(s,s,s)
(s,s,s)
Asher51
2021-03-12 19:46:51
(s,s,s)
(s,s,s)
foodisgood
2021-03-12 19:46:51
(s,s,s,)
(s,s,s,)
Summer2019
2021-03-12 19:46:51
G=(s,s,s)
G=(s,s,s)
dolphindash
2021-03-12 19:46:51
(s,s,s)
(s,s,s)
awesomeandy
2021-03-12 19:46:51
(s,s,s)
(s,s,s)
MaryM
2021-03-12 19:46:53
We have G = (s,s,s).
We have G = (s,s,s).
MaryM
2021-03-12 19:46:56
And what about P?
And what about P?
Irving1004
2021-03-12 19:47:25
(a, b, c)
(a, b, c)
sugar_rush
2021-03-12 19:47:25
let's call it (p, q, r)
let's call it (p, q, r)
Unevenlogic
2021-03-12 19:47:25
(a, b, c)
(a, b, c)
BillXu21
2021-03-12 19:47:25
(x,y,z)
(x,y,z)
Summer2019
2021-03-12 19:47:25
(p,q,r)
(p,q,r)
functionalmath
2021-03-12 19:47:25
(x_0,y_0,z_0)
(x_0,y_0,z_0)
pinkpig
2021-03-12 19:47:25
we should let P be (x,y,z)
we should let P be (x,y,z)
yayy
2021-03-12 19:47:25
(x, y, z)
(x, y, z)
MaryM
2021-03-12 19:47:28
We don't really know where P is, so let's just say P = (x,y,z).
We don't really know where P is, so let's just say P = (x,y,z).
MaryM
2021-03-12 19:47:32
Now what?
Now what?
sugar_rush
2021-03-12 19:47:57
3d distance formula bash!
3d distance formula bash!
BillXu21
2021-03-12 19:47:57
distance formula for 3D space
distance formula for 3D space
pinkpig
2021-03-12 19:47:57
set up a system of equations
set up a system of equations
Summer2019
2021-03-12 19:47:57
distance formula with coordinates
distance formula with coordinates
yayy
2021-03-12 19:47:57
equations
equations
Z_Lu
2021-03-12 19:47:57
make uses of the distances to find P
make uses of the distances to find P
Unevenlogic
2021-03-12 19:47:57
Use the 3d Pythagorean theorem to form equations
Use the 3d Pythagorean theorem to form equations
functionalmath
2021-03-12 19:47:57
use the distances given to set up a system of 4 by 4 equations and solve
use the distances given to set up a system of 4 by 4 equations and solve
MaryM
2021-03-12 19:48:01
Now we can write equations for the distances.
Now we can write equations for the distances.
MaryM
2021-03-12 19:48:05
For instance, what is BP?
For instance, what is BP?
MaryM
2021-03-12 19:48:52
(In terms of x,y,z and s, I mean.)
(In terms of x,y,z and s, I mean.)
functionalmath
2021-03-12 19:49:22
(x-s)^2+y^2+z^2
(x-s)^2+y^2+z^2
sugar_rush
2021-03-12 19:49:22
PB^{2}=p^{2}+(q-s)^{2}+r^{2}
PB^{2}=p^{2}+(q-s)^{2}+r^{2}
pinkpig
2021-03-12 19:49:22
(x-s)^2+(y^2)+z^2 = 36000
(x-s)^2+(y^2)+z^2 = 36000
ancientwarrior
2021-03-12 19:49:22
\sqrt{(x-s)^2+y^2+z^2}
\sqrt{(x-s)^2+y^2+z^2}
foodisgood
2021-03-12 19:49:22
sqrt[(x-s)^2+y^2+z^2]
sqrt[(x-s)^2+y^2+z^2]
Lilavigne
2021-03-12 19:49:22
sqrt{(x-s)^2+y^2+z^2}
sqrt{(x-s)^2+y^2+z^2}
Summer2019
2021-03-12 19:49:22
x^2+(y-s)^2+z^2=(60sqrt(10))^2
x^2+(y-s)^2+z^2=(60sqrt(10))^2
yayy
2021-03-12 19:49:22
\sqrt[3]{(s-x)^2 + y^2 + z^2}
\sqrt[3]{(s-x)^2 + y^2 + z^2}
Irving1004
2021-03-12 19:49:22
sqrt((x-s)^2 + y^2 + z^2)
sqrt((x-s)^2 + y^2 + z^2)
sugar_rush
2021-03-12 19:49:22
PB=\sqrt{(p-0)^2+(q-s)^2+(r-0)^2}
PB=\sqrt{(p-0)^2+(q-s)^2+(r-0)^2}
BillXu21
2021-03-12 19:49:22
(x-s)^2+y^2+z^2 = 36000
(x-s)^2+y^2+z^2 = 36000
GabeW1234
2021-03-12 19:49:22
\sqrt{(x-s)^2+(y-0)^2+(z-0)^2}
\sqrt{(x-s)^2+(y-0)^2+(z-0)^2}
Asher51
2021-03-12 19:49:22
\sqrt{(x-s)^2 + y^2 + z^2}
\sqrt{(x-s)^2 + y^2 + z^2}
MaryM
2021-03-12 19:49:26
BP is the distance from (s,0,0) to (x,y,z).
BP is the distance from (s,0,0) to (x,y,z).
MaryM
2021-03-12 19:49:30
So BP = \sqrt{(x-s)^2 + y^2 + z^2}.
So BP = \sqrt{(x-s)^2 + y^2 + z^2}.
MaryM
2021-03-12 19:49:33
Hence, we have the equation \sqrt{(x-s)^2 + y^2 + z^2} = 60\sqrt{10}.
Hence, we have the equation \sqrt{(x-s)^2 + y^2 + z^2} = 60\sqrt{10}.
MaryM
2021-03-12 19:49:37
Ick, that's going to be hard to work with. Any ideas?
Ick, that's going to be hard to work with. Any ideas?
math9990
2021-03-12 19:50:09
square it
square it
yayy
2021-03-12 19:50:09
square
square
Unevenlogic
2021-03-12 19:50:09
Square both sides
Square both sides
Summer2019
2021-03-12 19:50:09
square both sides
square both sides
happyhari
2021-03-12 19:50:09
square both sides?
square both sides?
dolphindash
2021-03-12 19:50:09
square it
square it
pinkpig
2021-03-12 19:50:09
we should square both sides of the equation and do the same for the rest of them to see if they can cancel out.
we should square both sides of the equation and do the same for the rest of them to see if they can cancel out.
Z_Lu
2021-03-12 19:50:09
sqaure both sides
sqaure both sides
MaryM
2021-03-12 19:50:12
Let's square it.
Let's square it.
MaryM
2021-03-12 19:50:15
Now we have (x-s)^2 + y^2 + z^2 = 60^2 \cdot 10.
Now we have (x-s)^2 + y^2 + z^2 = 60^2 \cdot 10.
MaryM
2021-03-12 19:50:19
Now what?
Now what?
GabeW1234
2021-03-12 19:51:01
do the same for CP, DP, and GP
do the same for CP, DP, and GP
Irving1004
2021-03-12 19:51:01
Find CP and DP and GP and find AP
Find CP and DP and GP and find AP
pinkpig
2021-03-12 19:51:01
do the same for the rest of them
do the same for the rest of them
BillXu21
2021-03-12 19:51:01
now write the rest of the equasions
now write the rest of the equasions
yayy
2021-03-12 19:51:01
set up equations for all of the other points
set up equations for all of the other points
Lilavigne
2021-03-12 19:51:01
do the same for all other segments
do the same for all other segments
Summer2019
2021-03-12 19:51:01
same with CP, DP, and GP
same with CP, DP, and GP
sugar_rush
2021-03-12 19:51:01
similarly for PC, PD, PG
similarly for PC, PD, PG
happyhari
2021-03-12 19:51:01
find other equations
find other equations
foodisgood
2021-03-12 19:51:01
set up the other equations?
set up the other equations?
Bananaman27
2021-03-12 19:51:01
do it for more lengths we know
do it for more lengths we know
MaryM
2021-03-12 19:51:04
We can write equations for the other three distances.
We can write equations for the other three distances.
MaryM
2021-03-12 19:51:08
When we do, we get four equations in total:
When we do, we get four equations in total:
MaryM
2021-03-12 19:51:14
\begin{align*} (x-s)^2 + y^2 + z^2 &= 60^2 \cdot 10, \\ x^2 + (y-s)^2 + z^2 &= 60^2 \cdot 5, \\ x^2 + y^2 + (z-s)^2 &= 120^2 \cdot 2, \\ (x-s)^2 + (y-s)^2 + (z-s)^2 &= 36^2 \cdot 7. \end{align*}
\begin{align*} (x-s)^2 + y^2 + z^2 &= 60^2 \cdot 10, \\ x^2 + (y-s)^2 + z^2 &= 60^2 \cdot 5, \\ x^2 + y^2 + (z-s)^2 &= 120^2 \cdot 2, \\ (x-s)^2 + (y-s)^2 + (z-s)^2 &= 36^2 \cdot 7. \end{align*}
MaryM
2021-03-12 19:51:19
It's four equations in four variables, but unfortunately they're quadratics, not linear.
It's four equations in four variables, but unfortunately they're quadratics, not linear.
MaryM
2021-03-12 19:51:26
Can we solve them nonetheless?
Can we solve them nonetheless?
MaryM
2021-03-12 19:51:33
Do we have to?
Do we have to?
Z2589013
2021-03-12 19:52:02
no
no
math-Passion
2021-03-12 19:52:02
No
No
jxwis2010
2021-03-12 19:52:02
no
no
yayy
2021-03-12 19:52:02
we just need x^2 + y^2 + z^2
we just need x^2 + y^2 + z^2
gracefulharmony
2021-03-12 19:52:02
No
No
Z_Lu
2021-03-12 19:52:02
no, since we are trying to find AP
no, since we are trying to find AP
MaryM
2021-03-12 19:52:05
Keep you eye on the ball! What are we trying to find?
Keep you eye on the ball! What are we trying to find?
pinkpig
2021-03-12 19:52:27
AP
AP
ancientwarrior
2021-03-12 19:52:27
we want \sqrt{x^2+y^2+z^2}, so maybe there's a way to manipulate
we want \sqrt{x^2+y^2+z^2}, so maybe there's a way to manipulate
sugar_rush
2021-03-12 19:52:27
the distance PA
the distance PA
awesomeandy
2021-03-12 19:52:27
AP
AP
happyhari
2021-03-12 19:52:27
AP
AP
Summer2019
2021-03-12 19:52:27
sqrt(x^2+y^2+z^2)
sqrt(x^2+y^2+z^2)
functionalmath
2021-03-12 19:52:27
sqrt(x^2+y^2+z^2)
sqrt(x^2+y^2+z^2)
Z_Lu
2021-03-12 19:52:27
we just need to find what x^2+y^2+z^2 will be equal to
we just need to find what x^2+y^2+z^2 will be equal to
BillXu21
2021-03-12 19:52:27
AP
AP
MaryM
2021-03-12 19:52:28
We want AP.
We want AP.
MaryM
2021-03-12 19:52:33
But AP = \sqrt{x^2 + y^2 + z^2}.
But AP = \sqrt{x^2 + y^2 + z^2}.
MaryM
2021-03-12 19:52:37
Can we find that quantity without solving the system?
Can we find that quantity without solving the system?
helloworld_
2021-03-12 19:53:14
No, subtract the first 3 all from the third.
No, subtract the first 3 all from the third.
math9990
2021-03-12 19:53:14
add up the first three and subtract the fourth from the sum
add up the first three and subtract the fourth from the sum
b20081
2021-03-12 19:53:14
add the first 3 and subtract the 4th
add the first 3 and subtract the 4th
GabeW1234
2021-03-12 19:53:14
yes
yes
BillXu21
2021-03-12 19:53:14
yes
yes
rockyrockrock
2021-03-12 19:53:14
add the first 3 and subtract the 4th and u have 2(x^2+y^2+z^2)
add the first 3 and subtract the 4th and u have 2(x^2+y^2+z^2)
yayy
2021-03-12 19:53:14
add first 3 equations and subtract 4th
add first 3 equations and subtract 4th
pinkpig
2021-03-12 19:53:14
the terms cancel out! ((first equation)+(second equation)+(third equation)-(fourth equation))/2 = AP^2
the terms cancel out! ((first equation)+(second equation)+(third equation)-(fourth equation))/2 = AP^2
gorefeebuddie
2021-03-12 19:53:14
Yes, add everything up divide 2 and subtract the last equation
Yes, add everything up divide 2 and subtract the last equation
happyhari
2021-03-12 19:53:14
add all of the equations together?
add all of the equations together?
BestAOPS
2021-03-12 19:53:14
add together the first three then subtract the last equation
add together the first three then subtract the last equation
Z2589013
2021-03-12 19:53:14
we can solve for AP^2
we can solve for AP^2
sugar_rush
2021-03-12 19:53:14
Yes! Just add the first three equations and subtract off the fourth.
Yes! Just add the first three equations and subtract off the fourth.
MaryM
2021-03-12 19:53:18
Yes! Add the first three equations and subtract the fourth.
Yes! Add the first three equations and subtract the fourth.
MaryM
2021-03-12 19:53:24
Then all the (x-s)^2, (y-s)^2, and (z-s)^2 terms cancel!
Then all the (x-s)^2, (y-s)^2, and (z-s)^2 terms cancel!
MaryM
2021-03-12 19:53:29
And what's left on the left side?
And what's left on the left side?
Summer2019
2021-03-12 19:54:02
2(x^2+y^2+z^2)
2(x^2+y^2+z^2)
GabeW1234
2021-03-12 19:54:02
2 times the square of AP
2 times the square of AP
Z2589013
2021-03-12 19:54:02
2(x^2+y^2+z^2)
2(x^2+y^2+z^2)
sugar_rush
2021-03-12 19:54:02
2(x^{2}+y^{2}+z^{2}), which is 2PA^{2}
2(x^{2}+y^{2}+z^{2}), which is 2PA^{2}
pinkpig
2021-03-12 19:54:02
2(x^2+y^2+z^2)
2(x^2+y^2+z^2)
Z_Lu
2021-03-12 19:54:02
2x^2+2y^2+2z^2
2x^2+2y^2+2z^2
GabeW1234
2021-03-12 19:54:02
2x^2+2y^2+2Z^2
2x^2+2y^2+2Z^2
MaryM
2021-03-12 19:54:05
We're left with 2(x^2+y^2+z^2).
We're left with 2(x^2+y^2+z^2).
MaryM
2021-03-12 19:54:09
But that's 2(AP)^2!
But that's 2(AP)^2!
MaryM
2021-03-12 19:54:14
So we get 2(AP)^2 = 60^2 \cdot 10 + 60^2 \cdot 5 + 120^2 \cdot 2 - 36^2 \cdot 7.
So we get 2(AP)^2 = 60^2 \cdot 10 + 60^2 \cdot 5 + 120^2 \cdot 2 - 36^2 \cdot 7.
MaryM
2021-03-12 19:54:22
To finish, we have to compute AP.
To finish, we have to compute AP.
pinkpig
2021-03-12 19:54:42
we can just simplify this
we can just simplify this
functionalmath
2021-03-12 19:54:42
divide and then factor out a 6^2
divide and then factor out a 6^2
MaryM
2021-03-12 19:54:45
I'd probably factor out a common factor of 12^2 first: 2(AP)^2 = 12^2(5^2 \cdot 10 + 5^2 \cdot 5 + 10^2 \cdot 2 - 3^2 \cdot 7).
I'd probably factor out a common factor of 12^2 first: 2(AP)^2 = 12^2(5^2 \cdot 10 + 5^2 \cdot 5 + 10^2 \cdot 2 - 3^2 \cdot 7).
MaryM
2021-03-12 19:54:52
Then dividing by 2 and computing the squares gives (AP)^2 = 72(25 \cdot 10 + 25 \cdot 5 + 100 \cdot 2 - 9 \cdot 7).
Then dividing by 2 and computing the squares gives (AP)^2 = 72(25 \cdot 10 + 25 \cdot 5 + 100 \cdot 2 - 9 \cdot 7).
MaryM
2021-03-12 19:54:59
This is (AP)^2 = 72(250 + 125 + 200 - 63).
This is (AP)^2 = 72(250 + 125 + 200 - 63).
MaryM
2021-03-12 19:55:04
And this is (AP)^2 = 72(512).
And this is (AP)^2 = 72(512).
MaryM
2021-03-12 19:55:11
Aha! This is (AP)^2 = 36(1024) = 6^2 \cdot 2^{10}.
Aha! This is (AP)^2 = 36(1024) = 6^2 \cdot 2^{10}.
pinkpig
2021-03-12 19:55:22
the answer is 192
the answer is 192
sugar_rush
2021-03-12 19:55:22
PA=\boxed{192}
PA=\boxed{192}
pimath
2021-03-12 19:55:22
AP = 192
AP = 192
MaryM
2021-03-12 19:55:24
So its square root is AP = 6 \cdot 2^5 = 6 \cdot 32 = \boxed{192}.
So its square root is AP = 6 \cdot 2^5 = 6 \cdot 32 = \boxed{192}.
MaryM
2021-03-12 19:55:40
7. Find the number of pairs (m, n) of positive integers with 1 \leq m < n \leq 30 such that there exists a real number x satisfying \sin(mx) + \sin(nx) = 2.
7. Find the number of pairs (m, n) of positive integers with 1 \leq m < n \leq 30 such that there exists a real number x satisfying \sin(mx) + \sin(nx) = 2.
MaryM
2021-03-12 19:55:50
Hmmm...that's a weird equation.
Hmmm...that's a weird equation.
MaryM
2021-03-12 19:55:54
What's really going on here?
What's really going on here?
MaryM
2021-03-12 19:56:14
How can two sines sum to 2?
How can two sines sum to 2?
rockyrockrock
2021-03-12 19:56:30
both have to be 1
both have to be 1
ancientwarrior
2021-03-12 19:56:30
they both must be 1
they both must be 1
sugar_rush
2021-03-12 19:56:30
sin(mx)=sin(nx)=1
sin(mx)=sin(nx)=1
Irving1004
2021-03-12 19:56:30
Well sin has range -1 to 1
Well sin has range -1 to 1
pinkpig
2021-03-12 19:56:30
sin(x)<=1 so sin(mx)=sin(nx)=1
sin(x)<=1 so sin(mx)=sin(nx)=1
kante314
2021-03-12 19:56:30
algebra
algebra
GabeW1234
2021-03-12 19:56:30
1+1=2
1+1=2
Summer2019
2021-03-12 19:56:30
both are equal to 1
both are equal to 1
CircleInvert
2021-03-12 19:56:30
\sin(mx)=\sin(nx)=1
\sin(mx)=\sin(nx)=1
mathisfun17
2021-03-12 19:56:30
each are 1
each are 1
foodisgood
2021-03-12 19:56:30
both 1
both 1
Irving1004
2021-03-12 19:56:30
and sin nx = 1
and sin nx = 1
jxwis2010
2021-03-12 19:56:30
sin(mx)=1, sin(nx)=1
sin(mx)=1, sin(nx)=1
MaryM
2021-03-12 19:56:34
Sines are always between -1 and 1 inclusive.
Sines are always between -1 and 1 inclusive.
MaryM
2021-03-12 19:56:40
So the only way to satisfy that equation is to have \sin(mx) = \sin(nx) = 1.
So the only way to satisfy that equation is to have \sin(mx) = \sin(nx) = 1.
MaryM
2021-03-12 19:56:43
But we can say more than that, right?
But we can say more than that, right?
MaryM
2021-03-12 19:56:50
When is \sin\theta= 1?
When is \sin\theta= 1?
uneducatedpotato
2021-03-12 19:57:55
mx and nx = 90(mod 360)
mx and nx = 90(mod 360)
sugar_rush
2021-03-12 19:57:55
when \theta=90\pmod{360}
when \theta=90\pmod{360}
Summer2019
2021-03-12 19:57:55
theta=pi/2+2kpi
theta=pi/2+2kpi
Z2589013
2021-03-12 19:57:55
\pi/2+2\pi{n} where n is a positive integer
\pi/2+2\pi{n} where n is a positive integer
CircleInvert
2021-03-12 19:57:55
When \theta=k\pi+\frac{pi}{2}
When \theta=k\pi+\frac{pi}{2}
Asher51
2021-03-12 19:57:55
\theta = \frac{\pi}{2} + 2 \pi n
\theta = \frac{\pi}{2} + 2 \pi n
rockyrockrock
2021-03-12 19:57:55
xpi+1/2pi
xpi+1/2pi
mathisfun17
2021-03-12 19:57:55
theta = pi/2 + 2pi*k, k is an integer
theta = pi/2 + 2pi*k, k is an integer
pinkpig
2021-03-12 19:57:55
when the angle is 90 more than 360k for any constant k
when the angle is 90 more than 360k for any constant k
jxwis2010
2021-03-12 19:57:55
$\theta = 2k\pi + \pi/2
$\theta = 2k\pi + \pi/2
MaryM
2021-03-12 19:58:01
Think about the graph of sine:
Think about the graph of sine:
MaryM
2021-03-12 19:58:07

MaryM
2021-03-12 19:58:14
We have \sin\left(\dfrac\pi2\right) = 1.
We have \sin\left(\dfrac\pi2\right) = 1.
MaryM
2021-03-12 19:58:37
Also, we can add any multiple of 2\pi to \dfrac{\pi}{2}, because sine is periodic.
Also, we can add any multiple of 2\pi to \dfrac{\pi}{2}, because sine is periodic.
MaryM
2021-03-12 19:58:45
So if \sin\theta = 1, then we must have \theta = \left(2k + \dfrac12\right)\pi for some integer k.
So if \sin\theta = 1, then we must have \theta = \left(2k + \dfrac12\right)\pi for some integer k.
coolotter
2021-03-12 19:58:49
are we doing this in degrees or radians?
are we doing this in degrees or radians?
happyhari
2021-03-12 19:58:49
is pi/2 radians equivalent to 90 degrees?
is pi/2 radians equivalent to 90 degrees?
MaryM
2021-03-12 19:59:01
We will work with radians, though the logic for degrees is the same.
We will work with radians, though the logic for degrees is the same.
MaryM
2021-03-12 19:59:18
We have that \pi/2 radians is the same as 90^\circ.
We have that \pi/2 radians is the same as 90^\circ.
MaryM
2021-03-12 19:59:38
Okay?
Okay?
happyhari
2021-03-12 19:59:54
ok
ok
GabeW1234
2021-03-12 19:59:54
ok
ok
coolotter
2021-03-12 19:59:54
sounds good
sounds good
Irving1004
2021-03-12 19:59:54
yes
yes
uneducatedpotato
2021-03-12 19:59:54
yep
yep
MaryM
2021-03-12 19:59:57
Back to our equation, that means that we must find some x such that: \begin{align*} mx &= \left(2k + \frac12\right)\pi, \\ nx &= \left(2\ell + \frac12\right)\pi, \end{align*}
for some integers k and \ell.
Back to our equation, that means that we must find some x such that: \begin{align*} mx &= \left(2k + \frac12\right)\pi, \\ nx &= \left(2\ell + \frac12\right)\pi, \end{align*}
for some integers k and \ell.
MaryM
2021-03-12 20:00:03
What can we do with these equations?
What can we do with these equations?
MaryM
2021-03-12 20:00:33
Hint: think about an way to eliminate x.
Hint: think about an way to eliminate x.
Kevinisawesome
2021-03-12 20:00:49
divide
divide
Unevenlogic
2021-03-12 20:00:49
Divide?
Divide?
CircleInvert
2021-03-12 20:00:49
Divide them
Divide them
foodisgood
2021-03-12 20:00:49
divide?
divide?
pinkpig
2021-03-12 20:00:49
divide them
divide them
MaryM
2021-03-12 20:00:51
We can divide them, which gives us
\frac{m}{n} = \frac{2k + 1/2}{2 \ell + 1/2} = \frac{4k + 1}{4 \ell + 1}.
We can divide them, which gives us
\frac{m}{n} = \frac{2k + 1/2}{2 \ell + 1/2} = \frac{4k + 1}{4 \ell + 1}.
MaryM
2021-03-12 20:00:57
So we need \dfrac{m}{n} to be equivalent to a fraction of the form \dfrac{4k + 1}{4 \ell + 1}, for some integers k and \ell.
So we need \dfrac{m}{n} to be equivalent to a fraction of the form \dfrac{4k + 1}{4 \ell + 1}, for some integers k and \ell.
MaryM
2021-03-12 20:01:06
Before we proceed, since we eliminated x, let's do a quick reversibility check. If
\frac{m}{n} = \frac{4k + 1}{4 \ell + 1},
then we can find an x that works?
Before we proceed, since we eliminated x, let's do a quick reversibility check. If
\frac{m}{n} = \frac{4k + 1}{4 \ell + 1},
then we can find an x that works?
peace09
2021-03-12 20:01:39
Yes!
Yes!
yayy
2021-03-12 20:01:39
yes
yes
Unevenlogic
2021-03-12 20:01:39
Yes
Yes
pinkpig
2021-03-12 20:01:39
Yes?
Yes?
Irving1004
2021-03-12 20:01:39
yes always
yes always
MaryM
2021-03-12 20:01:43
If \dfrac{m}{n} = \dfrac{4k + 1}{4 \ell + 1}, then
\frac{2k + 1/2}{m} = \frac{2 \ell + 1/2}{n}.
If \dfrac{m}{n} = \dfrac{4k + 1}{4 \ell + 1}, then
\frac{2k + 1/2}{m} = \frac{2 \ell + 1/2}{n}.
MaryM
2021-03-12 20:01:48
Then we can let
\frac{x}{\pi} = \frac{2k + 1/2}{m} = \frac{2 \ell + 1/2}{n}.
Then we can let
\frac{x}{\pi} = \frac{2k + 1/2}{m} = \frac{2 \ell + 1/2}{n}.
MaryM
2021-03-12 20:01:55
This gives us
\begin{align*} mx &= \left(2k + \frac12\right)\pi, \\ nx &= \left(2\ell + \frac12\right)\pi, \end{align*}
so \sin (mx) + \sin (nx) = 2.
This gives us
\begin{align*} mx &= \left(2k + \frac12\right)\pi, \\ nx &= \left(2\ell + \frac12\right)\pi, \end{align*}
so \sin (mx) + \sin (nx) = 2.
MaryM
2021-03-12 20:02:02
Thus, the condition
\frac{m}{n} = \frac{4k + 1}{4 \ell + 1}
is both necessary and sufficient.
Thus, the condition
\frac{m}{n} = \frac{4k + 1}{4 \ell + 1}
is both necessary and sufficient.
BillXu21
2021-03-12 20:02:04

MaryM
2021-03-12 20:02:15
One thing we can note right away is that 4k + 1 and 4 \ell + 1 are always odd. So we can think about how factors of 2 play a role.
One thing we can note right away is that 4k + 1 and 4 \ell + 1 are always odd. So we can think about how factors of 2 play a role.
MaryM
2021-03-12 20:02:19
If
\frac{m}{n} = \frac{4k + 1}{4 \ell + 1},
then what does that say about the factors of 2 in m and n?
If
\frac{m}{n} = \frac{4k + 1}{4 \ell + 1},
then what does that say about the factors of 2 in m and n?
Lamboreghini
2021-03-12 20:03:09
they're the same
they're the same
Unevenlogic
2021-03-12 20:03:09
They have the same powers of 2
They have the same powers of 2
happyhari
2021-03-12 20:03:09
they cancel out
they cancel out
functionalmath
2021-03-12 20:03:09
must be able to cancel out
must be able to cancel out
foodisgood
2021-03-12 20:03:09
they are the same power
they are the same power
pinkpig
2021-03-12 20:03:09
m and n have the same amount of two's in their prime factorization
m and n have the same amount of two's in their prime factorization
dbasu1
2021-03-12 20:03:09
m and n have the same power of 2 in their prime factorisation
m and n have the same power of 2 in their prime factorisation
MaryM
2021-03-12 20:03:17
The number of factors of 2 in m must be equal to the number of factors of 2 in n. This is the only way that \dfrac{m}{n} is equivalent to a fraction where both the numerator and denominator are odd. (The powers of 2 in m and n must cancel perfectly.)
The number of factors of 2 in m must be equal to the number of factors of 2 in n. This is the only way that \dfrac{m}{n} is equivalent to a fraction where both the numerator and denominator are odd. (The powers of 2 in m and n must cancel perfectly.)
MaryM
2021-03-12 20:03:24
That means m = 2^e a and n = 2^e b, where e is a nonnegative integer, and a and b are odd integers. (In particular, the exponents e must be the same.)
That means m = 2^e a and n = 2^e b, where e is a nonnegative integer, and a and b are odd integers. (In particular, the exponents e must be the same.)
MaryM
2021-03-12 20:03:29
We can then sort all the numbers from 1 to 30 into individual "buckets", depending on the exponent e. Then m and n must be in the same bucket.
We can then sort all the numbers from 1 to 30 into individual "buckets", depending on the exponent e. Then m and n must be in the same bucket.
MaryM
2021-03-12 20:03:40
\begin{array}{r|l} \text{exponent of $2$} & m,n \\ \hline 0 & 1,3,5,\ldots,29 \\ 1 & 2,6,10,\ldots,30 \\ 2 & 4,12,20,28 \\ 3 & 8,24 \\ 4 & 16 \end{array}
\begin{array}{r|l} \text{exponent of $2$} & m,n \\ \hline 0 & 1,3,5,\ldots,29 \\ 1 & 2,6,10,\ldots,30 \\ 2 & 4,12,20,28 \\ 3 & 8,24 \\ 4 & 16 \end{array}
MaryM
2021-03-12 20:04:05
So with m = 2^e a and n = 2^e b, the condition \dfrac{m}{n} = \dfrac{4k + 1}{4 \ell + 1} becomes
\frac{a}{b} = \frac{4k + 1}{4 \ell + 1}.
So with m = 2^e a and n = 2^e b, the condition \dfrac{m}{n} = \dfrac{4k + 1}{4 \ell + 1} becomes
\frac{a}{b} = \frac{4k + 1}{4 \ell + 1}.
MaryM
2021-03-12 20:04:09
Are there any easy cases where the condition \dfrac{a}{b} = \dfrac{4k + 1}{4 \ell + 1} is satisfied?
Are there any easy cases where the condition \dfrac{a}{b} = \dfrac{4k + 1}{4 \ell + 1} is satisfied?
MaryM
2021-03-12 20:04:37
Hint: think in terms of \mod 4.
Hint: think in terms of \mod 4.
coolotter
2021-03-12 20:05:29
m = n = 1 mod 4 then
m = n = 1 mod 4 then
dbasu1
2021-03-12 20:05:29
a and b are both congruent to 1 mod 4
a and b are both congruent to 1 mod 4
Lamboreghini
2021-03-12 20:05:29
when a and b are both congruent to 1 mod 4???
when a and b are both congruent to 1 mod 4???
ancientwarrior
2021-03-12 20:05:29
they are both 1 mod 4
they are both 1 mod 4
yayy
2021-03-12 20:05:29
a = 1 mod 4 and b = 1 mod 4
a = 1 mod 4 and b = 1 mod 4
functionalmath
2021-03-12 20:05:29
both are 1 mod 4
both are 1 mod 4
MaryM
2021-03-12 20:05:32
If a is 1 mod 4 and b is 1 mod 4, then the condition
\dfrac{a}{b} = \dfrac{4k + 1}{4 \ell + 1}
is easily satisfied: Just take 4k + 1 = a and 4 \ell + 1 = b.
If a is 1 mod 4 and b is 1 mod 4, then the condition
\dfrac{a}{b} = \dfrac{4k + 1}{4 \ell + 1}
is easily satisfied: Just take 4k + 1 = a and 4 \ell + 1 = b.
MaryM
2021-03-12 20:05:38
Are there any other cases that work?
Are there any other cases that work?
MaryM
2021-03-12 20:06:30
Any other combination of residues \mod 4 that work?
Any other combination of residues \mod 4 that work?
GabeW1234
2021-03-12 20:07:20
yes
yes
rockyrockrock
2021-03-12 20:07:20
3
3
pinkpig
2021-03-12 20:07:20
both are equal to 3 mod 4
both are equal to 3 mod 4
GabeW1234
2021-03-12 20:07:20
3 times both?
3 times both?
functionalmath
2021-03-12 20:07:20
would 3 work?
would 3 work?
pinkpig
2021-03-12 20:07:20
3?
3?
MaryM
2021-03-12 20:07:25
If a is 3 mod 4 and b is 3 mod 4, then 3a is 1 mod 4 and 3b is 1 mod 4, and
\frac{a}{b} = \frac{3a}{3b}.
If a is 3 mod 4 and b is 3 mod 4, then 3a is 1 mod 4 and 3b is 1 mod 4, and
\frac{a}{b} = \frac{3a}{3b}.
MaryM
2021-03-12 20:07:33
So in this case, we can take 4k + 1 = 3a and 4k + 1 = 3b.
So in this case, we can take 4k + 1 = 3a and 4k + 1 = 3b.
MaryM
2021-03-12 20:07:38
What if a is 1 mod 4 and b is 3 mod 4?
What if a is 1 mod 4 and b is 3 mod 4?
MaryM
2021-03-12 20:08:18
Will that work?
Will that work?
functionalmath
2021-03-12 20:08:45
that wont work
that wont work
kante314
2021-03-12 20:08:45
no
no
GabeW1234
2021-03-12 20:08:45
no?
no?
pinkpig
2021-03-12 20:08:45
that case wouldn't work
that case wouldn't work
dbasu1
2021-03-12 20:08:45
it will not
it will not
rockyrockrock
2021-03-12 20:08:45
no
no
MaryM
2021-03-12 20:08:48
It doesn't look like it will work.
It doesn't look like it will work.
MaryM
2021-03-12 20:08:53
To make sure, we can start with the equation
\frac{a}{b} = \frac{4k + 1}{4 \ell + 1}.
To make sure, we can start with the equation
\frac{a}{b} = \frac{4k + 1}{4 \ell + 1}.
MaryM
2021-03-12 20:08:57
We can write this as (4 \ell + 1) b = (4k + 1) a.
We can write this as (4 \ell + 1) b = (4k + 1) a.
MaryM
2021-03-12 20:09:09
What happens if we take \mod 4 of this equation?
What happens if we take \mod 4 of this equation?
pinkpig
2021-03-12 20:10:21
b=a mod 4
b=a mod 4
Unevenlogic
2021-03-12 20:10:21
a congruent to b
a congruent to b
foodisgood
2021-03-12 20:10:21
it means b mod 4 must equal a mod 4
it means b mod 4 must equal a mod 4
awesomeandy
2021-03-12 20:10:21
b=a
b=a
GabeW1234
2021-03-12 20:10:21
b=a
b=a
yayy
2021-03-12 20:10:21
b = a mod 4
b = a mod 4
happyhari
2021-03-12 20:10:21
then we get b = a
then we get b = a
MaryM
2021-03-12 20:10:25
Taking both sides modulo 4, we get
b \equiv a \pmod{4}.
Taking both sides modulo 4, we get
b \equiv a \pmod{4}.
MaryM
2021-03-12 20:10:30
So if a is 1 mod 4 and b is 3 mod 4, then no such k and \ell exist.
So if a is 1 mod 4 and b is 3 mod 4, then no such k and \ell exist.
MaryM
2021-03-12 20:10:35
We reach the same conclusion if a is 3 mod 4 and b is 1 mod 4.
We reach the same conclusion if a is 3 mod 4 and b is 1 mod 4.
MaryM
2021-03-12 20:10:41
(Remember that a and b are odd, so this covers all the cases.)
(Remember that a and b are odd, so this covers all the cases.)
MaryM
2021-03-12 20:10:57
This tells us that we count the pair (m,n), where m < n, if and only if m = 2^e a and n = 2^e b, where a and b are odd, and a \equiv b \pmod{4}.
This tells us that we count the pair (m,n), where m < n, if and only if m = 2^e a and n = 2^e b, where a and b are odd, and a \equiv b \pmod{4}.
MaryM
2021-03-12 20:11:07
So to count the number of pairs (m,n), we must take each of our buckets and separate further by a, b \pmod{4}.
So to count the number of pairs (m,n), we must take each of our buckets and separate further by a, b \pmod{4}.
MaryM
2021-03-12 20:11:17
\begin{array}{r|c|l} \text{exponent of $2$} & a,b \pmod{4} & m,n \\ \hline 0 & 1 & 1,5,9,13,17,21,25,29 \\ 0 & 3 & 3,7,11,15,19,23,27 \\ 1 & 1 & 2,10,18,26 \\ 1 & 3 & 6,14,22,30 \\ 2 & 1 & 4,20 \\ 2 & 3 & 12,28 \\ 3 & 1 & 8 \\ 3 & 3 & 24 \\ 4 & 1 & 16 \end{array}
\begin{array}{r|c|l} \text{exponent of $2$} & a,b \pmod{4} & m,n \\ \hline 0 & 1 & 1,5,9,13,17,21,25,29 \\ 0 & 3 & 3,7,11,15,19,23,27 \\ 1 & 1 & 2,10,18,26 \\ 1 & 3 & 6,14,22,30 \\ 2 & 1 & 4,20 \\ 2 & 3 & 12,28 \\ 3 & 1 & 8 \\ 3 & 3 & 24 \\ 4 & 1 & 16 \end{array}
byung806
2021-03-12 20:11:24
wow
wow
MaryM
2021-03-12 20:11:38
For each bucket, how many pairs do we have?
For each bucket, how many pairs do we have?
MaryM
2021-03-12 20:13:15
(Your answer will depend on the qnantity of items in each bucket)
(Your answer will depend on the qnantity of items in each bucket)
MaryM
2021-03-12 20:13:34
I mean, quantity.
I mean, quantity.
ancientwarrior
2021-03-12 20:14:00
choose 2
choose 2
Unevenlogic
2021-03-12 20:14:00
(number of items in bucket) choose 2
(number of items in bucket) choose 2
GabeW1234
2021-03-12 20:14:00
(the number of numbers)(the number or numbers-1)
(the number of numbers)(the number or numbers-1)
Summer2019
2021-03-12 20:14:00
(size choose 2)
(size choose 2)
CircleInvert
2021-03-12 20:14:00
28, 21, 6, 6, 1, 1, 0, 0, and 0 respectively
28, 21, 6, 6, 1, 1, 0, 0, and 0 respectively
MaryM
2021-03-12 20:14:03
We can pick any two numbers from our bucket.
We can pick any two numbers from our bucket.
MaryM
2021-03-12 20:14:08
So if there are k numbers, we have \dbinom{k}{2} pairs.
So if there are k numbers, we have \dbinom{k}{2} pairs.
MaryM
2021-03-12 20:14:20
The buckets have sizes 8, 7, 4, 4, 2, and 2 (the buckets with only a single element can be thrown out).
The buckets have sizes 8, 7, 4, 4, 2, and 2 (the buckets with only a single element can be thrown out).
MaryM
2021-03-12 20:14:26
So the count of pairs is \dbinom82 + \dbinom72 + \dbinom42 + \dbinom42 + \dbinom22 + \dbinom22.
So the count of pairs is \dbinom82 + \dbinom72 + \dbinom42 + \dbinom42 + \dbinom22 + \dbinom22.
MaryM
2021-03-12 20:14:30
What does this work out to?
What does this work out to?
functionalmath
2021-03-12 20:15:07
63
63
pinkpig
2021-03-12 20:15:07
28+21+6+6+1+1=063
28+21+6+6+1+1=063
happyhari
2021-03-12 20:15:07
63
63
mop
2021-03-12 20:15:07
63!
63!

MaryM
2021-03-12 20:15:10
We get 28 + 21 + 6 + 6 + 1 + 1.
We get 28 + 21 + 6 + 6 + 1 + 1.
MaryM
2021-03-12 20:15:12
And the sum is \boxed{063}.
And the sum is \boxed{063}.
BillXu21
2021-03-12 20:15:18
yay
yay
Lamboreghini
2021-03-12 20:15:22
yay!
yay!
MaryM
2021-03-12 20:15:32
8. Find the number of integers c such that the equation \Big|\big|20|x| - x^2\big|-c\Big|=21 has 12 distinct real solutions.
8. Find the number of integers c such that the equation \Big|\big|20|x| - x^2\big|-c\Big|=21 has 12 distinct real solutions.
MaryM
2021-03-12 20:15:41
Yikes.
Yikes.
MaryM
2021-03-12 20:15:49
Often problems that look scary are not that bad once you get past all the notation. Hopefully that's the case here.
Often problems that look scary are not that bad once you get past all the notation. Hopefully that's the case here.
skyleristhecoolest
2021-03-12 20:16:05
so. many. abosouloute. values.
so. many. abosouloute. values.
MaryM
2021-03-12 20:16:11
How can we start investigating this?
How can we start investigating this?
gorefeebuddie
2021-03-12 20:17:07
handle the left hand side?
handle the left hand side?
Z_Lu
2021-03-12 20:17:07
break the absolute value in the middle?
break the absolute value in the middle?
Asher51
2021-03-12 20:17:13
from the inside out
from the inside out
MaryM
2021-03-12 20:17:15
Let's look at the innermost part first: 20|x| - x^2.
Let's look at the innermost part first: 20|x| - x^2.
MaryM
2021-03-12 20:17:43
What do we know about this function?
What do we know about this function?
happyhari
2021-03-12 20:18:15
x^2 = |x|^2
x^2 = |x|^2
mathisfun17
2021-03-12 20:18:15
x^2 = |x|^2
x^2 = |x|^2
functionalmath
2021-03-12 20:18:15
x^2 is always nonngeative
x^2 is always nonngeative
CircleInvert
2021-03-12 20:18:15
It is even
It is even
Kevinisawesome
2021-03-12 20:18:15
two parabolas jammed together based on whether x is positive or negative
two parabolas jammed together based on whether x is positive or negative
helloworld_
2021-03-12 20:18:15
It is even
It is even
yayy
2021-03-12 20:18:15
its even
its even
MaryM
2021-03-12 20:18:21
If we replace x with -x, we get the same result.
If we replace x with -x, we get the same result.
MaryM
2021-03-12 20:18:29
Maybe trying to sketch a graph will be helpful?
Maybe trying to sketch a graph will be helpful?
MaryM
2021-03-12 20:18:38
We know that the graph will be symmetric across the y-axis.
We know that the graph will be symmetric across the y-axis.
MaryM
2021-03-12 20:18:48
That is, it's the graph of 20x - x^2 on the right side of the y-axis, and then the mirror-image of that graph across the y-axis to give us the left side.
That is, it's the graph of 20x - x^2 on the right side of the y-axis, and then the mirror-image of that graph across the y-axis to give us the left side.
MaryM
2021-03-12 20:18:52
And what does the graph of 20x - x^2 look like?
And what does the graph of 20x - x^2 look like?
sugar_rush
2021-03-12 20:19:32
inverted parabola
inverted parabola
klpiguy
2021-03-12 20:19:32
quadratic
quadratic
happyhari
2021-03-12 20:19:32
a parabola facing downwards
a parabola facing downwards
MaryM
2021-03-12 20:19:35
It's a downward-opening parabola.
It's a downward-opening parabola.
MaryM
2021-03-12 20:19:48
Where is its vertex?
Where is its vertex?
peace09
2021-03-12 20:20:14
An upside down parabola with center (10, 100)
An upside down parabola with center (10, 100)
peace09
2021-03-12 20:20:14
(10, 100)
(10, 100)
functionalmath
2021-03-12 20:20:14
x=10
x=10
learning0119
2021-03-12 20:20:14
on the y axis
on the y axis
Kevinisawesome
2021-03-12 20:20:14
10,100
10,100
happyhari
2021-03-12 20:20:21
(10,100)
(10,100)
ancientwarrior
2021-03-12 20:20:21
10, 100
10, 100
MaryM
2021-03-12 20:20:26
It's at x = -\dfrac{20}{-2} = 10.
It's at x = -\dfrac{20}{-2} = 10.
MaryM
2021-03-12 20:20:29
And at this point, we have y = 20(10) - 10^2 = 200 - 100 = 100.
And at this point, we have y = 20(10) - 10^2 = 200 - 100 = 100.
MaryM
2021-03-12 20:20:33
So the vertex is at (10,100).
So the vertex is at (10,100).
MaryM
2021-03-12 20:20:42
What else will be helpful to graph this?
What else will be helpful to graph this?
foodisgood
2021-03-12 20:21:07
the roots
the roots
Z_Lu
2021-03-12 20:21:07
intercepts
intercepts
MaryM
2021-03-12 20:21:10
Knowing the roots -- that is, knowing where it crosses the x-axis.
Knowing the roots -- that is, knowing where it crosses the x-axis.
MaryM
2021-03-12 20:21:14
What are the roots?
What are the roots?
functionalmath
2021-03-12 20:21:30
x=0,20
x=0,20
klpiguy
2021-03-12 20:21:30
0,20
0,20
justin6688
2021-03-12 20:21:30
20, 0
20, 0
pinkpig
2021-03-12 20:21:30
20 and 0
20 and 0
happyhari
2021-03-12 20:21:30
0 and 20
0 and 20
klpiguy
2021-03-12 20:21:30
0,20
0,20
MaryM
2021-03-12 20:21:33
Since 20x - x^2 = x(20-x), the roots are at x=0 and x=20.
Since 20x - x^2 = x(20-x), the roots are at x=0 and x=20.
MaryM
2021-03-12 20:21:41
So this is a downward-opening parabola, with vertex at (10,100), and roots at (0,0) and (20,0).
So this is a downward-opening parabola, with vertex at (10,100), and roots at (0,0) and (20,0).
MaryM
2021-03-12 20:21:47
This is enough for us to sketch a graph of this. Remember that we're going to graph this for x \ge 0, and then take its mirror image to give us the graph for x \le 0:
This is enough for us to sketch a graph of this. Remember that we're going to graph this for x \ge 0, and then take its mirror image to give us the graph for x \le 0:
MaryM
2021-03-12 20:21:53

MaryM
2021-03-12 20:22:00
Note: this is definitely not to scale! (If we drew it to scale, it'd be huge, because (10,100) is *way* above the x-axis!)
Note: this is definitely not to scale! (If we drew it to scale, it'd be huge, because (10,100) is *way* above the x-axis!)
MaryM
2021-03-12 20:22:10
Now what?
Now what?
yayy
2021-03-12 20:22:40
take the absolute value
take the absolute value
RaymondZhu
2021-03-12 20:22:40
deal with the absolute value around all that
deal with the absolute value around all that
peace09
2021-03-12 20:22:40
We take the absolute value of this.
We take the absolute value of this.
MaryM
2021-03-12 20:22:42
Let's add a layer of absolute values.
Let's add a layer of absolute values.
MaryM
2021-03-12 20:22:54
What does that mean in terms of our graph?
What does that mean in terms of our graph?
Kevinisawesome
2021-03-12 20:23:22
reflect stuff below x axis above x axis
reflect stuff below x axis above x axis
klpiguy
2021-03-12 20:23:22
everything after |x|>20 gets inverted
everything after |x|>20 gets inverted
Lamboreghini
2021-03-12 20:23:22
all the points below the x-axis are reflected across the x axis
all the points below the x-axis are reflected across the x axis
happyhari
2021-03-12 20:23:29
neglect the graph below the x-axis
neglect the graph below the x-axis
Benranger
2021-03-12 20:23:32
all the points below the x-axis are reflected across the x axis
all the points below the x-axis are reflected across the x axis
MaryM
2021-03-12 20:23:35
Anything that's below the x-axis (that is, is negative) gets flipped above the x-axis (that is, becomes positive).
Anything that's below the x-axis (that is, is negative) gets flipped above the x-axis (that is, becomes positive).
MaryM
2021-03-12 20:23:38
So it looks like this:
So it looks like this:
MaryM
2021-03-12 20:23:42

MaryM
2021-03-12 20:23:52
Let's give this a name so I don't have to keep typing it!
Let's give this a name so I don't have to keep typing it!
MaryM
2021-03-12 20:23:57
Let's set f(x) = \left|20|x| - x^2\right|.
Let's set f(x) = \left|20|x| - x^2\right|.
MaryM
2021-03-12 20:24:02
So now we need to find the integers c such that |f(x) - c| = 21 has exactly 12 solutions.
So now we need to find the integers c such that |f(x) - c| = 21 has exactly 12 solutions.
MaryM
2021-03-12 20:24:06
What does this mean in terms of f(x)?
What does this mean in terms of f(x)?
MaryM
2021-03-12 20:24:44
How can we simplify |f(x)-c|=21?
How can we simplify |f(x)-c|=21?
Kevinisawesome
2021-03-12 20:25:03
its 21+c or -21+c
its 21+c or -21+c
peace09
2021-03-12 20:25:03
f(x)=\pm{21}+c.
f(x)=\pm{21}+c.
Frestho
2021-03-12 20:25:03
f(x) = \pm 21 + c
f(x) = \pm 21 + c
sugar_rush
2021-03-12 20:25:03
f(x)=21+c or f(x)=-21+c
f(x)=21+c or f(x)=-21+c
rockyrockrock
2021-03-12 20:25:07
f(x)-c=21 or -21
f(x)-c=21 or -21
MaryM
2021-03-12 20:25:09
We must have either f(x) = c+21 or f(x) = c-21.
We must have either f(x) = c+21 or f(x) = c-21.
MaryM
2021-03-12 20:25:15
But how many solutions can f(x) = a have for any given a?
But how many solutions can f(x) = a have for any given a?
Kevinisawesome
2021-03-12 20:26:17
6
6
klpiguy
2021-03-12 20:26:17
2, or
2, or
klpiguy
2021-03-12 20:26:17
6
6
Frestho
2021-03-12 20:26:17
6
6
sugar_rush
2021-03-12 20:26:17
6?
6?
ASweatyAsianBoie
2021-03-12 20:26:17
6
6
adsupermath
2021-03-12 20:26:17
6
6
rockyrockrock
2021-03-12 20:26:17
6
6
MaryM
2021-03-12 20:26:23
We draw the line y=a and count intersection points with our graph.
We draw the line y=a and count intersection points with our graph.
MaryM
2021-03-12 20:26:28

MaryM
2021-03-12 20:26:51
The number of solutions will be either 6 or 2 depending on how big a is.
The number of solutions will be either 6 or 2 depending on how big a is.
MaryM
2021-03-12 20:27:12
Since we want 12 solutions in total, we need 6 solutions for each of the two cases.
Since we want 12 solutions in total, we need 6 solutions for each of the two cases.
MaryM
2021-03-12 20:27:43
(We can also have 3 solutions for a=0, or 0 solutions for negative a.)
(We can also have 3 solutions for a=0, or 0 solutions for negative a.)
MaryM
2021-03-12 20:28:07
For which a do we have 6 solutions?
For which a do we have 6 solutions?
pinkpig
2021-03-12 20:28:34
0<(c+21 and c-21)<100
0<(c+21 and c-21)<100
Frestho
2021-03-12 20:28:34
0<a<100
0<a<100
peace09
2021-03-12 20:28:34
0<a<100.
0<a<100.
pinkpig
2021-03-12 20:28:34
from 0 to 100 exclusive
from 0 to 100 exclusive
happyhari
2021-03-12 20:28:34
0<a<100
0<a<100
Z_Lu
2021-03-12 20:28:34
0<a<100
0<a<100
yayy
2021-03-12 20:28:34
0 < a < 100
0 < a < 100
foodisgood
2021-03-12 20:28:34
(0,100)
(0,100)
happyhari
2021-03-12 20:28:34
0 < a < 100
0 < a < 100
MaryM
2021-03-12 20:28:38
If 0 < a < 100, then we'll get exactly 6 intersection points, so we'll get 6 solutions.
If 0 < a < 100, then we'll get exactly 6 intersection points, so we'll get 6 solutions.
MaryM
2021-03-12 20:28:46
That means we need 0 < c+21 < 100 and 0 < c-21 < 100.
That means we need 0 < c+21 < 100 and 0 < c-21 < 100.
MaryM
2021-03-12 20:28:51
How do we solve these inequalities?
How do we solve these inequalities?
sugar_rush
2021-03-12 20:29:39
21<c<79
21<c<79
adsupermath
2021-03-12 20:29:39
21<c<79
21<c<79
Lamboreghini
2021-03-12 20:29:39
subtract 21, add 21
subtract 21, add 21
MaryM
2021-03-12 20:29:43
They combine as 0 < c-21 < c+21 < 100.
They combine as 0 < c-21 < c+21 < 100.
MaryM
2021-03-12 20:29:47
So we must have 0 < c-21 and c+21 < 100.
So we must have 0 < c-21 and c+21 < 100.
MaryM
2021-03-12 20:29:53
This gives us 21 < c and c < 79.
This gives us 21 < c and c < 79.
MaryM
2021-03-12 20:29:57
So c must be an integer strictly between 21 and 79.
So c must be an integer strictly between 21 and 79.
MaryM
2021-03-12 20:30:01
And how many of these are there?
And how many of these are there?
pinkpig
2021-03-12 20:30:25
057 integer solutions for c.
057 integer solutions for c.
learning0119
2021-03-12 20:30:25
57
57
foodisgood
2021-03-12 20:30:25
57
57
Poki
2021-03-12 20:30:25
79-21-1 = 57
79-21-1 = 57
happyhari
2021-03-12 20:30:25
57
57
sugar_rush
2021-03-12 20:30:25
\fbox{57}
\fbox{57}
adsupermath
2021-03-12 20:30:25
57
57
MaryM
2021-03-12 20:30:27
It's the list 22,23,\ldots,78.
It's the list 22,23,\ldots,78.
MaryM
2021-03-12 20:30:31
There are 78-22+1 = 57 integers in this list.
There are 78-22+1 = 57 integers in this list.
MaryM
2021-03-12 20:30:37
Thus there are \boxed{057} valid values for c.
Thus there are \boxed{057} valid values for c.
MaryM
2021-03-12 20:30:52
Be careful with the counting at the end of a problem like this! It's always unfortunate to do the hard work to get to 21 < c < 79 and then miscount the number of c's.
Be careful with the counting at the end of a problem like this! It's always unfortunate to do the hard work to get to 21 < c < 79 and then miscount the number of c's.
pinkpig
2021-03-12 20:31:16
another problem down, just 7 more to go.
another problem down, just 7 more to go.
Poki
2021-03-12 20:31:29
7 more
7 more

MaryM
2021-03-12 20:31:32
9. Let ABCD be an isosceles trapezoid with AD=BC and AB < CD. Suppose that the distances from A to the lines BC, CD, and BD are 15, 18, and 10, respectively. Let K be the area of ABCD. Find \sqrt2\cdot K.
9. Let ABCD be an isosceles trapezoid with AD=BC and AB < CD. Suppose that the distances from A to the lines BC, CD, and BD are 15, 18, and 10, respectively. Let K be the area of ABCD. Find \sqrt2\cdot K.
MaryM
2021-03-12 20:31:38
Let's try to draw a picture of what we have here. I'll also label the feet of the perpendiculars, so we can talk about them.
Let's try to draw a picture of what we have here. I'll also label the feet of the perpendiculars, so we can talk about them.
MaryM
2021-03-12 20:31:48

MaryM
2021-03-12 20:31:54
I think this captures all the given info, right? Note that we have AB < CD and BC = AD.
I think this captures all the given info, right? Note that we have AB < CD and BC = AD.
MaryM
2021-03-12 20:32:02
OK, what can we do now?
OK, what can we do now?
MaryM
2021-03-12 20:32:56
Hint: We have some altitudes in this diagram.
Hint: We have some altitudes in this diagram.
klpiguy
2021-03-12 20:33:56
areas
areas
peace09
2021-03-12 20:33:56
Area?
Area?
mathgeek23
2021-03-12 20:33:56
areas?
areas?
ryanli1366
2021-03-12 20:33:56
find area by bh/2?
find area by bh/2?
MaryM
2021-03-12 20:33:59
Altitudes suggest that using areas might be useful.
Altitudes suggest that using areas might be useful.
MaryM
2021-03-12 20:34:07
For example, we notice that AR=10 is the height of \triangle ABD. Does that help?
For example, we notice that AR=10 is the height of \triangle ABD. Does that help?
klpiguy
2021-03-12 20:35:02
yes
yes
pinkpig
2021-03-12 20:35:02
yes
yes
MaryM
2021-03-12 20:35:04
Do you see another height of this same triangle?
Do you see another height of this same triangle?
Mathaddict3825
2021-03-12 20:35:28
AQ is also an altitude?
AQ is also an altitude?
peace09
2021-03-12 20:35:28
AQ.
AQ.
foodisgood
2021-03-12 20:35:28
AQ=18
AQ=18
ASweatyAsianBoie
2021-03-12 20:35:28
AQ
AQ
adsupermath
2021-03-12 20:35:28
AQ
AQ
ancientwarrior
2021-03-12 20:35:28
AQ is the altitude of the same triangle with base AB
AQ is the altitude of the same triangle with base AB
MaryM
2021-03-12 20:35:30
AQ also a height (a different height) of that same triangle \triangle ABD!
AQ also a height (a different height) of that same triangle \triangle ABD!
MaryM
2021-03-12 20:35:39
If we slide it over, it's the height from D to side \overline{AB}:
If we slide it over, it's the height from D to side \overline{AB}:
MaryM
2021-03-12 20:35:44

MaryM
2021-03-12 20:35:49
Gee, it sure would be nice if the third length were also a height of that triangle...
Gee, it sure would be nice if the third length were also a height of that triangle...
MaryM
2021-03-12 20:36:20
Is it?
Is it?
MaryM
2021-03-12 20:36:54
Hint: this figure is pretty symmetric.
Hint: this figure is pretty symmetric.
floradaisy136
2021-03-12 20:37:02
good news
good news
justin6688
2021-03-12 20:37:02
yes
yes

ryanli1366
2021-03-12 20:37:02
AP, if you flip APB
AP, if you flip APB
functionalmath
2021-03-12 20:37:02
yes
yes
CircleInvert
2021-03-12 20:37:02
It is
It is
pinkpig
2021-03-12 20:37:02
yes
yes
Z_Lu
2021-03-12 20:37:02
yes? AP, since it is an isoceles trapezoid
yes? AP, since it is an isoceles trapezoid
Lamboreghini
2021-03-12 20:37:06
yes it is!!
yes it is!!
Zhaom
2021-03-12 20:37:07
yes because we reflect APB across the axis of symmetry of the trapezoid
yes because we reflect APB across the axis of symmetry of the trapezoid
MaryM
2021-03-12 20:37:10
It is, by symmetry!
It is, by symmetry!
MaryM
2021-03-12 20:37:15

MaryM
2021-03-12 20:37:23
So these are all different heights for the same triangle \triangle ABD!
So these are all different heights for the same triangle \triangle ABD!
MaryM
2021-03-12 20:37:33
How does that help?
How does that help?
Kevinisawesome
2021-03-12 20:37:58
ratio of the bases are known
ratio of the bases are known
ASweatyAsianBoie
2021-03-12 20:37:58
Ratios of all sides?
Ratios of all sides?
Z_Lu
2021-03-12 20:37:58
we now have ratios of the side lengths
we now have ratios of the side lengths
peace09
2021-03-12 20:37:58
15AD=10BD=18AB...
15AD=10BD=18AB...
Mathaddict3825
2021-03-12 20:37:58
ratio of the lengths?
ratio of the lengths?
Zhaom
2021-03-12 20:37:58
15AD=10BD=18AB
15AD=10BD=18AB
adsupermath
2021-03-12 20:37:58
Set different expressions for area equal to each other
Set different expressions for area equal to each other
sugar_rush
2021-03-12 20:37:58
different expressions for the same area
different expressions for the same area
MaryM
2021-03-12 20:38:00
We can compute the area of this triangle in three ways, using the different heights.
We can compute the area of this triangle in three ways, using the different heights.
MaryM
2021-03-12 20:38:03
That is, [ABD] = \dfrac12(10)(BD) = \dfrac12(15)(AD) = \dfrac12(18)(AB).
That is, [ABD] = \dfrac12(10)(BD) = \dfrac12(15)(AD) = \dfrac12(18)(AB).
MaryM
2021-03-12 20:38:08
Let's drop the \dfrac12's, so we have 10(BD) = 15(AD) = 18(AB).
Let's drop the \dfrac12's, so we have 10(BD) = 15(AD) = 18(AB).
MaryM
2021-03-12 20:38:19
What's the least common multiple of 10, 15, and 18?
What's the least common multiple of 10, 15, and 18?
peace09
2021-03-12 20:38:44
90.
90.
klpiguy
2021-03-12 20:38:44
90
90
happyhari
2021-03-12 20:38:44
90
90
Zhaom
2021-03-12 20:38:44
90
90
ryanli1366
2021-03-12 20:38:44
90
90
math-Passion
2021-03-12 20:38:44
90
90
Mathaddict3825
2021-03-12 20:38:48
90
90
Summer2019
2021-03-12 20:38:48
90
90
pinkpig
2021-03-12 20:38:48
90
90
MaryM
2021-03-12 20:38:51
It's 90.
It's 90.
MaryM
2021-03-12 20:38:54
So we can assume that 10(BD) = 15(AD) = 18(AB) = 90k for some constant k.
So we can assume that 10(BD) = 15(AD) = 18(AB) = 90k for some constant k.
MaryM
2021-03-12 20:38:58
That makes BD = 9k, AD = 6k, and AB = 5k.
That makes BD = 9k, AD = 6k, and AB = 5k.
MaryM
2021-03-12 20:39:03
Let's refresh the picture with these lengths:
Let's refresh the picture with these lengths:
MaryM
2021-03-12 20:39:16

MaryM
2021-03-12 20:39:29
Too bad we don't know CD.
Too bad we don't know CD.
MaryM
2021-03-12 20:39:42
Any ideas?
Any ideas?
MaryM
2021-03-12 20:40:17
We could use Pythagoras, but there's a more elegant way.
We could use Pythagoras, but there's a more elegant way.
MaryM
2021-03-12 20:40:38
Hint: The trapezoid is a cyclic quadrilateral.
Hint: The trapezoid is a cyclic quadrilateral.
gorefeebuddie
2021-03-12 20:40:55
ptolemy theorem
ptolemy theorem
uneducatedpotato
2021-03-12 20:40:55
ptolemy's?
ptolemy's?
justin6688
2021-03-12 20:40:55
PTOLEMY'S THEOREM
PTOLEMY'S THEOREM
Mathaddict3825
2021-03-12 20:40:55
ptolemy's?
ptolemy's?
bjc
2021-03-12 20:40:55
Ptolemy
Ptolemy
Zhaom
2021-03-12 20:40:57
Ptolemy's
Ptolemy's
MaryM
2021-03-12 20:41:00
We can use Ptolemy's Theorem!
We can use Ptolemy's Theorem!
MaryM
2021-03-12 20:41:04
Ptolemy's Theorem says that for any cyclic quadrilateral ABCD, we have (AB)(CD) + (AD)(BC) = (AC)(BD). That is, the sum of the products of opposite sides equals the product of the diagonals.
Ptolemy's Theorem says that for any cyclic quadrilateral ABCD, we have (AB)(CD) + (AD)(BC) = (AC)(BD). That is, the sum of the products of opposite sides equals the product of the diagonals.
MaryM
2021-03-12 20:41:13
And an isosceles trapezoid is definitely cyclic!
And an isosceles trapezoid is definitely cyclic!
MaryM
2021-03-12 20:41:16
So what do we get when we apply Ptolemy's Theorem?
So what do we get when we apply Ptolemy's Theorem?
Zhaom
2021-03-12 20:41:46
4kCD+36k^2=81k^2
4kCD+36k^2=81k^2
Mathaddict3825
2021-03-12 20:41:46
DC = 9k
DC = 9k
bjc
2021-03-12 20:41:46
CD=9k
CD=9k
adsupermath
2021-03-12 20:41:46
9k
9k
peace09
2021-03-12 20:41:46
5k(CD)+36k^2=81k^2.
5k(CD)+36k^2=81k^2.
MaryM
2021-03-12 20:41:50
We get (5k)(CD) + (6k)(6k) = (9k)(9k).
We get (5k)(CD) + (6k)(6k) = (9k)(9k).
MaryM
2021-03-12 20:42:02
So (5k)(CD) = 45k^2, and hence CD = 9k.
So (5k)(CD) = 45k^2, and hence CD = 9k.
MaryM
2021-03-12 20:42:09

MaryM
2021-03-12 20:42:27
Unfortunately, we don't know what k is yet.
Unfortunately, we don't know what k is yet.
MaryM
2021-03-12 20:42:42
How can we figure that out?
How can we figure that out?
GabeW1234
2021-03-12 20:43:11
DQ=2k!
DQ=2k!
Mathaddict3825
2021-03-12 20:43:11
pythagoras on ADQ?
pythagoras on ADQ?
nathanqiu
2021-03-12 20:43:11
Pythagorean Theorem
Pythagorean Theorem
happyhari
2021-03-12 20:43:11
use pythagoras
use pythagoras
Kevinisawesome
2021-03-12 20:43:11
pythag
pythag
foodisgood
2021-03-12 20:43:16
draw height from B to DC
draw height from B to DC
MaryM
2021-03-12 20:43:18
How about using \triangle ADQ?
How about using \triangle ADQ?
MaryM
2021-03-12 20:43:22
Indeed, we can break up CD into parts:
Indeed, we can break up CD into parts:
MaryM
2021-03-12 20:43:26

MaryM
2021-03-12 20:43:46
What do we get when we use the Pythagorean Theorem on \triangle ADQ?
What do we get when we use the Pythagorean Theorem on \triangle ADQ?
Zhaom
2021-03-12 20:44:48
4k^2+18^2=36k^2
4k^2+18^2=36k^2
justin6688
2021-03-12 20:44:48
32k^2=324
32k^2=324
Poki
2021-03-12 20:44:48
32k^2 = 324
32k^2 = 324
Lamboreghini
2021-03-12 20:44:48
4k^2+324=36k^2, solving gives k^2=81/8
4k^2+324=36k^2, solving gives k^2=81/8
awesomeandy
2021-03-12 20:44:48
k=sqrt(324/32)
k=sqrt(324/32)
MaryM
2021-03-12 20:44:50
It gives us 18^2 + (2k)^2 = (6k)^2.
It gives us 18^2 + (2k)^2 = (6k)^2.
MaryM
2021-03-12 20:44:54
This simplifies to 18^2 = 32k^2.
This simplifies to 18^2 = 32k^2.
MaryM
2021-03-12 20:45:03
So k = \sqrt{\dfrac{18^2}{32}}.
So k = \sqrt{\dfrac{18^2}{32}}.
MaryM
2021-03-12 20:45:11
Or k = \dfrac{18}{\sqrt{32}} = \dfrac{18}{4\sqrt2} = \dfrac{9}{2\sqrt2}.
Or k = \dfrac{18}{\sqrt{32}} = \dfrac{18}{4\sqrt2} = \dfrac{9}{2\sqrt2}.
MaryM
2021-03-12 20:45:15
And now can we finish?
And now can we finish?
justin6688
2021-03-12 20:45:56
yes!
yes!
foodisgood
2021-03-12 20:45:56
yep
yep
Lamboreghini
2021-03-12 20:45:56
now we can find the area of ABCD and multiply by sqrt2
now we can find the area of ABCD and multiply by sqrt2
happyhari
2021-03-12 20:45:56
find the area of the trapezoid
find the area of the trapezoid
gorefeebuddie
2021-03-12 20:45:56
Yes, the area of a trapezoid is 7k*18 or 136 k
Yes, the area of a trapezoid is 7k*18 or 136 k
peace09
2021-03-12 20:45:56
Yes!
Yes!
MaryM
2021-03-12 20:46:01
Yes! The area of the trapezoid is the height times the average of the bases.
Yes! The area of the trapezoid is the height times the average of the bases.
rockyrockrock
2021-03-12 20:46:27
7k*18
7k*18
MaryM
2021-03-12 20:46:30
Therefore, K = 18(7k) = 18 \cdot 7 \cdot \dfrac{9}{2\sqrt2}.
Therefore, K = 18(7k) = 18 \cdot 7 \cdot \dfrac{9}{2\sqrt2}.
MaryM
2021-03-12 20:46:35
This simplifies to K = \dfrac{9 \cdot 7 \cdot 9}{\sqrt2} = \dfrac{567}{\sqrt2}.
This simplifies to K = \dfrac{9 \cdot 7 \cdot 9}{\sqrt2} = \dfrac{567}{\sqrt2}.
happyhari
2021-03-12 20:46:49
multiply by sqrt(2)
multiply by sqrt(2)
Lamboreghini
2021-03-12 20:46:49
\boxed{567} yay!
\boxed{567} yay!
peace09
2021-03-12 20:46:49
Our answer is \boxed{567}.
Our answer is \boxed{567}.
ryanli1366
2021-03-12 20:46:49
so 567
so 567
MaryM
2021-03-12 20:46:52
So our final answer is \sqrt2 \cdot K = \boxed{567}.
So our final answer is \sqrt2 \cdot K = \boxed{567}.
Irving1004
2021-03-12 20:47:01
Yay!
Yay!
MaryM
2021-03-12 20:47:04
As promised, let’s take a short break. I’ll be back in 5 minutes!
As promised, let’s take a short break. I’ll be back in 5 minutes!
MaryM
2021-03-12 20:52:02
I'm back. Are you still there?
I'm back. Are you still there?
kante314
2021-03-12 20:52:14
Yep
Yep
Zhaom
2021-03-12 20:52:14
yes
yes
nathanqiu
2021-03-12 20:52:14
yes!
yes!
coolotter
2021-03-12 20:52:14
yes we are!
yes we are!
MaryM
2021-03-12 20:52:17
Great, let’s tackle the last 6 problems of the test!
Great, let’s tackle the last 6 problems of the test!
kante314
2021-03-12 20:52:30
Yay
Yay
MaryM
2021-03-12 20:52:33
10. Consider the sequence (a_k)_{k\geq 1} of positive rational numbers defined by a_1 = \dfrac{2020}{2021} and for k \geq 1, if a_k = \dfrac{m}{n} for relatively prime positive integers m and n, then a_{k+1} = \dfrac{m+18}{n+19}. Determine the sum of all positive integers j such that the rational number a_j can be written in the form \dfrac{t}{t+1} for some positive integer t.
10. Consider the sequence (a_k)_{k\geq 1} of positive rational numbers defined by a_1 = \dfrac{2020}{2021} and for k \geq 1, if a_k = \dfrac{m}{n} for relatively prime positive integers m and n, then a_{k+1} = \dfrac{m+18}{n+19}. Determine the sum of all positive integers j such that the rational number a_j can be written in the form \dfrac{t}{t+1} for some positive integer t.
MaryM
2021-03-12 20:52:43
Let's call a positive integer j "good" if is satisfies the condition that a_j = \dfrac{t}{t+1} for some positive integer t.
Let's call a positive integer j "good" if is satisfies the condition that a_j = \dfrac{t}{t+1} for some positive integer t.
MaryM
2021-03-12 20:52:49
We want to find the sum of all the good j's.
We want to find the sum of all the good j's.
MaryM
2021-03-12 20:52:59
Hopefully there are finitely many!
Hopefully there are finitely many!
Zhaom
2021-03-12 20:53:10
compute a few terms and see what happens
compute a few terms and see what happens
MaryM
2021-03-12 20:53:12
Do we know any right away?
Do we know any right away?
sugar_rush
2021-03-12 20:53:59
j=1
j=1
Zhaom
2021-03-12 20:53:59
a_1 works
a_1 works
learning0119
2021-03-12 20:53:59
k = 1
k = 1
functionalmath
2021-03-12 20:53:59
a_1
a_1
klpiguy
2021-03-12 20:53:59
2020/2021
2020/2021
Hector2173
2021-03-12 20:53:59
j = 1
j = 1
Irving1004
2021-03-12 20:53:59
1 satisfies this
1 satisfies this
MaryM
2021-03-12 20:54:05
Yes: j=1 is good, since a_1 = \dfrac{2020}{2021} has the right form.
Yes: j=1 is good, since a_1 = \dfrac{2020}{2021} has the right form.
MaryM
2021-03-12 20:54:12
Any others?
Any others?
adsupermath
2021-03-12 20:54:32
1, 2
1, 2
Zhaom
2021-03-12 20:54:32
a_1 and a_2 work
a_1 and a_2 work
nathanqiu
2021-03-12 20:54:32
yes, a_2
yes, a_2
gorefee
2021-03-12 20:54:32
1,2 works
1,2 works
peace09
2021-03-12 20:54:32
a_2=\tfrac{1019}{1020}.
a_2=\tfrac{1019}{1020}.
gorefee
2021-03-12 20:54:32
2
2
jhao23
2021-03-12 20:54:32
2 also works
2 also works
adsupermath
2021-03-12 20:54:32
j=2
j=2
MaryM
2021-03-12 20:54:34
Following the instructions, we get a_2 = \dfrac{2020+18}{2021+19}.
Following the instructions, we get a_2 = \dfrac{2020+18}{2021+19}.
MaryM
2021-03-12 20:54:39
This is a_2 = \dfrac{2038}{2040}.
This is a_2 = \dfrac{2038}{2040}.
MaryM
2021-03-12 20:54:44
Hey, that simplifies to a_2 = \dfrac{1019}{1020}, and that's lowest terms.
Hey, that simplifies to a_2 = \dfrac{1019}{1020}, and that's lowest terms.
MaryM
2021-03-12 20:54:50
So j=2 is good too.
So j=2 is good too.
MaryM
2021-03-12 20:55:03
Let's continue and compute a_3.
Let's continue and compute a_3.
MaryM
2021-03-12 20:55:07
What do we get?
What do we get?
jhao23
2021-03-12 20:55:39
1037/1039
1037/1039
Zhaom
2021-03-12 20:55:39
\frac{1037}{1039}
\frac{1037}{1039}
learning0119
2021-03-12 20:55:39
1037/1039
1037/1039
sugar_rush
2021-03-12 20:55:39
\tfrac{1037}{1039}
\tfrac{1037}{1039}
adsupermath
2021-03-12 20:55:39
\frac{1037}{1039}
\frac{1037}{1039}
functionalmath
2021-03-12 20:55:39
1037/1039
1037/1039
MaryM
2021-03-12 20:55:42
We get a_3 = \dfrac{1019+18}{1020+19} = \dfrac{1037}{1039}.
We get a_3 = \dfrac{1019+18}{1020+19} = \dfrac{1037}{1039}.
MaryM
2021-03-12 20:55:46
It doesn't look like it simplifies, but how can we be sure?
It doesn't look like it simplifies, but how can we be sure?
functionalmath
2021-03-12 20:56:21
euclids algorithm
euclids algorithm
adsupermath
2021-03-12 20:56:21
Difference of 2, but both are odd
Difference of 2, but both are odd
nathanqiu
2021-03-12 20:56:21
its odd, and has difference of 2
its odd, and has difference of 2
Zhaom
2021-03-12 20:56:21
\gcd(1037,1039)=\gcd(1037,2)=1
\gcd(1037,1039)=\gcd(1037,2)=1
foodisgood
2021-03-12 20:56:21
they don't share two so no other factors
they don't share two so no other factors
Irving1004
2021-03-12 20:56:21
they are consecutive odd numbers
they are consecutive odd numbers
nathanqiu
2021-03-12 20:56:21
numbers are both odd, and both have difference of 2
numbers are both odd, and both have difference of 2
ancientwarrior
2021-03-12 20:56:21
their difference
their difference
ryanli1366
2021-03-12 20:56:23
their difference, 2 is only divisible by 2, and because they are both odd, there is no common divisor
their difference, 2 is only divisible by 2, and because they are both odd, there is no common divisor
MaryM
2021-03-12 20:56:25
We can use the Euclidean Algorithm!
We can use the Euclidean Algorithm!
MaryM
2021-03-12 20:56:28
It tells us that \gcd(1039,1037) = \gcd(1039-1037,1037) = \gcd(2,1037) = 1.
It tells us that \gcd(1039,1037) = \gcd(1039-1037,1037) = \gcd(2,1037) = 1.
MaryM
2021-03-12 20:56:33
So 1037 and 1039 don't have any common factors, and the fraction can't be simplified.
So 1037 and 1039 don't have any common factors, and the fraction can't be simplified.
MaryM
2021-03-12 20:56:41
Hmmm...does this give us any clues on how to generalize this?
Hmmm...does this give us any clues on how to generalize this?
MaryM
2021-03-12 20:57:20
Hint: think about how can we write the next few terms of the sequence in terms of a variable.
Hint: think about how can we write the next few terms of the sequence in terms of a variable.
sugar_rush
2021-03-12 20:58:17
find a_{2+q} in terms of q
find a_{2+q} in terms of q
Zhaom
2021-03-12 20:58:17
\frac{1001+18(j-1)}{1001+19(j-1)} until simplification
\frac{1001+18(j-1)}{1001+19(j-1)} until simplification
peace09
2021-03-12 20:58:17
How about a_{k+2}=\tfrac{1019+18k}{1020+19k}?
How about a_{k+2}=\tfrac{1019+18k}{1020+19k}?
MaryM
2021-03-12 20:58:20
We know that for any k \ge 1, as long as we can't simplify, we'll have a_{2+k} = \dfrac{1019+18k}{1020+19k}.
We know that for any k \ge 1, as long as we can't simplify, we'll have a_{2+k} = \dfrac{1019+18k}{1020+19k}.
MaryM
2021-03-12 20:58:42
When will this first be able to be simplified?
When will this first be able to be simplified?
MaryM
2021-03-12 20:59:23
Hint: think in terms of k.
Hint: think in terms of k.
Zhaom
2021-03-12 20:59:48
when \gcd(1019+18k,1020+19k) \ge 2
when \gcd(1019+18k,1020+19k) \ge 2
peace09
2021-03-12 20:59:48
When \text{gcd}[1019+18k, 1020+19k] > 1.
When \text{gcd}[1019+18k, 1020+19k] > 1.
MaryM
2021-03-12 20:59:51
We'll be able to reduce when \gcd(1019+18k,1020+19k) > 1.
We'll be able to reduce when \gcd(1019+18k,1020+19k) > 1.
MaryM
2021-03-12 20:59:59
But now we can apply the Euclidean Algorithm!
But now we can apply the Euclidean Algorithm!
MaryM
2021-03-12 21:00:09
We get \gcd(1019+18k,1020+19k) = \gcd(1019+18k,(1020+19k)-(1019+18k)).
We get \gcd(1019+18k,1020+19k) = \gcd(1019+18k,(1020+19k)-(1019+18k)).
MaryM
2021-03-12 21:00:16
So the gcd is \gcd(1019+18k,1+k).
So the gcd is \gcd(1019+18k,1+k).
MaryM
2021-03-12 21:00:25
That is, a_{2+k} will first simplify when we have k \ge 1 such that \gcd(1019+18k,1+k) > 1.
That is, a_{2+k} will first simplify when we have k \ge 1 such that \gcd(1019+18k,1+k) > 1.
MaryM
2021-03-12 21:01:01
Now what?
Now what?
peace09
2021-03-12 21:01:40
Apply the Euclidean Algorithm Again to Get \text{gcd}[1001, 1+k] > 1.
Apply the Euclidean Algorithm Again to Get \text{gcd}[1001, 1+k] > 1.
klpiguy
2021-03-12 21:01:40
euclidiean algorithm again
euclidiean algorithm again
foodisgood
2021-03-12 21:01:40
go further with Euclidean algorithm
go further with Euclidean algorithm
MaryM
2021-03-12 21:01:45
We can apply the Euclidean Algorithm again! How?
We can apply the Euclidean Algorithm again! How?
foodisgood
2021-03-12 21:02:15
eliminate k one one side
eliminate k one one side
MaryM
2021-03-12 21:02:18
We can repeatedly subtract 1+k from 1019+18k.
We can repeatedly subtract 1+k from 1019+18k.
MaryM
2021-03-12 21:02:26
How many times do you think we'll want to subtract it?
How many times do you think we'll want to subtract it?
functionalmath
2021-03-12 21:02:47
multiply (1+k)b y 18
multiply (1+k)b y 18
peace09
2021-03-12 21:02:47
Apply It... 18 Times
Apply It... 18 Times

learning0119
2021-03-12 21:02:47
18
18
Zhaom
2021-03-12 21:02:47
18 times
18 times
happyhari
2021-03-12 21:02:47
18
18
foodisgood
2021-03-12 21:02:47
18 times
18 times
YaoAOPS
2021-03-12 21:02:47
18
18
MaryM
2021-03-12 21:02:50
How about 18 times, to get rid of the k?
How about 18 times, to get rid of the k?
MaryM
2021-03-12 21:02:54
That is, we get \gcd(1019+18k,1+k) = \gcd(1019+18k - 18(1+k),1+k) = \gcd(1001,1+k).
That is, we get \gcd(1019+18k,1+k) = \gcd(1019+18k - 18(1+k),1+k) = \gcd(1001,1+k).
MaryM
2021-03-12 21:03:05
And when is this gcd greater than 1?
And when is this gcd greater than 1?
Irving1004
2021-03-12 21:03:30
gcd(1001, k+1) > 1 therefore k+1=7 works and k=6 satisfies
gcd(1001, k+1) > 1 therefore k+1=7 works and k=6 satisfies
sugar_rush
2021-03-12 21:03:30
This happens when k+1 divides 1001 for the first time, so k=6.
This happens when k+1 divides 1001 for the first time, so k=6.
b20081
2021-03-12 21:03:30
when k=6
when k=6
Hector2173
2021-03-12 21:03:30
k = 6
k = 6
foodisgood
2021-03-12 21:03:33
when k+1 is a factor of 1001
when k+1 is a factor of 1001
MaryM
2021-03-12 21:03:35
Note that 1001 = 7 \cdot 11 \cdot 13. (This is a number whose factorization seems to come up a lot in contests.)
Note that 1001 = 7 \cdot 11 \cdot 13. (This is a number whose factorization seems to come up a lot in contests.)
MaryM
2021-03-12 21:03:40
So k=6 is the first time, and we get \gcd(1001,7) = 7, meaning that a 7 will cancel out.
So k=6 is the first time, and we get \gcd(1001,7) = 7, meaning that a 7 will cancel out.
MaryM
2021-03-12 21:04:10
What do we get when k=6?
What do we get when k=6?
MaryM
2021-03-12 21:04:41
I mean, what value will we get for the sequence?
I mean, what value will we get for the sequence?
MaryM
2021-03-12 21:05:48
When k=6, we will be dealing with a_8. And a_8 equals....
When k=6, we will be dealing with a_8. And a_8 equals....
Hector2173
2021-03-12 21:06:01
161/162
161/162
Hector2173
2021-03-12 21:06:05
\frac{161}{162}
\frac{161}{162}
Zhaom
2021-03-12 21:06:05
\frac{161}{162}
\frac{161}{162}
ancientwarrior
2021-03-12 21:06:05
161/162
161/162
MaryM
2021-03-12 21:06:08
We get a_8 = \dfrac{1019 + 18(6)}{1020 + 19(6)} = \dfrac{1127}{1134}.
We get a_8 = \dfrac{1019 + 18(6)}{1020 + 19(6)} = \dfrac{1127}{1134}.
MaryM
2021-03-12 21:06:19
And, as expected, 7 is a common factor! We have a_8 = \dfrac{1127}{1134} = \dfrac{7 \cdot 161}{7 \cdot 162} = \dfrac{161}{162}.
And, as expected, 7 is a common factor! We have a_8 = \dfrac{1127}{1134} = \dfrac{7 \cdot 161}{7 \cdot 162} = \dfrac{161}{162}.
MaryM
2021-03-12 21:06:25
So j=8 is a good number!
So j=8 is a good number!
MaryM
2021-03-12 21:06:34
We've get j \in \{1,2,8\} so far.
We've get j \in \{1,2,8\} so far.
MaryM
2021-03-12 21:06:38
Now what?
Now what?
Hector2173
2021-03-12 21:06:59
do it again
do it again
MaryM
2021-03-12 21:07:13
We can play the same game!
We can play the same game!
MaryM
2021-03-12 21:07:21
That is, we'll have a_{8+k} = \dfrac{161+18k}{162+19k} until we get a common factor.
That is, we'll have a_{8+k} = \dfrac{161+18k}{162+19k} until we get a common factor.
MaryM
2021-03-12 21:07:33
So we're looking for the smallest k such that \gcd(161+18k,162+19k) > 1.
So we're looking for the smallest k such that \gcd(161+18k,162+19k) > 1.
MaryM
2021-03-12 21:07:57
Time for the Euclidean Algorithm again! What do we get when we simplify that \gcd?
Time for the Euclidean Algorithm again! What do we get when we simplify that \gcd?
Zhaom
2021-03-12 21:08:31
Simplifying, we get that \gcd(143,k+1) \ge 2.
Simplifying, we get that \gcd(143,k+1) \ge 2.
MaryM
2021-03-12 21:08:35
We get \gcd(161+18k,162+19k) = \gcd(161+18k,1+k).
We get \gcd(161+18k,162+19k) = \gcd(161+18k,1+k).
MaryM
2021-03-12 21:08:45
Then subtract 1+k 18 times to get \gcd(143,1+k).
Then subtract 1+k 18 times to get \gcd(143,1+k).
MaryM
2021-03-12 21:08:49
What's the smallest value of k that makes this greater than 1?
What's the smallest value of k that makes this greater than 1?
Irving1004
2021-03-12 21:09:10
then we get that k+1 divides into 143 and is 11
then we get that k+1 divides into 143 and is 11
klpiguy
2021-03-12 21:09:10
10
10
foodisgood
2021-03-12 21:09:10
10
10
Zhaom
2021-03-12 21:09:10
k=10
k=10
happyhari
2021-03-12 21:09:10
10
10
Mathaddict3825
2021-03-12 21:09:13
10
10
b20081
2021-03-12 21:09:13
10
10
MaryM
2021-03-12 21:09:15
Since 143 = 13 \cdot 11, we get k+1 = 11, or k=10.
Since 143 = 13 \cdot 11, we get k+1 = 11, or k=10.
MaryM
2021-03-12 21:09:19
Let's try it!
Let's try it!
MaryM
2021-03-12 21:09:25
We get a_{18} = \dfrac{161 + 18(10)}{162 + 19(10)} = \dfrac{341}{352}.
We get a_{18} = \dfrac{161 + 18(10)}{162 + 19(10)} = \dfrac{341}{352}.
klpiguy
2021-03-12 21:09:41
31/32
31/32
MaryM
2021-03-12 21:09:46
Simplifying a_{18} = \dfrac{11 \cdot 31}{11 \cdot 32} = \dfrac{31}{32}.
Simplifying a_{18} = \dfrac{11 \cdot 31}{11 \cdot 32} = \dfrac{31}{32}.
MaryM
2021-03-12 21:09:51
So j= 18 is good!
So j= 18 is good!
MaryM
2021-03-12 21:09:55
And our list so far is j \in \{1,2,8,18\}.
And our list so far is j \in \{1,2,8,18\}.
MaryM
2021-03-12 21:10:00
Now what?
Now what?
Zhaom
2021-03-12 21:10:11
DO IT AGAIN
DO IT AGAIN
Mathaddict3825
2021-03-12 21:10:11
again
again
happyhari
2021-03-12 21:10:11
again
again
foodisgood
2021-03-12 21:10:11
again!
again!
Irving1004
2021-03-12 21:10:11
now we do the same thing agaibn
now we do the same thing agaibn
MaryM
2021-03-12 21:10:13
Lather, rinse, repeat! Let's do it again!
Lather, rinse, repeat! Let's do it again!
MaryM
2021-03-12 21:10:18
We have a_{18+k} = \dfrac{31 + 18k}{32 + 19k} until we get a common factor.
We have a_{18+k} = \dfrac{31 + 18k}{32 + 19k} until we get a common factor.
MaryM
2021-03-12 21:10:25
Euclidean Algorithm time!
Euclidean Algorithm time!
MaryM
2021-03-12 21:10:38
We have \gcd(31+18k,32+19k) = \gcd(31+18k,1+k).
We have \gcd(31+18k,32+19k) = \gcd(31+18k,1+k).
MaryM
2021-03-12 21:10:42
And then?
And then?
Zhaom
2021-03-12 21:11:09
Simplifies to \gcd(13,k+1) \ge 2.
Simplifies to \gcd(13,k+1) \ge 2.
foodisgood
2021-03-12 21:11:09
(13,k+1)
(13,k+1)
learning0119
2021-03-12 21:11:09
gcd(13, 1+k)
gcd(13, 1+k)
happyhari
2021-03-12 21:11:09
gcd(13,1+k)
gcd(13,1+k)
MaryM
2021-03-12 21:11:11
And subtracting 1+k 18 times gives \gcd(13,1+k).
And subtracting 1+k 18 times gives \gcd(13,1+k).
MaryM
2021-03-12 21:11:16
So what is k?
So what is k?
GabeW1234
2021-03-12 21:11:32
k=12
k=12
happyhari
2021-03-12 21:11:32
k = 12
k = 12
Mathaddict3825
2021-03-12 21:11:32
12
12
happyhari
2021-03-12 21:11:32
12
12
Zhaom
2021-03-12 21:11:32
k=12
k=12
foodisgood
2021-03-12 21:11:32
12
12
learning0119
2021-03-12 21:11:32
12
12
MaryM
2021-03-12 21:11:34
13 is prime, so we need k=12 to get a common factor of 13.
13 is prime, so we need k=12 to get a common factor of 13.
MaryM
2021-03-12 21:11:39
Let's check it and see!
Let's check it and see!
MaryM
2021-03-12 21:11:43
We have a_{30} = \dfrac{31 + 18(12)}{32 + 19(12)} = \dfrac{247}{260}.
We have a_{30} = \dfrac{31 + 18(12)}{32 + 19(12)} = \dfrac{247}{260}.
MaryM
2021-03-12 21:11:47
And indeed, we cancel a 13, and we have a_{30} = \dfrac{19}{20}.
And indeed, we cancel a 13, and we have a_{30} = \dfrac{19}{20}.
MaryM
2021-03-12 21:11:50
So j=30 is good!
So j=30 is good!
MaryM
2021-03-12 21:11:58
And our list now is j \in \{1,2,8,18,30\}.
And our list now is j \in \{1,2,8,18,30\}.
MaryM
2021-03-12 21:12:04
Now what?
Now what?
learning0119
2021-03-12 21:12:15
again
again
MaryM
2021-03-12 21:12:20
Let's do it again!
Let's do it again!
MaryM
2021-03-12 21:12:24
We have a_{30+k} = \dfrac{19 + 18k}{20 + 19k} until there's a common factor.
We have a_{30+k} = \dfrac{19 + 18k}{20 + 19k} until there's a common factor.
MaryM
2021-03-12 21:12:31
The common factor is \gcd(19+18k,20+19k) = \gcd(19+18k,1+k), as before.
The common factor is \gcd(19+18k,20+19k) = \gcd(19+18k,1+k), as before.
MaryM
2021-03-12 21:12:37
And when we subtract 1+k 18 times, we get \gcd(1,1+k).
And when we subtract 1+k 18 times, we get \gcd(1,1+k).
MaryM
2021-03-12 21:12:44
Hmmm...what does that tell us?
Hmmm...what does that tell us?
klpiguy
2021-03-12 21:12:55
there are no more good numbers
there are no more good numbers
GabeW1234
2021-03-12 21:12:55
no more
no more
rockyrockrock
2021-03-12 21:12:55
it cannot be greater than 1
it cannot be greater than 1
ancientwarrior
2021-03-12 21:12:55
we're done!
we're done!
Zhaom
2021-03-12 21:12:55
Simplifies to \gcd(1,1+k) \ge 2, which is impossible.
Simplifies to \gcd(1,1+k) \ge 2, which is impossible.
Hector2173
2021-03-12 21:12:55
we're done
we're done
learning0119
2021-03-12 21:12:57
we are done
we are done
MaryM
2021-03-12 21:13:00
That gcd is always 1.
That gcd is always 1.
MaryM
2021-03-12 21:13:04
So 19+18k and 20+19k are always relatively prime.
So 19+18k and 20+19k are always relatively prime.
MaryM
2021-03-12 21:13:08
Which means a_{30+k} will never reduce!
Which means a_{30+k} will never reduce!
MaryM
2021-03-12 21:13:14
That's good, because it means we're done! We've found all the good numbers: j \in \{1,2,8,18,30\}.
That's good, because it means we're done! We've found all the good numbers: j \in \{1,2,8,18,30\}.
happyhari
2021-03-12 21:13:18
finally!
finally!
GabeW1234
2021-03-12 21:13:23
so we stop, and add
so we stop, and add
MaryM
2021-03-12 21:13:26
So what is the final answer?
So what is the final answer?
Zhaom
2021-03-12 21:13:37
1+2+8+18+30=\boxed{059}
1+2+8+18+30=\boxed{059}
GabeW1234
2021-03-12 21:13:37
59
59
happyhari
2021-03-12 21:13:37
59
59
ancientwarrior
2021-03-12 21:13:37
59
59
Irving1004
2021-03-12 21:13:39
059
059
learning0119
2021-03-12 21:13:43
$\boxed{059}
$\boxed{059}
MaryM
2021-03-12 21:13:46
Their sum is 1+2+8+18+30 = \boxed{059}.
Their sum is 1+2+8+18+30 = \boxed{059}.
MaryM
2021-03-12 21:14:15
11. Let ABCD be a cyclic quadrilateral with AB=4, BC=5, CD=6, and DA=7. Let A_1 and C_1 be the feet of the perpendiculars from A and C, respectively, to line BD, and let B_1 and D_1 be the feet of the perpendiculars from B and D, respectively, to line AC. The perimeter of A_1B_1C_1D_1 is \dfrac{m}{n}, where m and n are relatively prime positive integers. Find m+n.
11. Let ABCD be a cyclic quadrilateral with AB=4, BC=5, CD=6, and DA=7. Let A_1 and C_1 be the feet of the perpendiculars from A and C, respectively, to line BD, and let B_1 and D_1 be the feet of the perpendiculars from B and D, respectively, to line AC. The perimeter of A_1B_1C_1D_1 is \dfrac{m}{n}, where m and n are relatively prime positive integers. Find m+n.
MaryM
2021-03-12 21:14:31
Let's see if we can sketch a picture of this. First, here's the quadrilateral:
Let's see if we can sketch a picture of this. First, here's the quadrilateral:
MaryM
2021-03-12 21:14:39

CrystalFlower
2021-03-12 21:14:59
kool
kool
MaryM
2021-03-12 21:15:01
Any observations before we clutter it further with all the perpendiculars?
Any observations before we clutter it further with all the perpendiculars?
functionalmath
2021-03-12 21:15:23
ptolemy?
ptolemy?
functionalmath
2021-03-12 21:15:23
use ptolemys theorem and find the diagonals?
use ptolemys theorem and find the diagonals?
learning0119
2021-03-12 21:15:23
ptolemy
ptolemy
MaryM
2021-03-12 21:15:28
Well, we do have a cyclic quadrilateral, and we know all the side lengths.
Well, we do have a cyclic quadrilateral, and we know all the side lengths.
MaryM
2021-03-12 21:15:31
So we do have Ptolemy's Theorem. (Second time this contest!)
So we do have Ptolemy's Theorem. (Second time this contest!)
MaryM
2021-03-12 21:15:35
What does it tell us?
What does it tell us?
Zhaom
2021-03-12 21:16:09
AC \cdot BD=59
AC \cdot BD=59
GabeW1234
2021-03-12 21:16:09
diagonals x and y have a product of 59
diagonals x and y have a product of 59
ancientwarrior
2021-03-12 21:16:09
product of diagonals is 59
product of diagonals is 59
MeepMurp5
2021-03-12 21:16:09
AC\cdotBC=59
AC\cdotBC=59
Mathaddict3825
2021-03-12 21:16:09
(AC)(BD) = 59
(AC)(BD) = 59
Lamboreghini
2021-03-12 21:16:09
5*7+6*4=BD*AC
5*7+6*4=BD*AC
MaryM
2021-03-12 21:16:11
We get (AB)(CD) + (AD)(BC) = (AC)(BD).
We get (AB)(CD) + (AD)(BC) = (AC)(BD).
MaryM
2021-03-12 21:16:24
So (AC)(BD) = (4)(6) + (5)(7) = 24 + 35 = 59.
So (AC)(BD) = (4)(6) + (5)(7) = 24 + 35 = 59.
MaryM
2021-03-12 21:16:30
Not sure if this will help, so let's set it aside for now.
Not sure if this will help, so let's set it aside for now.
MaryM
2021-03-12 21:16:37
(AC)(BD) = 59
(AC)(BD) = 59
rockyrockrock
2021-03-12 21:16:40
wacky number
wacky number
MaryM
2021-03-12 21:16:44
OK, let's add all the other stuff. I'll make the pic a little bigger so we can see all the detail.
OK, let's add all the other stuff. I'll make the pic a little bigger so we can see all the detail.
MaryM
2021-03-12 21:16:51

MaryM
2021-03-12 21:17:01
I've also labeled the point where the diagonals intersect as X. I'm guessing that point might be important.
I've also labeled the point where the diagonals intersect as X. I'm guessing that point might be important.
Irving1004
2021-03-12 21:17:06
This is where they get hard
This is where they get hard
klpiguy
2021-03-12 21:17:06
it looks messy
it looks messy
MaryM
2021-03-12 21:17:13
Any thoughts?
Any thoughts?
rockyrockrock
2021-03-12 21:18:01
quadrilateral 1 looks similar to original
quadrilateral 1 looks similar to original
Lamboreghini
2021-03-12 21:18:01
maybe A1B1C1D1 is also cyclic? maybe it's similar to ABCD?
maybe A1B1C1D1 is also cyclic? maybe it's similar to ABCD?
happyhari
2021-03-12 21:18:01
are the two quadrilaterals similar?
are the two quadrilaterals similar?
MaryM
2021-03-12 21:18:06
The two quadrilaterals do look similar. But we don’t have many tools to deal with similar quadrilaterals. Why don’t we try finding some similar triangles instead?
The two quadrilaterals do look similar. But we don’t have many tools to deal with similar quadrilaterals. Why don’t we try finding some similar triangles instead?
MaryM
2021-03-12 21:19:29
But to find similar triangles we will need equal angles, so we need to take a closer look into this picture.
But to find similar triangles we will need equal angles, so we need to take a closer look into this picture.
MaryM
2021-03-12 21:19:35
We've got the big quadrilateral ABCD and the little one A_1B_1C_1D_1, but there are others in this picture too, right?
We've got the big quadrilateral ABCD and the little one A_1B_1C_1D_1, but there are others in this picture too, right?
MaryM
2021-03-12 21:19:44
For example, ABA_1B_1. What do we know about it?
For example, ABA_1B_1. What do we know about it?
CircleInvert
2021-03-12 21:20:28
It is cyclic
It is cyclic
ancientwarrior
2021-03-12 21:20:28
it is cyclic
it is cyclic
Irving1004
2021-03-12 21:20:28
It is cyclic
It is cyclic
MaryM
2021-03-12 21:20:30
It's cyclic!
It's cyclic!
MaryM
2021-03-12 21:20:43
The right angles tell us it's cyclic: specifically, its circumcircle has diameter \overline{AB}:
The right angles tell us it's cyclic: specifically, its circumcircle has diameter \overline{AB}:
MaryM
2021-03-12 21:20:49

MaryM
2021-03-12 21:21:00
Note how the two right angles intercept the diameter of the red circle.
Note how the two right angles intercept the diameter of the red circle.
MaryM
2021-03-12 21:21:13
Great -- how can we use this cyclic quadrilateral to make progress on the problem?
Great -- how can we use this cyclic quadrilateral to make progress on the problem?
MaryM
2021-03-12 21:21:43
Hint: Try to find some equal angles.
Hint: Try to find some equal angles.
Zhaom
2021-03-12 21:22:12
\angle C_1A_1B_1=\angle BAB_1
\angle C_1A_1B_1=\angle BAB_1
MaryM
2021-03-12 21:22:17
There are lots of cyclic quadrilateral facts, but a particularly useful one here is that opposite angles of a cyclic quadrilateral sum to 180^\circ.
There are lots of cyclic quadrilateral facts, but a particularly useful one here is that opposite angles of a cyclic quadrilateral sum to 180^\circ.
MaryM
2021-03-12 21:22:27
It means that an angle of a cyclic quadrilateral is congruent to the exterior angle at the opposite vertex.
It means that an angle of a cyclic quadrilateral is congruent to the exterior angle at the opposite vertex.
MaryM
2021-03-12 21:22:36
That is, \angle BAB_1 is congruent to the exterior angle \angle B_1A_1X on the opposite side.
That is, \angle BAB_1 is congruent to the exterior angle \angle B_1A_1X on the opposite side.
MaryM
2021-03-12 21:22:50
And in the same way, \angle ABA_1 is congruent to the opposite exterior angle \angle A_1B_1X.
And in the same way, \angle ABA_1 is congruent to the opposite exterior angle \angle A_1B_1X.
MaryM
2021-03-12 21:22:54
This might be easier to see if I mark these in the picture:
This might be easier to see if I mark these in the picture:
MaryM
2021-03-12 21:23:00

MaryM
2021-03-12 21:23:05
So what do we get?
So what do we get?
Zhaom
2021-03-12 21:23:47
\triangle XBA\sim triangle XA_1B_1
\triangle XBA\sim triangle XA_1B_1
happyhari
2021-03-12 21:23:47
similar triangles
similar triangles
Lamboreghini
2021-03-12 21:23:47
A1XB1~AXB
A1XB1~AXB
Irving1004
2021-03-12 21:23:47
Similar triangles that take too long to type out!
Similar triangles that take too long to type out!
MaryM
2021-03-12 21:23:49
We get that \triangle ABX and \triangle A_1B_1X are similar! They have the same two angles away from X (and the same third angle at X).
We get that \triangle ABX and \triangle A_1B_1X are similar! They have the same two angles away from X (and the same third angle at X).
MaryM
2021-03-12 21:23:54
In terms of lengths, what does this give us?
In terms of lengths, what does this give us?
GabeW1234
2021-03-12 21:24:52
4:A_1B_1=XA_1:XB_1=XB_1:A
4:A_1B_1=XA_1:XB_1=XB_1:A
Zhaom
2021-03-12 21:24:52
\frac{XB}{XB_1}=\frac{XA}{XA_1}=\frac{AB}{A_1B_1}
\frac{XB}{XB_1}=\frac{XA}{XA_1}=\frac{AB}{A_1B_1}
MaryM
2021-03-12 21:24:55
We get \dfrac{AB}{A_1B_1} = \dfrac{AX}{A_1X} = \dfrac{BX}{B_1X}.
We get \dfrac{AB}{A_1B_1} = \dfrac{AX}{A_1X} = \dfrac{BX}{B_1X}.
MaryM
2021-03-12 21:24:59
Aha -- there's the length A_1B_1 we want!
Aha -- there's the length A_1B_1 we want!
MaryM
2021-03-12 21:25:08
And we know AB = 4.
And we know AB = 4.
MaryM
2021-03-12 21:25:18
Furthermore, do you recognize those other ratios?
Furthermore, do you recognize those other ratios?
MaryM
2021-03-12 21:25:33
Hint: think about trigonometry and right triangles.
Hint: think about trigonometry and right triangles.
MaryM
2021-03-12 21:27:30
These ratios appear in the right triangles \triangle AA_1X and \triangle BB_1X. If we set \angle AXB=\theta, how can we write the ratios?
These ratios appear in the right triangles \triangle AA_1X and \triangle BB_1X. If we set \angle AXB=\theta, how can we write the ratios?
jxwis2010
2021-03-12 21:29:00
/frac{1}{\cos\theta}
/frac{1}{\cos\theta}
MaryM
2021-03-12 21:29:06
These ratios are the secant of angle \angle AXA_1 = \angle BXB_1, using the right triangles \triangle AA_1X or BB_1X. (That is, they're the reciprocal of \cos\angle AXA_1: A_1X is the "adjacent" side and AX is the hypotenuse.)
These ratios are the secant of angle \angle AXA_1 = \angle BXB_1, using the right triangles \triangle AA_1X or BB_1X. (That is, they're the reciprocal of \cos\angle AXA_1: A_1X is the "adjacent" side and AX is the hypotenuse.)
MaryM
2021-03-12 21:29:20
So if we set \theta = \angle AXB, we get \dfrac{AB}{A_1B_1} = \sec\theta = \dfrac{1}{\cos\theta}.
So if we set \theta = \angle AXB, we get \dfrac{AB}{A_1B_1} = \sec\theta = \dfrac{1}{\cos\theta}.
MaryM
2021-03-12 21:29:25
That is, A_1B_1 = AB \cos \theta.
That is, A_1B_1 = AB \cos \theta.
MaryM
2021-03-12 21:29:32
Now what?
Now what?
MaryM
2021-03-12 21:30:21
We handled one of the sides of the smaller quadrilateral. How can we handle the other ones?
We handled one of the sides of the smaller quadrilateral. How can we handle the other ones?
jxwis2010
2021-03-12 21:30:37
do the same thing for the other 3 segments
do the same thing for the other 3 segments
MaryM
2021-03-12 21:30:41
We can do the same thing on the other three sides!
We can do the same thing on the other three sides!
MaryM
2021-03-12 21:30:51
CDC_1D_1 works exactly the same way, and since \angle CXD = \angle AXB = \theta, we get C_1D_1 = CD \cos \theta as well.
CDC_1D_1 works exactly the same way, and since \angle CXD = \angle AXB = \theta, we get C_1D_1 = CD \cos \theta as well.
MaryM
2021-03-12 21:30:56
The other two are slightly different, because X is inside the circle given by the cyclic quadrilateral. For example, let's look at BCC_1B_1:
The other two are slightly different, because X is inside the circle given by the cyclic quadrilateral. For example, let's look at BCC_1B_1:
MaryM
2021-03-12 21:31:03

MaryM
2021-03-12 21:31:23
But we still get \triangle BCX \sim \triangle B_1C_1X.
But we still get \triangle BCX \sim \triangle B_1C_1X.
MaryM
2021-03-12 21:31:35
So, we get \dfrac{BC}{B_1C_1} = \dfrac{BX}{B_1X}.
So, we get \dfrac{BC}{B_1C_1} = \dfrac{BX}{B_1X}.
MaryM
2021-03-12 21:31:42
What expression do we get for B_1C_1?
What expression do we get for B_1C_1?
functionalmath
2021-03-12 21:32:06
same tihng
same tihng
Lamboreghini
2021-03-12 21:32:06
BC cos theta???
BC cos theta???
MaryM
2021-03-12 21:32:10
The right-hand side is still the reciprocal of \cos\theta using \triangle BB_1X.
The right-hand side is still the reciprocal of \cos\theta using \triangle BB_1X.
MaryM
2021-03-12 21:32:18
So we still get B_1C_1 = BC \cos \theta.
So we still get B_1C_1 = BC \cos \theta.
MaryM
2021-03-12 21:32:26
And on the other side, we get A_1D_1 = AD \cos \theta in the same way.
And on the other side, we get A_1D_1 = AD \cos \theta in the same way.
MaryM
2021-03-12 21:32:39
So, how can we write the perimeter of the smaller quadrilateral?
So, how can we write the perimeter of the smaller quadrilateral?
Zhaom
2021-03-12 21:33:10
(4+5+6+7)\cos(\theta)
(4+5+6+7)\cos(\theta)
YaoAOPS
2021-03-12 21:33:10
cos theta of each side added
cos theta of each side added
MaryM
2021-03-12 21:33:13
We get A_1B_1 + B_1C_1 + C_1D_1 + D_1A_1 = (AB + BC + CD + DA) \cos \theta.
We get A_1B_1 + B_1C_1 + C_1D_1 + D_1A_1 = (AB + BC + CD + DA) \cos \theta.
MaryM
2021-03-12 21:33:17
That is, the perimeter of the small brown quadrilateral is \cos\theta times the perimeter of ABCD.
That is, the perimeter of the small brown quadrilateral is \cos\theta times the perimeter of ABCD.
MaryM
2021-03-12 21:33:23
But we know the big perimeter!
But we know the big perimeter!
MaryM
2021-03-12 21:33:26
It's given to us: 4+5+6+7 = 22.
It's given to us: 4+5+6+7 = 22.
MaryM
2021-03-12 21:33:36
So the answer we want is \text{Perim}(A_1B_1C_1D_1) = 22\cos\theta.
So the answer we want is \text{Perim}(A_1B_1C_1D_1) = 22\cos\theta.
MaryM
2021-03-12 21:33:42
If we can figure out \cos\theta, we're done.
If we can figure out \cos\theta, we're done.
MaryM
2021-03-12 21:34:01
Let's go back to the simpler picture:
Let's go back to the simpler picture:
MaryM
2021-03-12 21:34:10

MaryM
2021-03-12 21:34:19
How can we compute the cosine of that angle?
How can we compute the cosine of that angle?
functionalmath
2021-03-12 21:34:42
cosine law
cosine law
CircleInvert
2021-03-12 21:34:46
LoC
LoC
MaryM
2021-03-12 21:34:49
Well, we have a bunch of Law of Cosines expressions that we can write down for the four small triangles.
Well, we have a bunch of Law of Cosines expressions that we can write down for the four small triangles.
MaryM
2021-03-12 21:34:53
For example, what do we get for \triangle ABX?
For example, what do we get for \triangle ABX?
ancientwarrior
2021-03-12 21:35:25
16=AX^2+BX^2-2AXBXcos(theta)
16=AX^2+BX^2-2AXBXcos(theta)
Zhaom
2021-03-12 21:35:25
BX^2+AX^2-2 \cdot BX \cdot AX\cdot\cos(\theta)
BX^2+AX^2-2 \cdot BX \cdot AX\cdot\cos(\theta)
MaryM
2021-03-12 21:35:28
We get 4^2 = (AB)^2 = (AX)^2 + (BX)^2 - 2(AX)(BX)\cos\theta.
We get 4^2 = (AB)^2 = (AX)^2 + (BX)^2 - 2(AX)(BX)\cos\theta.
MaryM
2021-03-12 21:35:32
How about \triangle BCX?
How about \triangle BCX?
Zhaom
2021-03-12 21:36:14
BX^2+CX^2-2 \cdot BX \cdot CX\cdot\cos(\theta)=25
BX^2+CX^2-2 \cdot BX \cdot CX\cdot\cos(\theta)=25
MaryM
2021-03-12 21:36:17
We get 5^2 = (BC)^2 = (BX)^2 + (CX)^2 - 2(BX)(CX)\cos(180^\circ-\theta).
We get 5^2 = (BC)^2 = (BX)^2 + (CX)^2 - 2(BX)(CX)\cos(180^\circ-\theta).
MaryM
2021-03-12 21:36:23
But \cos(180^\circ - \theta) = -\cos\theta.
But \cos(180^\circ - \theta) = -\cos\theta.
ancientwarrior
2021-03-12 21:36:31
25=BX^2+CX^2+2BXCXcos(theta)
25=BX^2+CX^2+2BXCXcos(theta)
MaryM
2021-03-12 21:36:34
So our equation is 5^2 = (BX)^2 + (CX)^2 + 2(BX)(CX)\cos\theta.
So our equation is 5^2 = (BX)^2 + (CX)^2 + 2(BX)(CX)\cos\theta.
MaryM
2021-03-12 21:36:38
The other two triangles are essentially the same, and we get the following system of four equations:
The other two triangles are essentially the same, and we get the following system of four equations:
MaryM
2021-03-12 21:36:41
\begin{align*} 4^2 &= (AX)^2 + (BX)^2 - 2(AX)(BX)\cos\theta, \\ 5^2 &= (BX)^2 + (CX)^2 + 2(BX)(CX)\cos\theta, \\ 6^2 &= (CX)^2 + (DX)^2 - 2(CX)(DX)\cos\theta, \\ 7^2 &= (DX)^2 + (AX)^2 + 2(DX)(AX)\cos\theta. \end{align*}
\begin{align*} 4^2 &= (AX)^2 + (BX)^2 - 2(AX)(BX)\cos\theta, \\ 5^2 &= (BX)^2 + (CX)^2 + 2(BX)(CX)\cos\theta, \\ 6^2 &= (CX)^2 + (DX)^2 - 2(CX)(DX)\cos\theta, \\ 7^2 &= (DX)^2 + (AX)^2 + 2(DX)(AX)\cos\theta. \end{align*}
MaryM
2021-03-12 21:36:49
What can we do with these?]
What can we do with these?]
MaryM
2021-03-12 21:36:54
Can we cancel out all the length-squared terms (like (AX)^2)?
Can we cancel out all the length-squared terms (like (AX)^2)?
CircleInvert
2021-03-12 21:37:58
Add the first and third equations, and subtract from the obtained equation the sum of the second and fourth equations
Add the first and third equations, and subtract from the obtained equation the sum of the second and fourth equations
Z_Lu
2021-03-12 21:37:58
subtract and add in alternating order?
subtract and add in alternating order?
MaryM
2021-03-12 21:38:01
Yes: if we add the two with the +2 cosine term and subtract the two with the -2 cosine term, those terms will all cancel out.
Yes: if we add the two with the +2 cosine term and subtract the two with the -2 cosine term, those terms will all cancel out.
MaryM
2021-03-12 21:38:06
We're left with -4^2 + 5^2 - 6^2 + 7^2 = 2((AX)(BX)+(BX)(CX)+(CX)(DX)+(DX)(AX))\cos\theta.
We're left with -4^2 + 5^2 - 6^2 + 7^2 = 2((AX)(BX)+(BX)(CX)+(CX)(DX)+(DX)(AX))\cos\theta.
MaryM
2021-03-12 21:38:16
The left side is -16+25-36+49 = 22.
The left side is -16+25-36+49 = 22.
MaryM
2021-03-12 21:38:24
So after dividing by 2, we get 11 = ((AX)(BX)+(BX)(CX)+(CX)(DX)+(DX)(AX))\cos\theta.
So after dividing by 2, we get 11 = ((AX)(BX)+(BX)(CX)+(CX)(DX)+(DX)(AX))\cos\theta.
MaryM
2021-03-12 21:38:27
Anything we can do with that expression involving the lengths?
Anything we can do with that expression involving the lengths?
Zhaom
2021-03-12 21:38:49
factor
factor
MaryM
2021-03-12 21:38:53
Aha! It factors!
Aha! It factors!
MaryM
2021-03-12 21:38:57
We get 11 = ((AX + CX)(BX + DX))\cos\theta.
We get 11 = ((AX + CX)(BX + DX))\cos\theta.
MaryM
2021-03-12 21:39:01
But what's that?
But what's that?
Irving1004
2021-03-12 21:39:21
Ptolemy's theorem from earlier
Ptolemy's theorem from earlier
Lamboreghini
2021-03-12 21:39:21
59 cos theta!
59 cos theta!
YaoAOPS
2021-03-12 21:39:21
ptolmey
ptolmey
rockyrockrock
2021-03-12 21:39:21
59
59
Zhaom
2021-03-12 21:39:21
AC \cdot BD\cos(\theta)=59\cos(\theta)
AC \cdot BD\cos(\theta)=59\cos(\theta)
rockyrockrock
2021-03-12 21:39:25
ac*bd
ac*bd
MaryM
2021-03-12 21:39:28
AX+CX = AC and BX+DX = BD.
AX+CX = AC and BX+DX = BD.
MaryM
2021-03-12 21:39:34
So this is 11 = (AC)(BD)\cos\theta.
So this is 11 = (AC)(BD)\cos\theta.
MaryM
2021-03-12 21:39:43
But we computed (AC)(BD) = 59 way back at the beginning!
But we computed (AC)(BD) = 59 way back at the beginning!
MaryM
2021-03-12 21:39:50
So we have 11 = 59\cos\theta, hence \cos\theta = \dfrac{11}{59}.
So we have 11 = 59\cos\theta, hence \cos\theta = \dfrac{11}{59}.
MaryM
2021-03-12 21:39:59
What's the answer, then?
What's the answer, then?
happyhari
2021-03-12 21:40:18
22(11/59) = 242/59
22(11/59) = 242/59
Zhaom
2021-03-12 21:40:18
242+59=\boxed{301}
242+59=\boxed{301}
Irving1004
2021-03-12 21:40:18
242/59
242/59
happyhari
2021-03-12 21:40:18
242/59
242/59
rockyrockrock
2021-03-12 21:40:20
301
301
MaryM
2021-03-12 21:40:22
The perimeter we wanted was 22\cos\theta.
The perimeter we wanted was 22\cos\theta.
MaryM
2021-03-12 21:40:26
So it's 22 \cdot \dfrac{11}{59} = \dfrac{242}{59}.
So it's 22 \cdot \dfrac{11}{59} = \dfrac{242}{59}.
MaryM
2021-03-12 21:40:30
This is in lowest terms, so our final answer is 242 + 59 = \boxed{301}.
This is in lowest terms, so our final answer is 242 + 59 = \boxed{301}.
ancientwarrior
2021-03-12 21:40:33
cool
cool
MaryM
2021-03-12 21:40:54
12. Let A_1A_2A_3 \dots A_{12} be a dodecagon (12-gon). Three frogs initially sit at A_4, A_8, and A_{12}. At the end of each minute, simultaneously each of the three frogs jumps to one of the two vertices adjacent to its current position, chosen randomly and independently with both choices being equally likely. All three frogs stop jumping as soon as two frogs arrive at the same vertex at the same time. The expected number of minutes until the frogs stop jumping is \dfrac{m}{n}, where m and n are relatively prime positive integers. Find m+n.
12. Let A_1A_2A_3 \dots A_{12} be a dodecagon (12-gon). Three frogs initially sit at A_4, A_8, and A_{12}. At the end of each minute, simultaneously each of the three frogs jumps to one of the two vertices adjacent to its current position, chosen randomly and independently with both choices being equally likely. All three frogs stop jumping as soon as two frogs arrive at the same vertex at the same time. The expected number of minutes until the frogs stop jumping is \dfrac{m}{n}, where m and n are relatively prime positive integers. Find m+n.
MaryM
2021-03-12 21:41:03
Let's draw a diagram of the frogs:
Let's draw a diagram of the frogs:
MaryM
2021-03-12 21:41:08

MaryM
2021-03-12 21:41:21
Let's call this position S (for "Start").
Let's call this position S (for "Start").
MaryM
2021-03-12 21:41:28
What can happen on the first move?
What can happen on the first move?
ancientwarrior
2021-03-12 21:42:08
two frogs get closer to each other or they all stay relatively the same
two frogs get closer to each other or they all stay relatively the same
Zhaom
2021-03-12 21:42:08
each of the frogs can either hop clockwise or counterclockwise 1 vertex
each of the frogs can either hop clockwise or counterclockwise 1 vertex
Irving1004
2021-03-12 21:42:08
8 things, two where essentially start over
8 things, two where essentially start over
MaryM
2021-03-12 21:42:15
They might all move in the same direction: that is, they might all move clockwise or counterclockwise.
They might all move in the same direction: that is, they might all move clockwise or counterclockwise.
MaryM
2021-03-12 21:42:22
That's doesn't change their relative position, so it essentially keeps us at S.
That's doesn't change their relative position, so it essentially keeps us at S.
MaryM
2021-03-12 21:42:27
The only other possibility is that two of them move in one direction and the third moves in the other direction.
The only other possibility is that two of them move in one direction and the third moves in the other direction.
MaryM
2021-03-12 21:42:32
For example, if the top and left frogs both move clockwise, but the right frog moves counterclockwise, we end up here:
For example, if the top and left frogs both move clockwise, but the right frog moves counterclockwise, we end up here:
MaryM
2021-03-12 21:42:36

MaryM
2021-03-12 21:42:42
Does it matter which frogs move in which direction?
Does it matter which frogs move in which direction?
Zhaom
2021-03-12 21:42:56
no
no
ancientwarrior
2021-03-12 21:42:56
not really
not really
Lamboreghini
2021-03-12 21:43:00
no, the relative position is the same
no, the relative position is the same
functionalmath
2021-03-12 21:43:01
no
no
MaryM
2021-03-12 21:43:04
Not really. The two frogs that move in the same direction will still be 4 vertices apart. The other frog will be distance 2 from whichever frog it moves towards (and distance 6 from the other frog). But all these positions are essentially the same by symmetry.
Not really. The two frogs that move in the same direction will still be 4 vertices apart. The other frog will be distance 2 from whichever frog it moves towards (and distance 6 from the other frog). But all these positions are essentially the same by symmetry.
MaryM
2021-03-12 21:43:09
Let's call this position "A" (for, um, "Amazing position").
Let's call this position "A" (for, um, "Amazing position").
MaryM
2021-03-12 21:43:28
What's the probability that we move S-to-S?
What's the probability that we move S-to-S?
Zhaom
2021-03-12 21:43:51
\frac{1}{4}
\frac{1}{4}
Irving1004
2021-03-12 21:43:51
1/4
1/4
MaryM
2021-03-12 21:43:54
Imagine they move one at a time. The second and third frogs have to copy what the first frog does.
Imagine they move one at a time. The second and third frogs have to copy what the first frog does.
MaryM
2021-03-12 21:43:58
They each do this with probability \dfrac12, so they both do it with probability \dfrac14.
They each do this with probability \dfrac12, so they both do it with probability \dfrac14.
MaryM
2021-03-12 21:44:02
So we go from S to S with probability \dfrac14.
So we go from S to S with probability \dfrac14.
MaryM
2021-03-12 21:44:08
And thus we go from S to A with probability \dfrac34.
And thus we go from S to A with probability \dfrac34.
MaryM
2021-03-12 21:44:13
Let's make a chart of what we have so far:
Let's make a chart of what we have so far:
MaryM
2021-03-12 21:44:20
\begin{array}{r|ll} \text{From }\downarrow & \text{S} & \text{A} \\ \hline \text{S} & \dfrac14 & \dfrac34 \\[1.5ex] \text{A} & \end{array}
\begin{array}{r|ll} \text{From }\downarrow & \text{S} & \text{A} \\ \hline \text{S} & \dfrac14 & \dfrac34 \\[1.5ex] \text{A} & \end{array}
MaryM
2021-03-12 21:44:27
Now what can happen starting from position A?
Now what can happen starting from position A?
MaryM
2021-03-12 21:44:36

MaryM
2021-03-12 21:45:05
Can we win from this position?
Can we win from this position?
functionalmath
2021-03-12 21:45:19
yes
yes
Hector2173
2021-03-12 21:45:19
yes
yes
klpiguy
2021-03-12 21:45:19
yup
yup
donguri
2021-03-12 21:45:19
Yes
Yes
MaryM
2021-03-12 21:45:21
We might win!
We might win!
MaryM
2021-03-12 21:45:25
If the two frogs that are 2 apart move towards each other, they land on top of each other, and we win.
If the two frogs that are 2 apart move towards each other, they land on top of each other, and we win.
MaryM
2021-03-12 21:45:28
What's the probability of that happening?
What's the probability of that happening?
Lamboreghini
2021-03-12 21:45:46
1/4
1/4
happyhari
2021-03-12 21:45:46
1/4
1/4
ancientwarrior
2021-03-12 21:45:46
1/4
1/4
Zhaom
2021-03-12 21:45:46
\frac{1}{4}
\frac{1}{4}
Hector2173
2021-03-12 21:45:48
1/4
1/4
MaryM
2021-03-12 21:45:50
Each of the bottom two frogs must pick the right direction. (It doesn't matter what the third frog does.)
Each of the bottom two frogs must pick the right direction. (It doesn't matter what the third frog does.)
MaryM
2021-03-12 21:45:59
This happens with probability \dfrac12 \cdot \dfrac12 = \dfrac14.
This happens with probability \dfrac12 \cdot \dfrac12 = \dfrac14.
MaryM
2021-03-12 21:46:03
So let's update the chart:
So let's update the chart:
MaryM
2021-03-12 21:46:19
\begin{array}{r|ccc} \text{From }\downarrow & \text{S} & \text{A} & \text{Win} \\ \hline \text{S} & \dfrac14 & \dfrac34 \\[1.5ex] \text{A} & & & \dfrac14 \end{array}
\begin{array}{r|ccc} \text{From }\downarrow & \text{S} & \text{A} & \text{Win} \\ \hline \text{S} & \dfrac14 & \dfrac34 \\[1.5ex] \text{A} & & & \dfrac14 \end{array}
MaryM
2021-03-12 21:46:49
Can we still be at A after the frogs move?
Can we still be at A after the frogs move?
Zhaom
2021-03-12 21:47:11
yes
yes
Hector2173
2021-03-12 21:47:11
yes
yes
happyhari
2021-03-12 21:47:11
yes
yes
klpiguy
2021-03-12 21:47:11
yes
yes
MaryM
2021-03-12 21:47:13
Yes, they could all move in the same direction. That keeps us at A.
Yes, they could all move in the same direction. That keeps us at A.
MaryM
2021-03-12 21:47:17
And what's the probability that this happens?
And what's the probability that this happens?
klpiguy
2021-03-12 21:47:34
1/4
1/4
Zhaom
2021-03-12 21:47:34
\frac{1}{4}
\frac{1}{4}
Z_Lu
2021-03-12 21:47:34
1/4
1/4
happyhari
2021-03-12 21:47:34
1/4
1/4
MaryM
2021-03-12 21:47:36
It's \dfrac14, just like before.
It's \dfrac14, just like before.
MaryM
2021-03-12 21:47:52
The two bottom frogs could also move in the same direction, but the other frog move opposite.
The two bottom frogs could also move in the same direction, but the other frog move opposite.
MaryM
2021-03-12 21:47:57
What happens then?
What happens then?
klpiguy
2021-03-12 21:48:29
A NEW POSITION!!!
A NEW POSITION!!!
Zhaom
2021-03-12 21:48:29
new position
new position
Lamboreghini
2021-03-12 21:48:29
then, the frogs are equally spaced with 1 place between each
then, the frogs are equally spaced with 1 place between each
MaryM
2021-03-12 21:48:32
They could move towards each other, like so:
They could move towards each other, like so:
MaryM
2021-03-12 21:48:35

MaryM
2021-03-12 21:48:41
This is a new position. Let's call it B (for, um, "Beautiful"?).
This is a new position. Let's call it B (for, um, "Beautiful"?).
MaryM
2021-03-12 21:48:45
What's the probability of this?
What's the probability of this?
Lamboreghini
2021-03-12 21:49:11
1/8
1/8
happyhari
2021-03-12 21:49:11
1/8
1/8
Zhaom
2021-03-12 21:49:11
\frac{1}{8}
\frac{1}{8}
klpiguy
2021-03-12 21:49:15
1/8
1/8
Irving1004
2021-03-12 21:49:17
1.8
1.8
Irving1004
2021-03-12 21:49:23
1/8
1/8
MaryM
2021-03-12 21:49:24
All three frogs have to move in the "right" direction to get this.
All three frogs have to move in the "right" direction to get this.
MaryM
2021-03-12 21:49:30
So it's \left(\dfrac12\right)^3 = \dfrac18.
So it's \left(\dfrac12\right)^3 = \dfrac18.
MaryM
2021-03-12 21:49:33
What if the bottom two move in the same direction, but the third moves away from them?
What if the bottom two move in the same direction, but the third moves away from them?
Hector2173
2021-03-12 21:50:16
A
A
happyhari
2021-03-12 21:50:16
same position as A
same position as A
Zhaom
2021-03-12 21:50:16
still A
still A
MaryM
2021-03-12 21:50:18
We end up here:
We end up here:
MaryM
2021-03-12 21:50:23

MaryM
2021-03-12 21:50:27
This is A again (though it's a mirror image).
This is A again (though it's a mirror image).
MaryM
2021-03-12 21:50:37
So that's another \dfrac18 that we stay in A. Combined with the \dfrac14 from before, that's \dfrac38 total.
So that's another \dfrac18 that we stay in A. Combined with the \dfrac14 from before, that's \dfrac38 total.
MaryM
2021-03-12 21:51:03
The two close frogs in A could also move away from each other.
The two close frogs in A could also move away from each other.
MaryM
2021-03-12 21:51:12
Then third frog ends up in one of the red positions here:
Then third frog ends up in one of the red positions here:
MaryM
2021-03-12 21:51:18

MaryM
2021-03-12 21:51:21
If it's the top red dot, what position are we in?
If it's the top red dot, what position are we in?
Zhaom
2021-03-12 21:51:36
S
S
Lamboreghini
2021-03-12 21:51:36
S
S
happyhari
2021-03-12 21:51:36
S
S
Z_Lu
2021-03-12 21:51:36
S
S
MaryM
2021-03-12 21:51:39
We're back to S.
We're back to S.
MaryM
2021-03-12 21:51:43
If it's the bottom red dot, what position are we in?
If it's the bottom red dot, what position are we in?
Lamboreghini
2021-03-12 21:51:55
A
A
Z_Lu
2021-03-12 21:51:55
A
A
Zhaom
2021-03-12 21:51:55
A again
A again
ancientwarrior
2021-03-12 21:51:55
A
A
Hector2173
2021-03-12 21:51:55
AA
AA
happyhari
2021-03-12 21:51:55
A
A
MaryM
2021-03-12 21:51:57
We'll still in A.
We'll still in A.
MaryM
2021-03-12 21:52:01
And that's another \dfrac18 probability for each.
And that's another \dfrac18 probability for each.
MaryM
2021-03-12 21:52:06
Note that's now a total of \dfrac12 that we stay in A.
Note that's now a total of \dfrac12 that we stay in A.
MaryM
2021-03-12 21:52:11
So here's the updated chart with all this data:
So here's the updated chart with all this data:
MaryM
2021-03-12 21:52:16
\begin{array}{r|cccc} \text{From }\downarrow & \text{S} & \text{A} & \text{B} & \text{Win} \\ \hline \text{S} & \dfrac14 & \dfrac34 \\[2.5ex] \text{A} & \dfrac18 & \dfrac12 & \dfrac18 & \dfrac14 \\[2.5ex] \text{B} & \end{array}
\begin{array}{r|cccc} \text{From }\downarrow & \text{S} & \text{A} & \text{B} & \text{Win} \\ \hline \text{S} & \dfrac14 & \dfrac34 \\[2.5ex] \text{A} & \dfrac18 & \dfrac12 & \dfrac18 & \dfrac14 \\[2.5ex] \text{B} & \end{array}
MaryM
2021-03-12 21:52:20
What's next?
What's next?
Zhaom
2021-03-12 21:52:41
do cases for B
do cases for B
klpiguy
2021-03-12 21:52:41
we use B
we use B
Z_Lu
2021-03-12 21:52:41
starting from b postion
starting from b postion
MaryM
2021-03-12 21:52:45
We need to figure out what can happen from B:
We need to figure out what can happen from B:
MaryM
2021-03-12 21:52:53

MaryM
2021-03-12 21:52:59
What's the probability we win from here?
What's the probability we win from here?
klpiguy
2021-03-12 21:53:24
1/2
1/2
happyhari
2021-03-12 21:53:24
1/2
1/2
Zhaom
2021-03-12 21:53:24
\frac{1}{2}
\frac{1}{2}
rockyrockrock
2021-03-12 21:53:24
1/2
1/2
MaryM
2021-03-12 21:53:28
The middle frog has to move one way or the other.
The middle frog has to move one way or the other.
MaryM
2021-03-12 21:53:33
In either case, if the frog on that side moves towards the middle frog, we win!
In either case, if the frog on that side moves towards the middle frog, we win!
MaryM
2021-03-12 21:53:55
The probability of this happening is \dfrac12. The outside frog has to make the right choice.
The probability of this happening is \dfrac12. The outside frog has to make the right choice.
MaryM
2021-03-12 21:54:01
So we win with probability \dfrac12.
So we win with probability \dfrac12.
MaryM
2021-03-12 21:54:06
What else could happen?
What else could happen?
Zhaom
2021-03-12 21:54:35
the frogs move in the same direction
the frogs move in the same direction
ancientwarrior
2021-03-12 21:54:35
they all shift in same direction
they all shift in same direction
MaryM
2021-03-12 21:54:37
As usual, all three frogs could move in the same direction with probability \dfrac14. This keeps us in B.
As usual, all three frogs could move in the same direction with probability \dfrac14. This keeps us in B.
MaryM
2021-03-12 21:55:02
It's also possible that the middle frog moves in one direction, and the other frog in that direction also goes in that direction, but the third frog goes in the other direction.
It's also possible that the middle frog moves in one direction, and the other frog in that direction also goes in that direction, but the third frog goes in the other direction.
MaryM
2021-03-12 21:55:07
Where does that take us?
Where does that take us?
ancientwarrior
2021-03-12 21:55:32
i think that's A?
i think that's A?
Z_Lu
2021-03-12 21:55:32
A
A
functionalmath
2021-03-12 21:55:36
A
A
MaryM
2021-03-12 21:55:38
It takes us back to A.
It takes us back to A.
MaryM
2021-03-12 21:55:44
And this occurs with probability \dfrac14, because the other two frogs have to move a certain way relative to the middle frog.
And this occurs with probability \dfrac14, because the other two frogs have to move a certain way relative to the middle frog.
MaryM
2021-03-12 21:55:49
So here's the final chart:
So here's the final chart:
MaryM
2021-03-12 21:56:10
\begin{array}{r|cccc} \text{From }\downarrow & \text{S} & \text{A} & \text{B} & \text{Win} \\ \hline \text{S} & \dfrac14 & \dfrac34 \\[2.5ex] \text{A} & \dfrac18 & \dfrac12 & \dfrac18 & \dfrac14 \\[2.5ex] \text{B} & & \dfrac14 & \dfrac14 & \dfrac12 \end{array}
\begin{array}{r|cccc} \text{From }\downarrow & \text{S} & \text{A} & \text{B} & \text{Win} \\ \hline \text{S} & \dfrac14 & \dfrac34 \\[2.5ex] \text{A} & \dfrac18 & \dfrac12 & \dfrac18 & \dfrac14 \\[2.5ex] \text{B} & & \dfrac14 & \dfrac14 & \dfrac12 \end{array}
MaryM
2021-03-12 21:56:18
As a check, each row sums to 1, so we've captured all the possible moves from each position.
As a check, each row sums to 1, so we've captured all the possible moves from each position.
MaryM
2021-03-12 21:56:33
Now, what do we do with this chart?
Now, what do we do with this chart?
Zhaom
2021-03-12 21:57:00
use states
use states
functionalmath
2021-03-12 21:57:00
find expected value
find expected value
MaryM
2021-03-12 21:57:05
We can write down equations for the expected number of moves until we win.
We can write down equations for the expected number of moves until we win.
MaryM
2021-03-12 21:57:09
Let's set s,a,b to be the expected number of moves needed to win from S, A, B, respectively.
Let's set s,a,b to be the expected number of moves needed to win from S, A, B, respectively.
MaryM
2021-03-12 21:57:15
What's the equation for s?
What's the equation for s?
MaryM
2021-03-12 21:57:49
Hint: We spend 1 move, and we end up back at S (with probability \dfrac14) or at A (with probability \dfrac34). If we end up at S, we expect to need s more moves to win. If we end up at A, we expect to need a more moves to win.
Hint: We spend 1 move, and we end up back at S (with probability \dfrac14) or at A (with probability \dfrac34). If we end up at S, we expect to need s more moves to win. If we end up at A, we expect to need a more moves to win.
MaryM
2021-03-12 21:58:21
Don't forget to account for the first move we make.
Don't forget to account for the first move we make.
Zhaom
2021-03-12 21:59:21
s=\frac{s+1}{4}+\frac{3(a+1)}{4}
s=\frac{s+1}{4}+\frac{3(a+1)}{4}
ancientwarrior
2021-03-12 21:59:21
s=1/4(s+1)+3/4(a+1)
s=1/4(s+1)+3/4(a+1)
MaryM
2021-03-12 21:59:26
We get s = 1 + \dfrac14s + \dfrac34a. The 1 is the first move, and then we need s or a more moves, depending on which position we land on.
We get s = 1 + \dfrac14s + \dfrac34a. The 1 is the first move, and then we need s or a more moves, depending on which position we land on.
MaryM
2021-03-12 21:59:41
In a similar fashion, what's the equation for a?
In a similar fashion, what's the equation for a?
ancientwarrior
2021-03-12 22:00:50
a=1/8(s+1)+1/2(a+1)+1/8(b+1)+1/4
a=1/8(s+1)+1/2(a+1)+1/8(b+1)+1/4
Zhaom
2021-03-12 22:00:50
a=\frac{s+1}{8}+\frac{a+1}{2}+\frac{b+1}{8}+\frac{1}{4}
a=\frac{s+1}{8}+\frac{a+1}{2}+\frac{b+1}{8}+\frac{1}{4}
MaryM
2021-03-12 22:00:54
We have a = 1 + \dfrac18s + \dfrac12a + \dfrac18b. (If we win, we don't need to make any more moves -- the game is over.)
We have a = 1 + \dfrac18s + \dfrac12a + \dfrac18b. (If we win, we don't need to make any more moves -- the game is over.)
MaryM
2021-03-12 22:01:05
And what's the equation for b?
And what's the equation for b?
Zhaom
2021-03-12 22:02:06
b=\frac{a+1}{4}+\frac{b+1}{4}+\frac{1}{2}
b=\frac{a+1}{4}+\frac{b+1}{4}+\frac{1}{2}
functionalmath
2021-03-12 22:02:06
(a+1)/4+(b+1)/4+1/2
(a+1)/4+(b+1)/4+1/2
ancientwarrior
2021-03-12 22:02:06
b=1/4(a+1)+1/4(b+1)+1/2
b=1/4(a+1)+1/4(b+1)+1/2
rockyrockrock
2021-03-12 22:02:06
b=1/4(a+1)+1/4(b+1)+1/2
b=1/4(a+1)+1/4(b+1)+1/2
MaryM
2021-03-12 22:02:09
We have b = 1 + \dfrac14a + \dfrac14b.
We have b = 1 + \dfrac14a + \dfrac14b.
MaryM
2021-03-12 22:02:15
So we have a system of three linear equations in three variables:
So we have a system of three linear equations in three variables:
MaryM
2021-03-12 22:02:38
\begin{align*} s &= 1 + \dfrac14s + \dfrac34a, \\ a &= 1 + \dfrac18s + \dfrac12a + \dfrac18b, \\ b &= 1 + \dfrac14a + \dfrac14b. \end{align*}
\begin{align*} s &= 1 + \dfrac14s + \dfrac34a, \\ a &= 1 + \dfrac18s + \dfrac12a + \dfrac18b, \\ b &= 1 + \dfrac14a + \dfrac14b. \end{align*}
MaryM
2021-03-12 22:02:44
Now what?
Now what?
rockyrockrock
2021-03-12 22:03:10
SOLVE!
SOLVE!
MaryM
2021-03-12 22:03:14
We need to solve for s to solve the problem.
We need to solve for s to solve the problem.
MaryM
2021-03-12 22:03:23
Let's clean these up by clearing denominators and moving the variables to one side.
Let's clean these up by clearing denominators and moving the variables to one side.
MaryM
2021-03-12 22:03:30
\begin{align*} 3s - 3a &= 4, \\ -s + 4a - b &= 8, \\ -a + 3b &= 4. \end{align*}
\begin{align*} 3s - 3a &= 4, \\ -s + 4a - b &= 8, \\ -a + 3b &= 4. \end{align*}
MaryM
2021-03-12 22:03:40
We can eliminate b by adding 3 times the second equation to the third.
We can eliminate b by adding 3 times the second equation to the third.
MaryM
2021-03-12 22:03:48
And then we have: \begin{align*} 3s - 3a &= 4, \\ -3s + 11a &= 28. \end{align*}
And then we have: \begin{align*} 3s - 3a &= 4, \\ -3s + 11a &= 28. \end{align*}
ancientwarrior
2021-03-12 22:03:49
we need to solve for s in the end
we need to solve for s in the end
Irving1004
2021-03-12 22:03:49
solve for s!
solve for s!
MaryM
2021-03-12 22:03:54
We eliminate a by multiplying the top equation by 11 and the bottom one by 3:
We eliminate a by multiplying the top equation by 11 and the bottom one by 3:
MaryM
2021-03-12 22:03:59
\begin{align*} 33s - 33a &= 44, \\ -9s + 33a &= 84. \end{align*}
\begin{align*} 33s - 33a &= 44, \\ -9s + 33a &= 84. \end{align*}
MaryM
2021-03-12 22:04:04
And then adding gives 24s = 128.
And then adding gives 24s = 128.
MaryM
2021-03-12 22:04:14
What is the answer?
What is the answer?
rockyrockrock
2021-03-12 22:04:30
s=16/3
s=16/3
rockyrockrock
2021-03-12 22:04:30
19
19
functionalmath
2021-03-12 22:04:30
16
16
MaryM
2021-03-12 22:04:33
So s = \dfrac{128}{24} = \dfrac{16}{3}.
So s = \dfrac{128}{24} = \dfrac{16}{3}.
MaryM
2021-03-12 22:04:37
And our final answer is 16 + 3 = \boxed{019}.
And our final answer is 16 + 3 = \boxed{019}.
MaryM
2021-03-12 22:05:06
Only three problems left!
Only three problems left!
happyhari
2021-03-12 22:05:17
yay!
yay!
Zhaom
2021-03-12 22:05:22
yay
yay
MaryM
2021-03-12 22:05:31
13. Circles \omega_1 and \omega_2 with radii 961 and 625, respectively, intersect at distinct points A and B. A third circle \omega is externally tangent to both \omega_1 and \omega_2. Suppose line AB intersects \omega at two points P and Q such that the measure of minor arc \stackrel{\displaystyle\frown}{PQ} is 120^\circ\!. Find the distance between the centers of \omega_1 and \omega_2.
13. Circles \omega_1 and \omega_2 with radii 961 and 625, respectively, intersect at distinct points A and B. A third circle \omega is externally tangent to both \omega_1 and \omega_2. Suppose line AB intersects \omega at two points P and Q such that the measure of minor arc \stackrel{\displaystyle\frown}{PQ} is 120^\circ\!. Find the distance between the centers of \omega_1 and \omega_2.
functionalmath
2021-03-12 22:05:47
daigram!
daigram!
Zhaom
2021-03-12 22:05:47
diagram
diagram
MaryM
2021-03-12 22:05:49
Let's try to sketch a picture.
Let's try to sketch a picture.
MaryM
2021-03-12 22:05:54
It's a little bit hard to draw because we don't know if \overline{AB} crosses the segment connecting the two centers or not.
It's a little bit hard to draw because we don't know if \overline{AB} crosses the segment connecting the two centers or not.
MaryM
2021-03-12 22:05:57
That is, we could have something like this:
That is, we could have something like this:
MaryM
2021-03-12 22:06:01

MaryM
2021-03-12 22:06:06
Or we could have something like this:
Or we could have something like this:
MaryM
2021-03-12 22:06:11

MaryM
2021-03-12 22:06:25
For now, let's use the first one, and sketch the third circle and \overline{BAPQ}:
For now, let's use the first one, and sketch the third circle and \overline{BAPQ}:
MaryM
2021-03-12 22:06:46

MaryM
2021-03-12 22:06:59
That looks reasonable. I could believe that arc PQ is 120^\circ in this picture.
That looks reasonable. I could believe that arc PQ is 120^\circ in this picture.
MaryM
2021-03-12 22:07:09
Let's also set d = O_1O_2 to be what we're looking for.
Let's also set d = O_1O_2 to be what we're looking for.
MaryM
2021-03-12 22:07:33
Do we know OO_1 and OO_2?
Do we know OO_1 and OO_2?
functionalmath
2021-03-12 22:08:06
yes
yes
panda2018
2021-03-12 22:08:06
961+r,625+r
961+r,625+r
Irving1004
2021-03-12 22:08:06
r+961 and r+625
r+961 and r+625
MaryM
2021-03-12 22:08:13
Yes: they connect two tangent circles, so these lengths are the sum of the radii.
Yes: they connect two tangent circles, so these lengths are the sum of the radii.
MaryM
2021-03-12 22:08:17
That is, we have OO_1 = 961+r and OO_2 = 625+r.
That is, we have OO_1 = 961+r and OO_2 = 625+r.
MaryM
2021-03-12 22:08:22
And we know that \angle POQ = 120^\circ.
And we know that \angle POQ = 120^\circ.
MaryM
2021-03-12 22:08:27
How can we get that angle to connect to the rest of the picture?
How can we get that angle to connect to the rest of the picture?
functionalmath
2021-03-12 22:09:04
connect OP and OQ
connect OP and OQ
MaryM
2021-03-12 22:09:10
One idea is to draw line OP. In particular, we can make the blue triangle shown:
One idea is to draw line OP. In particular, we can make the blue triangle shown:
MaryM
2021-03-12 22:09:17

MaryM
2021-03-12 22:09:32
Do you notice anything interesting about that triangle?
Do you notice anything interesting about that triangle?
ancientwarrior
2021-03-12 22:10:18
looks like a right triangle
looks like a right triangle
panda2018
2021-03-12 22:10:18
<D is right
<D is right
Irving1004
2021-03-12 22:10:18
Is it right
Is it right
MaryM
2021-03-12 22:10:26
It is a right triangle, but we know more than that.
It is a right triangle, but we know more than that.
Zhaom
2021-03-12 22:10:48
is a 30-60-90 triangle
is a 30-60-90 triangle
CircleInvert
2021-03-12 22:10:48
30-60-90
30-60-90
MaryM
2021-03-12 22:11:27
The blue triangle \triangle OCD is a 30-60-90 right triangle!
The blue triangle \triangle OCD is a 30-60-90 right triangle!
Zhaom
2021-03-12 22:11:39
wait, what's D?
wait, what's D?
MaryM
2021-03-12 22:11:52
Right, I think we didn't define D.
Right, I think we didn't define D.
MaryM
2021-03-12 22:12:14
But it is the intersection of AB and a perpendicular from O.
But it is the intersection of AB and a perpendicular from O.
MaryM
2021-03-12 22:12:17
Sorry about that.
Sorry about that.
Zhaom
2021-03-12 22:13:15
do you mean O_1O_2, not AB?
do you mean O_1O_2, not AB?
MaryM
2021-03-12 22:13:21
Yes! Sorry again.
Yes! Sorry again.
MaryM
2021-03-12 22:13:40
It's definitely not on AB.
It's definitely not on AB.
YaoAOPS
2021-03-12 22:14:14
Why is it 30-60-90
Why is it 30-60-90
MaryM
2021-03-12 22:14:23
That's because \angle OPQ=30^\circ.
That's because \angle OPQ=30^\circ.
MaryM
2021-03-12 22:14:42
Which follows from the fact that \angle POQ= 120^\circ.
Which follows from the fact that \angle POQ= 120^\circ.
Irving1004
2021-03-12 22:14:53
Yeah
Yeah
MaryM
2021-03-12 22:15:26
Well, we know that \angle OCO_1 = 120^\circ and \angle OCO_2 = 60^\circ.
Well, we know that \angle OCO_1 = 120^\circ and \angle OCO_2 = 60^\circ.
MaryM
2021-03-12 22:15:47
I don't have any really great ideas, but I know we eventually need to compute the length O_1O_2, which splits as O_1C + CO_2.
I don't have any really great ideas, but I know we eventually need to compute the length O_1O_2, which splits as O_1C + CO_2.
MaryM
2021-03-12 22:15:53
And we know a nice angle at C.
And we know a nice angle at C.
MaryM
2021-03-12 22:16:00
So maybe we can set up some Law of Cosines computations for some of these triangles to try to get at these lengths.
So maybe we can set up some Law of Cosines computations for some of these triangles to try to get at these lengths.
MaryM
2021-03-12 22:16:07
For example, we can write down the Law of Cosines for \triangle OCO_1, using angle \angle OCO_1 = 120^\circ.
For example, we can write down the Law of Cosines for \triangle OCO_1, using angle \angle OCO_1 = 120^\circ.
MaryM
2021-03-12 22:16:11
What do we get?
What do we get?
Zhaom
2021-03-12 22:16:55
O_1O^2=CO^2+CO_1^2+CO \cdot CO_1
O_1O^2=CO^2+CO_1^2+CO \cdot CO_1
MaryM
2021-03-12 22:16:58
We get (OO_1)^2 = (OC)^2 + (O_1C)^2 - 2(OC)(O_1C)\cos(120^\circ).
We get (OO_1)^2 = (OC)^2 + (O_1C)^2 - 2(OC)(O_1C)\cos(120^\circ).
MaryM
2021-03-12 22:17:10
But \cos(120^\circ)=\frac 12.
But \cos(120^\circ)=\frac 12.
MaryM
2021-03-12 22:17:15
So this is (OO_1)^2 = (OC)^2 + (O_1C)^2 + (OC)(O_1C).
So this is (OO_1)^2 = (OC)^2 + (O_1C)^2 + (OC)(O_1C).
MaryM
2021-03-12 22:17:20
That's not too bad, I guess.
That's not too bad, I guess.
MaryM
2021-03-12 22:17:27
We can do something similar for \triangle OCO_2.
We can do something similar for \triangle OCO_2.
MaryM
2021-03-12 22:17:32
What do we get?
What do we get?
Zhaom
2021-03-12 22:17:46
Do you mean \cos\left(120^\circ\right)=-\frac{1}{2}?
Do you mean \cos\left(120^\circ\right)=-\frac{1}{2}?
MaryM
2021-03-12 22:18:03
So many typos! Yes, it's -\frac 12.
So many typos! Yes, it's -\frac 12.
MaryM
2021-03-12 22:18:31
But well, what do we get for \triangle OCO_2 using the law of Cosines?
But well, what do we get for \triangle OCO_2 using the law of Cosines?
Zhaom
2021-03-12 22:19:55
I meant O_1O^2=CO^2+CO_1^2-CO \cdot CO_1
I meant O_1O^2=CO^2+CO_1^2-CO \cdot CO_1
MaryM
2021-03-12 22:19:58
We get essentially the same thing, except now we're using \cos \angle OCO_2 = \cos 60^\circ=\frac 12.
We get essentially the same thing, except now we're using \cos \angle OCO_2 = \cos 60^\circ=\frac 12.
MaryM
2021-03-12 22:20:09
(I guess typing is getting harder and harder.)
(I guess typing is getting harder and harder.)
MaryM
2021-03-12 22:20:22
But well, we get (OO_2)^2 = (OC)^2 + (O_2C)^2 - (OC)(O_2C).
But well, we get (OO_2)^2 = (OC)^2 + (O_2C)^2 - (OC)(O_2C).
MaryM
2021-03-12 22:20:32
Let's put these side-by-side and see if we can do anything:
Let's put these side-by-side and see if we can do anything:
MaryM
2021-03-12 22:20:37
\begin{align*} (OO_1)^2 &= (OC)^2 + (O_1C)^2 + (OC)(O_1C), \\ (OO_2)^2 &= (OC)^2 + (O_2C)^2 - (OC)(O_2C). \end{align*}
\begin{align*} (OO_1)^2 &= (OC)^2 + (O_1C)^2 + (OC)(O_1C), \\ (OO_2)^2 &= (OC)^2 + (O_2C)^2 - (OC)(O_2C). \end{align*}
MaryM
2021-03-12 22:20:42
What can we do with these equations?
What can we do with these equations?
Lamboreghini
2021-03-12 22:21:18
we can get rid of OC^2!
we can get rid of OC^2!
panda2018
2021-03-12 22:21:18
subtract them
subtract them
happyhari
2021-03-12 22:21:18
subtract and get rid of (OC)^2
subtract and get rid of (OC)^2
Irving1004
2021-03-12 22:21:18
subtract?
subtract?
Zhaom
2021-03-12 22:21:18
subtract the second from the first
subtract the second from the first
MaryM
2021-03-12 22:21:22
When we subtract them, the (OC)^2 term cancels, and we get (OO_1)^2 - (OO_2)^2 = (O_1C)^2 - (O_2C)^2 + (OC)(O_1C + O_2C).
When we subtract them, the (OC)^2 term cancels, and we get (OO_1)^2 - (OO_2)^2 = (O_1C)^2 - (O_2C)^2 + (OC)(O_1C + O_2C).
MaryM
2021-03-12 22:21:29
Hey...what that last term at the end, O_1C + O_2C?
Hey...what that last term at the end, O_1C + O_2C?
panda2018
2021-03-12 22:21:47
O_1O_2
O_1O_2
Lamboreghini
2021-03-12 22:21:47
O1O2
O1O2
MaryM
2021-03-12 22:21:50
That's O_1O_2. That's what we want to find!
That's O_1O_2. That's what we want to find!
MaryM
2021-03-12 22:21:54
We called it d, remember?
We called it d, remember?
MaryM
2021-03-12 22:21:59
So let's substitute d in: (OO_1)^2 - (OO_2)^2 = (O_1C)^2 - (O_2C)^2 + (OC)d.
So let's substitute d in: (OO_1)^2 - (OO_2)^2 = (O_1C)^2 - (O_2C)^2 + (OC)d.
MaryM
2021-03-12 22:22:05
So I think maybe we're making progress.
So I think maybe we're making progress.
MaryM
2021-03-12 22:22:18
But I don't really know what to do with the other terms that involve point C.
But I don't really know what to do with the other terms that involve point C.
MaryM
2021-03-12 22:22:26
Perhaps there's another point related to the blue angle that we can use?
Perhaps there's another point related to the blue angle that we can use?
YaoAOPS
2021-03-12 22:23:11
CPO2
CPO2
MaryM
2021-03-12 22:23:15
How about point P?
How about point P?
MaryM
2021-03-12 22:23:22
We can do the same Law of Cosines calculations using triangles \triangle PO_1C and \triangle PO_2C that we just did for \triangle OO_1C and \triangle OO_2C.
We can do the same Law of Cosines calculations using triangles \triangle PO_1C and \triangle PO_2C that we just did for \triangle OO_1C and \triangle OO_2C.
MaryM
2021-03-12 22:23:54
We will get two equations and then we will be able to subtract them, just like we did before.
We will get two equations and then we will be able to subtract them, just like we did before.
MaryM
2021-03-12 22:23:58
And we'll get exactly the same end equation, except O will be replaced by P.
And we'll get exactly the same end equation, except O will be replaced by P.
MaryM
2021-03-12 22:24:07
That is, we'll get (PO_1)^2 - (PO_2)^2 = (O_1C)^2 - (O_2C)^2 + (PC)d.
That is, we'll get (PO_1)^2 - (PO_2)^2 = (O_1C)^2 - (O_2C)^2 + (PC)d.
MaryM
2021-03-12 22:24:11
Let's put these two equations side-by-side:
Let's put these two equations side-by-side:
MaryM
2021-03-12 22:24:16
\begin{align*} (OO_1)^2 - (OO_2)^2 &= (O_1C)^2 - (O_2C)^2 + (OC)d, \\ (PO_1)^2 - (PO_2)^2 &= (O_1C)^2 - (O_2C)^2 + (PC)d. \end{align*}
\begin{align*} (OO_1)^2 - (OO_2)^2 &= (O_1C)^2 - (O_2C)^2 + (OC)d, \\ (PO_1)^2 - (PO_2)^2 &= (O_1C)^2 - (O_2C)^2 + (PC)d. \end{align*}
MaryM
2021-03-12 22:24:21
Any ideas?
Any ideas?
rockyrockrock
2021-03-12 22:24:44
subtract?
subtract?
Z_Lu
2021-03-12 22:24:44
subtract
subtract
panda2018
2021-03-12 22:24:44
subtract
subtract
MaryM
2021-03-12 22:24:47
We can get rid of a lot of stuff we didn't know how to deal with if we subtract them!
We can get rid of a lot of stuff we didn't know how to deal with if we subtract them!
MaryM
2021-03-12 22:24:51
If we subtract them, the (O_1C)^2 - (O_2C)^2 terms will cancel.
If we subtract them, the (O_1C)^2 - (O_2C)^2 terms will cancel.
MaryM
2021-03-12 22:24:56
What's left is (OO_1)^2 - (OO_2)^2 - (PO_1)^2 + (PO_2)^2 = (OC - PC)d.
What's left is (OO_1)^2 - (OO_2)^2 - (PO_1)^2 + (PO_2)^2 = (OC - PC)d.
MaryM
2021-03-12 22:25:05
But what is OC - PC?
But what is OC - PC?
Zhaom
2021-03-12 22:25:18
r
r
MaryM
2021-03-12 22:25:20
It's OP. But that's the radius of the little circle \omega, which we called r.
It's OP. But that's the radius of the little circle \omega, which we called r.
MaryM
2021-03-12 22:25:25
So we've got (OO_1)^2 - (OO_2)^2 - (PO_1)^2 + (PO_2)^2 = rd.
So we've got (OO_1)^2 - (OO_2)^2 - (PO_1)^2 + (PO_2)^2 = rd.
MaryM
2021-03-12 22:25:39
What about those other terms?
What about those other terms?
MaryM
2021-03-12 22:25:43
Can we rewrite OO_1 as something nicer?
Can we rewrite OO_1 as something nicer?
rockyrockrock
2021-03-12 22:25:58
Substitute 961+r,625+r
Substitute 961+r,625+r
panda2018
2021-03-12 22:25:58
961+r
961+r
Zhaom
2021-03-12 22:25:58
961+r
961+r
MaryM
2021-03-12 22:26:00
Yes: as we discussed a while earlier, OO_1 is the sum of the radii of \omega_1 and \omega.
Yes: as we discussed a while earlier, OO_1 is the sum of the radii of \omega_1 and \omega.
MaryM
2021-03-12 22:26:03
So OO_1 = 961 + r.
So OO_1 = 961 + r.
MaryM
2021-03-12 22:26:06
Similarly OO_2 = 625 + r.
Similarly OO_2 = 625 + r.
MaryM
2021-03-12 22:26:11
So now we have (961+r)^2 - (625+r)^2 - (PO_1)^2 + (PO_2)^2 = rd.
So now we have (961+r)^2 - (625+r)^2 - (PO_1)^2 + (PO_2)^2 = rd.
MaryM
2021-03-12 22:26:19
How can we get at (PO_1)^2 and (PO_2)^2?
How can we get at (PO_1)^2 and (PO_2)^2?
Irving1004
2021-03-12 22:27:09
Pythagoras?
Pythagoras?
MaryM
2021-03-12 22:27:12
If we let R be the foot of the perpendicular from P to \overline{O_1 O_2}, then we can apply Pythagoras to right triangles PRO_1 and PRO_2.
If we let R be the foot of the perpendicular from P to \overline{O_1 O_2}, then we can apply Pythagoras to right triangles PRO_1 and PRO_2.
MaryM
2021-03-12 22:27:16

MaryM
2021-03-12 22:27:25
Pythagoras gives us that
\begin{align*} (PO_1)^2 - (PO_2)^2 &= (O_1 R)^2 + (PR)^2 - (O_2 R)^2 - (PR)^2 \\ &= (O_1 R)^2 - (O_2 R)^2. \end{align*}
Pythagoras gives us that
\begin{align*} (PO_1)^2 - (PO_2)^2 &= (O_1 R)^2 + (PR)^2 - (O_2 R)^2 - (PR)^2 \\ &= (O_1 R)^2 - (O_2 R)^2. \end{align*}
MaryM
2021-03-12 22:27:32
How can we deal with (O_1 R)^2 and (O_2 R)^2?
How can we deal with (O_1 R)^2 and (O_2 R)^2?
MaryM
2021-03-12 22:28:06
Can we use Pythagoras again?
Can we use Pythagoras again?
functionalmath
2021-03-12 22:28:57
yes
yes
MaryM
2021-03-12 22:29:00
We can apply Pythagoras to right triangles ARO_1 and ARO_2.
We can apply Pythagoras to right triangles ARO_1 and ARO_2.
MaryM
2021-03-12 22:29:31
Pythagoras gives us that (AO_1)^2 = (O_1 R)^2 + AR^2 and (AO_2)^2 = (O_2 R)^2 + AR^2. Subtracting these equations gives us
(O_1 R)^2 - (O_2 R)^2 = (AO_1)^2 - (AO_2)^2 = 961^2 - 256^2.
Pythagoras gives us that (AO_1)^2 = (O_1 R)^2 + AR^2 and (AO_2)^2 = (O_2 R)^2 + AR^2. Subtracting these equations gives us
(O_1 R)^2 - (O_2 R)^2 = (AO_1)^2 - (AO_2)^2 = 961^2 - 256^2.
Irving1004
2021-03-12 22:29:45
Ohh I see
Ohh I see
MaryM
2021-03-12 22:29:47
Therefore, (PO_1)^2 - (PO_2)^2 = 961^2 - 256^2, so the equation (961+r)^2 - (625+r)^2 - (PO_1)^2 + (PO_2)^2 = rd becomes
(961 + r)^2 - (625 + r)^2 - 961^2 + 256^2 = rd.
Therefore, (PO_1)^2 - (PO_2)^2 = 961^2 - 256^2, so the equation (961+r)^2 - (625+r)^2 - (PO_1)^2 + (PO_2)^2 = rd becomes
(961 + r)^2 - (625 + r)^2 - 961^2 + 256^2 = rd.
MaryM
2021-03-12 22:30:07
If you're familiar with the radical axis, then it gives us this result quickly: Since A and B are the intersection points of circles \omega_1 and \omega_2, line AB is the radical axis of the two circles. And since P lies on this radical axis, it has the same power with respect to both circles. In other words, (PO_1)^2 - 961^2 = (PO_2)^2 - 256^2, so (PO_1)^2 - (PO_2)^2 = 961^2 - 256^2.
If you're familiar with the radical axis, then it gives us this result quickly: Since A and B are the intersection points of circles \omega_1 and \omega_2, line AB is the radical axis of the two circles. And since P lies on this radical axis, it has the same power with respect to both circles. In other words, (PO_1)^2 - 961^2 = (PO_2)^2 - 256^2, so (PO_1)^2 - (PO_2)^2 = 961^2 - 256^2.
Lamboreghini
2021-03-12 22:30:29
not 256, 625
not 256, 625
MaryM
2021-03-12 22:30:33
true!
true!
MaryM
2021-03-12 22:30:51
If you're familiar with the radical axis, then it gives us this result quickly: Since A and B are the intersection points of circles \omega_1 and \omega_2, line AB is the radical axis of the two circles. And since P lies on this radical axis, it has the same power with respect to both circles. In other words, (PO_1)^2 - 961^2 = (PO_2)^2 - 625^2, so (PO_1)^2 - (PO_2)^2 = 961^2 - 625^2.
If you're familiar with the radical axis, then it gives us this result quickly: Since A and B are the intersection points of circles \omega_1 and \omega_2, line AB is the radical axis of the two circles. And since P lies on this radical axis, it has the same power with respect to both circles. In other words, (PO_1)^2 - 961^2 = (PO_2)^2 - 625^2, so (PO_1)^2 - (PO_2)^2 = 961^2 - 625^2.
MaryM
2021-03-12 22:31:05
But well, what happens when we expand the left side?
But well, what happens when we expand the left side?
Irving1004
2021-03-12 22:31:10
Cancellations occurr!
Cancellations occurr!
Irving1004
2021-03-12 22:31:10
we get 672d = rd and r = 672!
we get 672d = rd and r = 672!
MaryM
2021-03-12 22:31:19
We get (961^2 + 2r \cdot 961 + r^2) - (625^2 + 2r \cdot 625 + r^2) + 625^2 - 961^2 = rd.
We get (961^2 + 2r \cdot 961 + r^2) - (625^2 + 2r \cdot 625 + r^2) + 625^2 - 961^2 = rd.
MaryM
2021-03-12 22:31:26
The 961^2 and 625^2 terms cancel!
The 961^2 and 625^2 terms cancel!
MaryM
2021-03-12 22:31:31
And the r^2 terms cancel!
And the r^2 terms cancel!
MaryM
2021-03-12 22:31:35
We have 2r(961-625) = rd.
We have 2r(961-625) = rd.
MaryM
2021-03-12 22:31:40
And the r's cancel too!
And the r's cancel too!
MaryM
2021-03-12 22:31:48
And we're left with 2(961-625) = d.
And we're left with 2(961-625) = d.
functionalmath
2021-03-12 22:31:58
d=672
d=672
MaryM
2021-03-12 22:32:00
We get d = 2(961 - 625) = 2(336) = \boxed{672}.
We get d = 2(961 - 625) = 2(336) = \boxed{672}.
MaryM
2021-03-12 22:32:12
This question was pretty hard!
This question was pretty hard!
Irving1004
2021-03-12 22:32:34
Ah I think I somehow guessed that.
Ah I think I somehow guessed that.
happyhari
2021-03-12 22:32:43
agreeable
agreeable
MaryM
2021-03-12 22:32:46
Wait a second...remember that we had two possible configurations at the beginning.
Wait a second...remember that we had two possible configurations at the beginning.
MaryM
2021-03-12 22:32:51
What if we chose the wrong one? Maybe we got the wrong answer?
What if we chose the wrong one? Maybe we got the wrong answer?
Irving1004
2021-03-12 22:33:27
Doesn't matter
Doesn't matter
rockyrockrock
2021-03-12 22:34:04
will it give u the same answer
will it give u the same answer
MaryM
2021-03-12 22:34:09
I think we're OK. The key idea of our solution used Law of Cosines repeatedly on \triangle OO_1O_2 and the cevian \overline{OC} that passed through P.
I think we're OK. The key idea of our solution used Law of Cosines repeatedly on \triangle OO_1O_2 and the cevian \overline{OC} that passed through P.
MaryM
2021-03-12 22:34:14
The other key step of our solution used power of a point at point P, along the lines AB, PO_1, and PO_2.
The other key step of our solution used power of a point at point P, along the lines AB, PO_1, and PO_2.
MaryM
2021-03-12 22:34:34
In either configuration, we'll end up doing the exactly the same calculations. None of these depend on whether \overline{AB} intersects \overline{O_1O_2}.
In either configuration, we'll end up doing the exactly the same calculations. None of these depend on whether \overline{AB} intersects \overline{O_1O_2}.
MaryM
2021-03-12 22:34:40
So we never actually used the fact whether \overline{AB} interesects \overline{O_1O_2} or not.
So we never actually used the fact whether \overline{AB} interesects \overline{O_1O_2} or not.
MaryM
2021-03-12 22:34:45
Hence, our solution is good, regardless of which initial drawing we used.
Hence, our solution is good, regardless of which initial drawing we used.
MaryM
2021-03-12 22:35:04
(And in fact, we used the wrong one. If you draw this carefully to scale, you'll get that \overline{AB} misses \overline{O_1O_2}.)
(And in fact, we used the wrong one. If you draw this carefully to scale, you'll get that \overline{AB} misses \overline{O_1O_2}.)
functionalmath
2021-03-12 22:35:13
2 more
2 more
MaryM
2021-03-12 22:35:23
14. For any positive integer a,\,\sigma(a) denotes the sum of the positive integer divisors of a. Let n be the least positive integer such that \sigma(a^n)-1 is divisible by 2021 for all positive integers a. Find the sum of the prime factors in the prime factorization of n.
14. For any positive integer a,\,\sigma(a) denotes the sum of the positive integer divisors of a. Let n be the least positive integer such that \sigma(a^n)-1 is divisible by 2021 for all positive integers a. Find the sum of the prime factors in the prime factorization of n.
MaryM
2021-03-12 22:35:32
Let's start by experimenting.
Let's start by experimenting.
MaryM
2021-03-12 22:35:37
What's a good number to experiment on?
What's a good number to experiment on?
Zhaom
2021-03-12 22:35:55
2
2
Mathdreams
2021-03-12 22:35:55
2
2
MaryM
2021-03-12 22:35:58
a = 2 is probably the simplest. (a=1 doesn't really give us any information.)
a = 2 is probably the simplest. (a=1 doesn't really give us any information.)
MaryM
2021-03-12 22:36:06
So let's try to answer a simpler question: what's the smallest value of n such that \sigma(2^n)-1 is divisible by 2021?
So let's try to answer a simpler question: what's the smallest value of n such that \sigma(2^n)-1 is divisible by 2021?
MaryM
2021-03-12 22:36:15
What exactly is \sigma(2^n)?
What exactly is \sigma(2^n)?
Zhaom
2021-03-12 22:36:50
\sum^n_{k=0}2^n
\sum^n_{k=0}2^n
ancientwarrior
2021-03-12 22:36:50
2^(n+1)-1?
2^(n+1)-1?
happyhari
2021-03-12 22:36:55
the sum of all powers of 2 from 2 to 2^n
the sum of all powers of 2 from 2 to 2^n
MaryM
2021-03-12 22:36:58
It's the sum of the divisors of 2^n, by definition.
It's the sum of the divisors of 2^n, by definition.
MaryM
2021-03-12 22:37:03
They're 1, 2, 2^2, etc. up to 2^n.
They're 1, 2, 2^2, etc. up to 2^n.
MaryM
2021-03-12 22:37:24
Their sum 1 + 2 + 2^2 + \cdots + 2^n is a geometric series.
Their sum 1 + 2 + 2^2 + \cdots + 2^n is a geometric series.
MaryM
2021-03-12 22:37:30
Its sum is \dfrac{2^{n+1}-1}{2-1} = 2^{n+1} - 1.
Its sum is \dfrac{2^{n+1}-1}{2-1} = 2^{n+1} - 1.
MaryM
2021-03-12 22:37:42
So \sigma(2^n)-1 = 2^{n+1} - 2.
So \sigma(2^n)-1 = 2^{n+1} - 2.
MaryM
2021-03-12 22:37:55
And the question is: for what value of n is this a multiple of 2021?
And the question is: for what value of n is this a multiple of 2021?
MaryM
2021-03-12 22:38:17
We can divide it by 2, because the 2 is not going to help us be a multiple of 2021.
We can divide it by 2, because the 2 is not going to help us be a multiple of 2021.
MaryM
2021-03-12 22:38:25
So the equivalent question is: when is 2^n - 1 a multiple of 2021?
So the equivalent question is: when is 2^n - 1 a multiple of 2021?
MaryM
2021-03-12 22:38:35
If we wanted to write this using modular arithmetic, the question becomes: when is 2^n \equiv 1 \pmod{2021}?
If we wanted to write this using modular arithmetic, the question becomes: when is 2^n \equiv 1 \pmod{2021}?
Zhaom
2021-03-12 22:39:01
2021=43 \cdot 47
2021=43 \cdot 47
Irving1004
2021-03-12 22:39:01
When it is a multiple of 43 and 47
When it is a multiple of 43 and 47
MaryM
2021-03-12 22:39:05
We know that 2021 = 43 \cdot 47. (Hopefully you memorized this before taking the test.)
We know that 2021 = 43 \cdot 47. (Hopefully you memorized this before taking the test.)
MaryM
2021-03-12 22:39:11
The Chinese Remainder Theorem tells us that we can separately solve for each prime:
The Chinese Remainder Theorem tells us that we can separately solve for each prime:
MaryM
2021-03-12 22:39:15
\begin{align*} 2^n &\equiv 1 \pmod{43}, \\ 2^n &\equiv 1 \pmod{47}. \end{align*}
\begin{align*} 2^n &\equiv 1 \pmod{43}, \\ 2^n &\equiv 1 \pmod{47}. \end{align*}
MaryM
2021-03-12 22:39:20
And now what? How do we solve these?
And now what? How do we solve these?
JimY
2021-03-12 22:39:36
FLT
FLT
Zhaom
2021-03-12 22:39:36
Fermat's Little Theorem
Fermat's Little Theorem
MaryM
2021-03-12 22:39:38
Now we can use Fermat's Little Theorem!
Now we can use Fermat's Little Theorem!
MaryM
2021-03-12 22:39:44
FLT states that a^{p-1} \equiv 1 \pmod{p} so long as a \not\equiv 0 \pmod{p}.
FLT states that a^{p-1} \equiv 1 \pmod{p} so long as a \not\equiv 0 \pmod{p}.
MaryM
2021-03-12 22:39:50
When we apply FLT to our system of equations, what does it tell us?
When we apply FLT to our system of equations, what does it tell us?
jxwis2010
2021-03-12 22:40:31
n = 42p and n=46q
n = 42p and n=46q
Mathdreams
2021-03-12 22:40:31
n is a multiple of 42 and 46!
n is a multiple of 42 and 46!
Arrowhead575
2021-03-12 22:40:31
Lcm Of 42 and 46
Lcm Of 42 and 46
Zhaom
2021-03-12 22:40:31
n \equiv 0\pmod{42} and n \equiv 0\pmod{46}
n \equiv 0\pmod{42} and n \equiv 0\pmod{46}
MaryM
2021-03-12 22:40:38
It tells us that n = 43-1 = 42 is a solution to the first equation, and n = 47-1 = 46 is a solution to the second equation.
It tells us that n = 43-1 = 42 is a solution to the first equation, and n = 47-1 = 46 is a solution to the second equation.
MaryM
2021-03-12 22:40:45
So, we can take n = \text{lcm}[42,46] as a common solution.
So, we can take n = \text{lcm}[42,46] as a common solution.
MaryM
2021-03-12 22:40:53
Given that 42 = 2 \cdot 3 \cdot 7 and 46 = 2 \cdot 23 are prime factorizations, what do we get for a common n?
Given that 42 = 2 \cdot 3 \cdot 7 and 46 = 2 \cdot 23 are prime factorizations, what do we get for a common n?
jxwis2010
2021-03-12 22:41:23
2*3*7*23
2*3*7*23
MaryM
2021-03-12 22:41:26
We get n = 2 \cdot 3 \cdot 7 \cdot 23. (It makes sense to write it this way -- we don't really care what n actually is, we just care about its prime factors. Remember what the question is asking!)
We get n = 2 \cdot 3 \cdot 7 \cdot 23. (It makes sense to write it this way -- we don't really care what n actually is, we just care about its prime factors. Remember what the question is asking!)
MaryM
2021-03-12 22:41:33
Anybody see a problem with all of this?
Anybody see a problem with all of this?
peace09
2021-03-12 22:42:07
...We only considered a=2...
...We only considered a=2...
mathgeek23
2021-03-12 22:42:07
this only works for a=2
this only works for a=2
Mathdreams
2021-03-12 22:42:10
We need to consider higher cases.
We need to consider higher cases.
MaryM
2021-03-12 22:42:12
We've only deal with one value of a.
We've only deal with one value of a.
jxwis2010
2021-03-12 22:42:15
this may not be the smallest n
this may not be the smallest n
MaryM
2021-03-12 22:42:22
Also, FLT doesn't guarantee us that this is the smallest such n. It only guarantees that this n will work. There might be a smaller n that also works.
Also, FLT doesn't guarantee us that this is the smallest such n. It only guarantees that this n will work. There might be a smaller n that also works.
MaryM
2021-03-12 22:42:30
Let's set this annoyance aside for now.
Let's set this annoyance aside for now.
MaryM
2021-03-12 22:42:34
What should we try next?
What should we try next?
MaryM
2021-03-12 22:42:54
Hint: pick a more general a this time.
Hint: pick a more general a this time.
Zhaom
2021-03-12 22:43:20
a is a prime number.
a is a prime number.
MaryM
2021-03-12 22:43:23
Let's be bold and try it for a general prime p.
Let's be bold and try it for a general prime p.
MaryM
2021-03-12 22:44:14
What is \sigma(p^n)?
What is \sigma(p^n)?
rockyrockrock
2021-03-12 22:45:01
1+p^1+p^2+...+p^n
1+p^1+p^2+...+p^n
MaryM
2021-03-12 22:45:05
The divisors of p^n are 1, p, p^2, etc. up to p^n.
The divisors of p^n are 1, p, p^2, etc. up to p^n.
MaryM
2021-03-12 22:45:08
So their sum is \sigma(p^n) = 1 + p + p^2 + \cdots + p^n.
So their sum is \sigma(p^n) = 1 + p + p^2 + \cdots + p^n.
MaryM
2021-03-12 22:45:12
And what does this work out to?
And what does this work out to?
CircleInvert
2021-03-12 22:45:35
\frac{p^{n+1}-1}{p-1}
\frac{p^{n+1}-1}{p-1}
Lamboreghini
2021-03-12 22:45:35
1+p+p^2+...+p^n=(p^(n+1)-1)/(p-1)
1+p+p^2+...+p^n=(p^(n+1)-1)/(p-1)
Zhaom
2021-03-12 22:45:35
\frac{p^{n+1}-1}{p-1}
\frac{p^{n+1}-1}{p-1}
MaryM
2021-03-12 22:45:38
This is a geometric series, so we get \sigma(p^n) = \dfrac{p^{n+1}-1}{p-1}.
This is a geometric series, so we get \sigma(p^n) = \dfrac{p^{n+1}-1}{p-1}.
MaryM
2021-03-12 22:45:59
So we must have \dfrac{p^{n+1}-1}{p-1} - 1 be a multiple of 2021.
So we must have \dfrac{p^{n+1}-1}{p-1} - 1 be a multiple of 2021.
Irving1004
2021-03-12 22:46:15
Yeah
Yeah
MaryM
2021-03-12 22:46:17
This simplifies to \dfrac{p^{n+1}-p}{p-1} being a multiple of 2021.
This simplifies to \dfrac{p^{n+1}-p}{p-1} being a multiple of 2021.
MaryM
2021-03-12 22:46:27
And factoring out a p, we need \dfrac{p(p^n-1)}{p-1} to be a multiple of 2021.
And factoring out a p, we need \dfrac{p(p^n-1)}{p-1} to be a multiple of 2021.
MaryM
2021-03-12 22:46:33
Is this the same as p(p^n-1) being a multiple of 2021?
Is this the same as p(p^n-1) being a multiple of 2021?
Ferocious_Bunny
2021-03-12 22:47:06
Not necessarily
Not necessarily
jj_ca888
2021-03-12 22:47:06
unless p-1 has some factors
unless p-1 has some factors
Zhaom
2021-03-12 22:47:06
no, what if p-1 is a multiple of 2021
no, what if p-1 is a multiple of 2021
CircleInvert
2021-03-12 22:47:06
Only if p-1 is relatively prime to 2021
Only if p-1 is relatively prime to 2021
JimY
2021-03-12 22:47:10
Almost, only if p-1 isn't a multiple of 2021
Almost, only if p-1 isn't a multiple of 2021
MaryM
2021-03-12 22:47:13
Not necessarily. If p-1 is a multiple of 43 or 47, we might have a problem.
Not necessarily. If p-1 is a multiple of 43 or 47, we might have a problem.
MaryM
2021-03-12 22:47:21
Let's put that aside for a moment, and assume (for now) that p-1 is not a multiple of 43 or 47.
Let's put that aside for a moment, and assume (for now) that p-1 is not a multiple of 43 or 47.
MaryM
2021-03-12 22:47:26
In that case, dividing by p-1 doesn't affect whether the number is a multiple of 2021 or not.
In that case, dividing by p-1 doesn't affect whether the number is a multiple of 2021 or not.
MaryM
2021-03-12 22:47:38
So we just need to determine when p(p^n-1) is a multiple of 2021.
So we just need to determine when p(p^n-1) is a multiple of 2021.
MaryM
2021-03-12 22:47:45
How do we go about it?
How do we go about it?
MaryM
2021-03-12 22:48:39
Can we get rid of the factor p?
Can we get rid of the factor p?
ancientwarrior
2021-03-12 22:49:34
it might be 43 or 47?
it might be 43 or 47?
Irving1004
2021-03-12 22:49:34
So we need p to not be 43 or 47
So we need p to not be 43 or 47
Irving1004
2021-03-12 22:49:34
And we are set
And we are set
MaryM
2021-03-12 22:49:41
If p = 43 or p = 47, we get one of the factors of 2021 for free!
If p = 43 or p = 47, we get one of the factors of 2021 for free!
MaryM
2021-03-12 22:50:20
So, let's suppose that p is not 43, not 47. In that case, we can ignore the factor p, since it won't affect divisibility by 2021.
So, let's suppose that p is not 43, not 47. In that case, we can ignore the factor p, since it won't affect divisibility by 2021.
palaashgang
2021-03-12 22:50:27
Yeah
Yeah
MaryM
2021-03-12 22:50:36
Then, it's just like the p=2 case: we need to solve the system:
Then, it's just like the p=2 case: we need to solve the system:
MaryM
2021-03-12 22:50:42
\begin{align*} p^n &\equiv 1 \pmod{43}, \\ p^n &\equiv 1 \pmod{47}. \end{align*}
\begin{align*} p^n &\equiv 1 \pmod{43}, \\ p^n &\equiv 1 \pmod{47}. \end{align*}
MaryM
2021-03-12 22:51:01
What solution do we get if we use the same method we used for p=2?
What solution do we get if we use the same method we used for p=2?
Zhaom
2021-03-12 22:52:09
n \equiv 0\pmod{42},n \equiv 0\pmod{46}
n \equiv 0\pmod{42},n \equiv 0\pmod{46}
YaoAOPS
2021-03-12 22:52:09
same
same
ancientwarrior
2021-03-12 22:52:09
the same thing?
the same thing?
jxwis2010
2021-03-12 22:52:09
Lcm(42,46)
Lcm(42,46)
MaryM
2021-03-12 22:52:12
As before, we want n to be a multiple of 43 and 47.
As before, we want n to be a multiple of 43 and 47.
MaryM
2021-03-12 22:52:18
So we'll definitely need n = 2 \cdot 3 \cdot 7 \cdot 23 (or any multiple of this n).
So we'll definitely need n = 2 \cdot 3 \cdot 7 \cdot 23 (or any multiple of this n).
MaryM
2021-03-12 22:52:23
Does this work for p=43 and p=47 too?
Does this work for p=43 and p=47 too?
functionalmath
2021-03-12 22:52:50
yes
yes
babytiger2010
2021-03-12 22:52:50
yes???
yes???
MaryM
2021-03-12 22:52:53
Yes. We get one factor (p) for free, and the other factor will be a factor of p^n-1 using the n that we just found.
Yes. We get one factor (p) for free, and the other factor will be a factor of p^n-1 using the n that we just found.
MaryM
2021-03-12 22:53:01
Are we done? Is this our n?
Are we done? Is this our n?
functionalmath
2021-03-12 22:53:21
no
no
Zhaom
2021-03-12 22:53:21
no, we still need to consider p-1
no, we still need to consider p-1
MaryM
2021-03-12 22:53:23
Not quite. We had to set a case aside, remember?
Not quite. We had to set a case aside, remember?
MaryM
2021-03-12 22:53:27
We needed \dfrac{p(p^n-1)}{p-1} to be a multiple of 2021.
We needed \dfrac{p(p^n-1)}{p-1} to be a multiple of 2021.
MaryM
2021-03-12 22:53:33
We were only allowed to ignore the p-1 denominator if it wasn't a multiple of 43 or 47.
We were only allowed to ignore the p-1 denominator if it wasn't a multiple of 43 or 47.
MaryM
2021-03-12 22:53:38
So we still have to deal with the case when p-1 is a multiple of 43 or 47.
So we still have to deal with the case when p-1 is a multiple of 43 or 47.
MaryM
2021-03-12 22:53:50
Let's try these cases one at a time.
Let's try these cases one at a time.
MaryM
2021-03-12 22:53:54
Suppose p-1 is a multiple of 43.
Suppose p-1 is a multiple of 43.
MaryM
2021-03-12 22:54:08
Then what do we need?
Then what do we need?
MaryM
2021-03-12 22:54:47
How many factors of 43 must divide p^n-1?
How many factors of 43 must divide p^n-1?
rockyrockrock
2021-03-12 22:55:01
2 powers of 43?
2 powers of 43?
happyhari
2021-03-12 22:55:01
then we need another 43 multiplied in the numerator
then we need another 43 multiplied in the numerator
ancientwarrior
2021-03-12 22:55:01
2
2
jxwis2010
2021-03-12 22:55:01
2
2
MaryM
2021-03-12 22:55:05
At a minimum, we need p^n-1 to be a multiple of 43^2, because the denominator will cancel out a factor of 43.
At a minimum, we need p^n-1 to be a multiple of 43^2, because the denominator will cancel out a factor of 43.
MaryM
2021-03-12 22:55:18
More generally, if p-1 is a multiple of 43^k, then we need p^n-1 to be a multiple of 43^{k+1} in order to preserve a power of 43 in the numerator after we cancel out a factor of 43^k.
More generally, if p-1 is a multiple of 43^k, then we need p^n-1 to be a multiple of 43^{k+1} in order to preserve a power of 43 in the numerator after we cancel out a factor of 43^k.
MaryM
2021-03-12 22:55:39
Suppose k is the maximum power of 43^k in p-1.
Suppose k is the maximum power of 43^k in p-1.
MaryM
2021-03-12 22:55:45
How we we write this statement using modular arithmetic?
How we we write this statement using modular arithmetic?
MaryM
2021-03-12 22:56:28
Hint: write a congruence \mod 43^{k+1}.
Hint: write a congruence \mod 43^{k+1}.
MaryM
2021-03-12 22:58:16
Further hint: we know that p-1=m43^k for some m that is not a multiple of 43. Try to write this as a congruence \pmod 43^{k+1}.
Further hint: we know that p-1=m43^k for some m that is not a multiple of 43. Try to write this as a congruence \pmod 43^{k+1}.
Zhaom
2021-03-12 22:58:49
typo?
typo?
MaryM
2021-03-12 22:59:01
Yes, typo.
Yes, typo.
MaryM
2021-03-12 22:59:06
I meant:
I meant:
MaryM
2021-03-12 22:59:17
Further hint: we know that p-1=m43^k for some m that is not a multiple of 43. Try to write this as a congruence \pmod {43^{k+1}}.
Further hint: we know that p-1=m43^k for some m that is not a multiple of 43. Try to write this as a congruence \pmod {43^{k+1}}.
Zhaom
2021-03-12 23:01:27
p-1 \equiv m \cdot 43^k\pmod{43^{k+1}} where m \cancel{\equiv} 0\pmod{43}
p-1 \equiv m \cdot 43^k\pmod{43^{k+1}} where m \cancel{\equiv} 0\pmod{43}
MaryM
2021-03-12 23:01:35
We can write p \equiv (1 + m43^k) \pmod{43^{k+1}} for some 0 \le m < 43.
We can write p \equiv (1 + m43^k) \pmod{43^{k+1}} for some 0 \le m < 43.
MaryM
2021-03-12 23:01:56
Let's raise it to n to get p^n:
Let's raise it to n to get p^n:
MaryM
2021-03-12 23:02:01
We get p^n \equiv (1 + m43^k)^n \pmod{43^{k+1}}.
We get p^n \equiv (1 + m43^k)^n \pmod{43^{k+1}}.
MaryM
2021-03-12 23:02:07
And we want this to be equivalent to 1 \pmod{43^{k+1}}.
And we want this to be equivalent to 1 \pmod{43^{k+1}}.
MaryM
2021-03-12 23:02:46
What do you notice if you expand (1+m43^k)^n using the Binomial Theorem?
What do you notice if you expand (1+m43^k)^n using the Binomial Theorem?
CircleInvert
2021-03-12 23:03:15
Using the Binomial Theorem, this is gives p^n\equiv 1+nm43%k \pmod{43^{k+1}} (all other terms are 0)
Using the Binomial Theorem, this is gives p^n\equiv 1+nm43%k \pmod{43^{k+1}} (all other terms are 0)
peace09
2021-03-12 23:03:15
All terms except for 1 and mn43^k are a multiple of 43^{k+1}
All terms except for 1 and mn43^k are a multiple of 43^{k+1}
MaryM
2021-03-12 23:03:18
By the Binomial Theorem, we get p^n \equiv (1 + nm43^k + \binom{n}{2}m^243^{2k} + \cdots + m^n43^{nk}) \pmod{43^{k+1}}.
By the Binomial Theorem, we get p^n \equiv (1 + nm43^k + \binom{n}{2}m^243^{2k} + \cdots + m^n43^{nk}) \pmod{43^{k+1}}.
MaryM
2021-03-12 23:03:22
All of the higher powers of 43^k are 0 \pmod{43^{k+1}}.
All of the higher powers of 43^k are 0 \pmod{43^{k+1}}.
MaryM
2021-03-12 23:03:26
So we're left with p^n \equiv (1 + nm43^k) \pmod{43^{k+1}}.
So we're left with p^n \equiv (1 + nm43^k) \pmod{43^{k+1}}.
MaryM
2021-03-12 23:03:32
How can we guarantee that the second term in the parentheses is 0 \pmod{43^{k+1}}?
How can we guarantee that the second term in the parentheses is 0 \pmod{43^{k+1}}?
jxwis2010
2021-03-12 23:04:11
n=43u
n=43u
JimY
2021-03-12 23:04:11
nm is a multiple of 43
nm is a multiple of 43
happyhari
2021-03-12 23:04:11
not if nm is not divisible by 43
not if nm is not divisible by 43
Irving1004
2021-03-12 23:04:11
If n is congruent to 0(mod 43).
If n is congruent to 0(mod 43).
MaryM
2021-03-12 23:04:13
We need to take n = 43. (Or any multiple of 43.) Then that term will be a multiple of 43^{k+1} and go away.
We need to take n = 43. (Or any multiple of 43.) Then that term will be a multiple of 43^{k+1} and go away.
MaryM
2021-03-12 23:04:19
So the conclusion is: if p-1 is a multiple of 43, we need to have n be a multiple of 43 in order to guarantee that \sigma(p^n)-1 is a multiple of 43.
So the conclusion is: if p-1 is a multiple of 43, we need to have n be a multiple of 43 in order to guarantee that \sigma(p^n)-1 is a multiple of 43.
MaryM
2021-03-12 23:04:37
What happens in the case where p-1 is a multiple of 47?
What happens in the case where p-1 is a multiple of 47?
jxwis2010
2021-03-12 23:04:57
n=47v
n=47v
happyhari
2021-03-12 23:04:57
same thing, but with 47
same thing, but with 47
Irving1004
2021-03-12 23:04:57
It needs to be a multiple of 47
It needs to be a multiple of 47
MaryM
2021-03-12 23:05:00
We can do exactly the same argument for 47 instead of 43.
We can do exactly the same argument for 47 instead of 43.
MaryM
2021-03-12 23:05:03
And we get the same conclusion: if p-1 is a multiple of 47, we need to have n be a multiple of 47 in order to guarantee that \sigma(p^n)-1 is a multiple of 47.
And we get the same conclusion: if p-1 is a multiple of 47, we need to have n be a multiple of 47 in order to guarantee that \sigma(p^n)-1 is a multiple of 47.
MaryM
2021-03-12 23:05:17
So, we've shown that in order to ensure that \sigma(p^n) - 1 is a multiple of 2021 for all primes p, we must have that n is a multiple of 2 \cdot 3 \cdot 7 \cdot 23 \cdot 43 \cdot 47.
So, we've shown that in order to ensure that \sigma(p^n) - 1 is a multiple of 2021 for all primes p, we must have that n is a multiple of 2 \cdot 3 \cdot 7 \cdot 23 \cdot 43 \cdot 47.
MaryM
2021-03-12 23:05:25
That takes care of the primes. But what if a is composite?
That takes care of the primes. But what if a is composite?
jxwis2010
2021-03-12 23:06:37
Decomposes into primes
Decomposes into primes
peace09
2021-03-12 23:06:37
Somehow use what we know about primes
Somehow use what we know about primes
peace09
2021-03-12 23:06:37
Prime factorization of a?
Prime factorization of a?
Irving1004
2021-03-12 23:06:37
Same thing
Same thing
Zhaom
2021-03-12 23:06:37
then, the sum of the divisors of a^n is the product of the sum of the divisors of p_k^n some primes p_k so it is still 1\pmod{2021}
then, the sum of the divisors of a^n is the product of the sum of the divisors of p_k^n some primes p_k so it is still 1\pmod{2021}
CircleInvert
2021-03-12 23:06:37
Use the fact that \sigma is multiplicative
Use the fact that \sigma is multiplicative
MaryM
2021-03-12 23:07:15
If we let a=p_1^{\alpha_1}p_2^{\alpha_2}\cdots p_k^{\alpha_k}, then
\begin{align*}\sigma(a^n)&=\sigma(p_1^{n\alpha_1})\sigma(p_2^{n\alpha_2})\cdots \sigma(p_k^{n\alpha_k}) \end{align*}
If we let a=p_1^{\alpha_1}p_2^{\alpha_2}\cdots p_k^{\alpha_k}, then
\begin{align*}\sigma(a^n)&=\sigma(p_1^{n\alpha_1})\sigma(p_2^{n\alpha_2})\cdots \sigma(p_k^{n\alpha_k}) \end{align*}
YaoAOPS
2021-03-12 23:08:05
why
why
MaryM
2021-03-12 23:08:07
The product on the right hand side spams all divisors of a^n.
The product on the right hand side spams all divisors of a^n.
MaryM
2021-03-12 23:08:34
For example \sigma(p^b)\sigma(q^c) = (1+p+\cdots+p^b)(1+q+\cdots+q^c).
For example \sigma(p^b)\sigma(q^c) = (1+p+\cdots+p^b)(1+q+\cdots+q^c).
MaryM
2021-03-12 23:08:53
If we expand the right hand side, we get all the terms of the form p^iq^j with 0 \le i \le b and 0 \le j \le c.
If we expand the right hand side, we get all the terms of the form p^iq^j with 0 \le i \le b and 0 \le j \le c.
MaryM
2021-03-12 23:09:01
But those are all the divisors of p^bq^c!
But those are all the divisors of p^bq^c!
YaoAOPS
2021-03-12 23:09:13
oh
oh
MaryM
2021-03-12 23:09:16
The same thing happens if we add more primes.
The same thing happens if we add more primes.
MaryM
2021-03-12 23:09:39
So,
\begin{align*}\sigma(a^n)&=\sigma(p_1^{n\alpha_1})\sigma(p_2^{n\alpha_2})\cdots \sigma(p_k^{n\alpha_k})\\ &\equiv 1\cdot 1\cdots 1\pmod{2021}. \end{align*} This means that it's enough to just handle the prime powers!
So,
\begin{align*}\sigma(a^n)&=\sigma(p_1^{n\alpha_1})\sigma(p_2^{n\alpha_2})\cdots \sigma(p_k^{n\alpha_k})\\ &\equiv 1\cdot 1\cdots 1\pmod{2021}. \end{align*} This means that it's enough to just handle the prime powers!
MaryM
2021-03-12 23:09:55
I think we're done!
I think we're done!
happyhari
2021-03-12 23:10:10
yay!
yay!
Zhaom
2021-03-12 23:10:12
but we didn't get the answer yet
but we didn't get the answer yet
MaryM
2021-03-12 23:10:18
And our final answer is the sum of these primes.
And our final answer is the sum of these primes.
MaryM
2021-03-12 23:10:22
That's 2+3+7+23+43+47 = \boxed{125}.
That's 2+3+7+23+43+47 = \boxed{125}.
MaryM
2021-03-12 23:10:39
Last problem!
Last problem!
Irving1004
2021-03-12 23:10:48
shoot
shoot
happyhari
2021-03-12 23:10:48
yay!!
yay!!
MaryM
2021-03-12 23:10:51
15. Let S be the set of positive integers k such that the two parabolas y = x^2-k \qquad \text{and} \qquad x = 2(y-20)^2 - k intersect in four distinct points, and these four points lie on a circle with radius at most 21. Find the sum of the least element of S and the greatest element of S.
15. Let S be the set of positive integers k such that the two parabolas y = x^2-k \qquad \text{and} \qquad x = 2(y-20)^2 - k intersect in four distinct points, and these four points lie on a circle with radius at most 21. Find the sum of the least element of S and the greatest element of S.
MaryM
2021-03-12 23:11:17
What does the graph of y=x^2-k look like?
What does the graph of y=x^2-k look like?
Lamboreghini
2021-03-12 23:11:38
parabola, upward facing
parabola, upward facing
functionalmath
2021-03-12 23:11:38
parabola
parabola
Zhaom
2021-03-12 23:11:38
a parabola with vertex (0,-k)
a parabola with vertex (0,-k)
JimY
2021-03-12 23:11:38
parabola with vertex at 0, -k
parabola with vertex at 0, -k
CircleInvert
2021-03-12 23:11:38
y=x^2 translated k down
y=x^2 translated k down
MaryM
2021-03-12 23:11:42
It's an upwards-opening parabola with vertex at (0,-k) that's symmetric across the y-axis.
It's an upwards-opening parabola with vertex at (0,-k) that's symmetric across the y-axis.
MaryM
2021-03-12 23:11:46

MaryM
2021-03-12 23:11:50
How about the other parabola?
How about the other parabola?
Lamboreghini
2021-03-12 23:12:12
sideways facing
sideways facing
jxwis2010
2021-03-12 23:12:12
Horizontal
Horizontal
happyhari
2021-03-12 23:12:12
a sideways parabola
a sideways parabola
rockyrockrock
2021-03-12 23:12:16
opens up to the side
opens up to the side
MaryM
2021-03-12 23:12:18
It's a right-opening parabola.
It's a right-opening parabola.
MaryM
2021-03-12 23:12:23
Where is its vertex?
Where is its vertex?
Irving1004
2021-03-12 23:12:49
horizontal with vertex (-k, 20)
horizontal with vertex (-k, 20)
Rubikscube3.1415
2021-03-12 23:12:49
(-k, 20)
(-k, 20)
ancientwarrior
2021-03-12 23:12:49
(-k, 20)
(-k, 20)
rockyrockrock
2021-03-12 23:12:52
-k, 20
-k, 20
JimY
2021-03-12 23:12:55
(-k, 20)
(-k, 20)
sugar_rush
2021-03-12 23:12:55
(-k, 20)
(-k, 20)
MaryM
2021-03-12 23:12:57
The minimum value of x occurs when y-20 = 0, or y = 20.
The minimum value of x occurs when y-20 = 0, or y = 20.
MaryM
2021-03-12 23:13:01
And at that point, x=-k.
And at that point, x=-k.
MaryM
2021-03-12 23:13:04
So the vertex is at (-k,20).
So the vertex is at (-k,20).
MaryM
2021-03-12 23:13:10

MaryM
2021-03-12 23:13:15
The example I've drawn shows 4 intersection points.
The example I've drawn shows 4 intersection points.
MaryM
2021-03-12 23:13:20
But does that always happen?
But does that always happen?
Zhaom
2021-03-12 23:13:44
no
no
Lamboreghini
2021-03-12 23:13:44
no
no
MaryM
2021-03-12 23:13:48
The vertex might be to the right of the left blue branch, like so:
The vertex might be to the right of the left blue branch, like so:
MaryM
2021-03-12 23:13:54

MaryM
2021-03-12 23:14:08
When does this happen?
When does this happen?
Lamboreghini
2021-03-12 23:15:17
when k is too small
when k is too small
MaryM
2021-03-12 23:15:19
This happens when -k is larger than the value in the right parabola that makes y=20.
This happens when -k is larger than the value in the right parabola that makes y=20.
MaryM
2021-03-12 23:15:24
What value of the left branch of the blue curve gives a y value of 20?
What value of the left branch of the blue curve gives a y value of 20?
ancientwarrior
2021-03-12 23:15:54
-\sqrt{20+k}
-\sqrt{20+k}
MaryM
2021-03-12 23:15:57
We must have 20 = x^2 - k.
We must have 20 = x^2 - k.
MaryM
2021-03-12 23:16:03
So x^2 = 20+k, and x = -\sqrt{20+k}.
So x^2 = 20+k, and x = -\sqrt{20+k}.
MaryM
2021-03-12 23:16:08
And what do we conclude?
And what do we conclude?
Zhaom
2021-03-12 23:16:40
when \sqrt{k+20}>k
when \sqrt{k+20}>k
CircleInvert
2021-03-12 23:16:40
When k^2-k<20
When k^2-k<20
ancientwarrior
2021-03-12 23:16:40
sqrt(20+k)>k
sqrt(20+k)>k
ancientwarrior
2021-03-12 23:16:40
k<sqrt(20+k)
k<sqrt(20+k)
MaryM
2021-03-12 23:16:44
The red parabola's vertex is too far to the right if -\sqrt{20+k} < -k.
The red parabola's vertex is too far to the right if -\sqrt{20+k} < -k.
MaryM
2021-03-12 23:16:47
That is, \sqrt{20+k} > k.
That is, \sqrt{20+k} > k.
MaryM
2021-03-12 23:16:58
we can square it (since both sides are positive) to get 20 + k > k^2.
we can square it (since both sides are positive) to get 20 + k > k^2.
MaryM
2021-03-12 23:17:05
This is 0 > k^2 - k - 20.
This is 0 > k^2 - k - 20.
MaryM
2021-03-12 23:17:11
And this factors as 0 > (k-5)(k+4).
And this factors as 0 > (k-5)(k+4).
MaryM
2021-03-12 23:17:20
So, what bound do we get for k?
So, what bound do we get for k?
sugar_rush
2021-03-12 23:17:39
-4\leq k\leq 5
-4\leq k\leq 5
ancientwarrior
2021-03-12 23:17:39
so any k in (-4, 5) is not going to intersect at 4 points
so any k in (-4, 5) is not going to intersect at 4 points
rockyrockrock
2021-03-12 23:17:39
-4<k<5?
-4<k<5?
MaryM
2021-03-12 23:17:45
So since k is positive, we must have k < 5.
So since k is positive, we must have k < 5.
MaryM
2021-03-12 23:17:56
When k < 5, we'll definitely not get 4 intersection points.
When k < 5, we'll definitely not get 4 intersection points.
MaryM
2021-03-12 23:18:11
When k>5, the red parabola's vertex is to the left of the blue parabola, so we'll get 4 intersection points, as in the first picture...
When k>5, the red parabola's vertex is to the left of the blue parabola, so we'll get 4 intersection points, as in the first picture...
MaryM
2021-03-12 23:18:50
...so long as the red parabola stays above (0,-k) at x=0. (That is, we don't want the red parabola to pass under (0,-k), since then we'd only get 2 points from the top red branch.)
...so long as the red parabola stays above (0,-k) at x=0. (That is, we don't want the red parabola to pass under (0,-k), since then we'd only get 2 points from the top red branch.)
MaryM
2021-03-12 23:18:59
Let's check this. What inequality do we need?
Let's check this. What inequality do we need?
Zhaom
2021-03-12 23:19:45
an upper bound for k
an upper bound for k
MaryM
2021-03-12 23:19:49
When x=0, we need the smaller y value of the red parabola to be greater than -k.
When x=0, we need the smaller y value of the red parabola to be greater than -k.
MaryM
2021-03-12 23:19:58
So we have to solve for y in the second equation when x=0.
So we have to solve for y in the second equation when x=0.
MaryM
2021-03-12 23:20:02
What do we get?
What do we get?
Zhaom
2021-03-12 23:21:05
y=-sqrt(k/2)+20
y=-sqrt(k/2)+20
ancientwarrior
2021-03-12 23:21:05
y=\sqrt{\frac{k}{2}}+20? and that has to be greater than -k
y=\sqrt{\frac{k}{2}}+20? and that has to be greater than -k
Rubikscube3.1415
2021-03-12 23:21:05
-\sqrt{\frac{k}{2}}+20<-k, or \sqrt{\frac{k}{2}}+20>k
-\sqrt{\frac{k}{2}}+20<-k, or \sqrt{\frac{k}{2}}+20>k
MaryM
2021-03-12 23:21:08
We get (y-20)^2 = \dfrac{k}{2}.
We get (y-20)^2 = \dfrac{k}{2}.
MaryM
2021-03-12 23:21:12
So y = \pm\sqrt{\dfrac{k}{2}} + 20.
So y = \pm\sqrt{\dfrac{k}{2}} + 20.
MaryM
2021-03-12 23:21:17
Since we want the smaller value, we take the minus sign.
Since we want the smaller value, we take the minus sign.
MaryM
2021-03-12 23:21:21
Therefore, the inequality we need to ensure 4 points is -\sqrt{\dfrac{k}{2}} + 20 > -k.
Therefore, the inequality we need to ensure 4 points is -\sqrt{\dfrac{k}{2}} + 20 > -k.
functionalmath
2021-03-12 23:21:35
solve
solve
MaryM
2021-03-12 23:21:37
We can rearrange this as -\sqrt{\dfrac{k}{2}} > -(k+20).
We can rearrange this as -\sqrt{\dfrac{k}{2}} > -(k+20).
MaryM
2021-03-12 23:21:43
And then multiplying by -1 gives \sqrt{\dfrac{k}{2}} < k+20.
And then multiplying by -1 gives \sqrt{\dfrac{k}{2}} < k+20.
MaryM
2021-03-12 23:21:48
And then squaring gives \dfrac{k}{2} < (k+20)^2.
And then squaring gives \dfrac{k}{2} < (k+20)^2.
MaryM
2021-03-12 23:21:55
This simplifies to 0 < 2k^2 + 79k + 800.
This simplifies to 0 < 2k^2 + 79k + 800.
MaryM
2021-03-12 23:21:58
For what values of k does this hold?
For what values of k does this hold?
MaryM
2021-03-12 23:22:47
Hint: We are already assuming that k>5, so in particular k is positive.
Hint: We are already assuming that k>5, so in particular k is positive.
Rubikscube3.1415
2021-03-12 23:22:54
all k I think
all k I think
Zhaom
2021-03-12 23:22:54
every value
every value
functionalmath
2021-03-12 23:22:54
always true
always true
MaryM
2021-03-12 23:23:00
This holds for all positive k, since every term on the right side is positive!
This holds for all positive k, since every term on the right side is positive!
MaryM
2021-03-12 23:23:05
So it'll hold for all the k's we care about.
So it'll hold for all the k's we care about.
MaryM
2021-03-12 23:23:08
Thus, when k > 5 we definitely get 4 intersection points.
Thus, when k > 5 we definitely get 4 intersection points.
MaryM
2021-03-12 23:23:12
What value(s) of k are we missing?
What value(s) of k are we missing?
MaryM
2021-03-12 23:23:54
We showed that k<5 don't work. But does k=5 work?
We showed that k<5 don't work. But does k=5 work?
palaashgang
2021-03-12 23:24:40
no
no
Zhaom
2021-03-12 23:24:40
maybe
maybe
happyhari
2021-03-12 23:24:40
yes
yes
Rubikscube3.1415
2021-03-12 23:24:40
no, we get exactly 3 intersection points
no, we get exactly 3 intersection points
sugar_rush
2021-03-12 23:24:40
yes
yes
MaryM
2021-03-12 23:24:44
Let's see!
Let's see!
MaryM
2021-03-12 23:24:51
In this case the vertex of the red parabola lies on the blue parabola:
In this case the vertex of the red parabola lies on the blue parabola:
MaryM
2021-03-12 23:24:55

MaryM
2021-03-12 23:24:58
This looks like 3 intersection points, but is it really?
This looks like 3 intersection points, but is it really?
MaryM
2021-03-12 23:25:08
Since I have a fancy computer graphics package, I could zoom in.
Since I have a fancy computer graphics package, I could zoom in.
MaryM
2021-03-12 23:25:12
But you didn't have a fancy computer graphics package available to you when you took the test.
But you didn't have a fancy computer graphics package available to you when you took the test.
MaryM
2021-03-12 23:25:16
So let's do it the old-fashioned way.
So let's do it the old-fashioned way.
Zhaom
2021-03-12 23:25:23
solve for x and y if k=5
solve for x and y if k=5
MaryM
2021-03-12 23:25:26
We plug in k=5 to our equations.
We plug in k=5 to our equations.
MaryM
2021-03-12 23:25:40
Let's see how many times do y = x^2 - 5 and x = 2(y-20)^2 - 5 intersect.
Let's see how many times do y = x^2 - 5 and x = 2(y-20)^2 - 5 intersect.
MaryM
2021-03-12 23:25:50
We can substitute y into the 2nd equation to get x = 2((x^2-5)-20)^2 - 5.
We can substitute y into the 2nd equation to get x = 2((x^2-5)-20)^2 - 5.
MaryM
2021-03-12 23:26:00
This is x = 2(x^2 - 25)^2 - 5.
This is x = 2(x^2 - 25)^2 - 5.
MaryM
2021-03-12 23:26:14
So, as a quartic in x, this is 2x^4 - 100x^2 - x + 1245 = 0.
So, as a quartic in x, this is 2x^4 - 100x^2 - x + 1245 = 0.
MaryM
2021-03-12 23:26:21
Do we know a root of this equation?
Do we know a root of this equation?
ancientwarrior
2021-03-12 23:27:02
-5
-5
JimY
2021-03-12 23:27:02
x=-5
x=-5
sugar_rush
2021-03-12 23:27:06
x=-5
x=-5
Rubikscube3.1415
2021-03-12 23:27:06
-5
-5
MaryM
2021-03-12 23:27:08
We know that x=-5 is a root, so this must be divisible by x+5.
We know that x=-5 is a root, so this must be divisible by x+5.
MaryM
2021-03-12 23:27:14
We could do the long division to get a cubic, but maybe we don't have to do all that work?
We could do the long division to get a cubic, but maybe we don't have to do all that work?
MaryM
2021-03-12 23:27:18
What will the constant term of the resulting cubic be?
What will the constant term of the resulting cubic be?
sugar_rush
2021-03-12 23:27:30
249
249
JimY
2021-03-12 23:27:30
249
249
Lamboreghini
2021-03-12 23:27:34
249
249
functionalmath
2021-03-12 23:27:35
249
249
MaryM
2021-03-12 23:27:38
It'll be 1245/5 = 249.
It'll be 1245/5 = 249.
MaryM
2021-03-12 23:27:41
In particular, it's not a multiple of 5.
In particular, it's not a multiple of 5.
MaryM
2021-03-12 23:27:46
So we can't have a second root at x=-5, by the Rational Root Theorem.
So we can't have a second root at x=-5, by the Rational Root Theorem.
MaryM
2021-03-12 23:27:55
What's the conclusion?
What's the conclusion?
functionalmath
2021-03-12 23:28:16
4 roots
4 roots
MaryM
2021-03-12 23:28:24
We know that x=-5 is not a double root.
We know that x=-5 is not a double root.
MaryM
2021-03-12 23:29:00
Otherwise, -5 would be a root of the resulting polynomial, but it isn't.
Otherwise, -5 would be a root of the resulting polynomial, but it isn't.
MaryM
2021-03-12 23:29:11
So since the quartic must have a second negative root, that root must be different than x=-5.
So since the quartic must have a second negative root, that root must be different than x=-5.
MaryM
2021-03-12 23:29:27
That is, there is indeed a fourth intersection point hiding over there on the left side.
That is, there is indeed a fourth intersection point hiding over there on the left side.
MaryM
2021-03-12 23:29:52
That means that k=5 works!
That means that k=5 works!
palaashgang
2021-03-12 23:30:01
oh yeah i found 4 now
oh yeah i found 4 now
MaryM
2021-03-12 23:30:07
So we've established that if k \ge 5, then there are 4 distinct intersection points.
So we've established that if k \ge 5, then there are 4 distinct intersection points.
ancientwarrior
2021-03-12 23:30:15
cool
cool
sugar_rush
2021-03-12 23:30:19
But we still have to find the largest value of k.
But we still have to find the largest value of k.
MaryM
2021-03-12 23:30:29
Now we have to determine when they lie on a circle with radius at most 21.
Now we have to determine when they lie on a circle with radius at most 21.
MaryM
2021-03-12 23:30:36
Can we determine what the equation of that circle might be?
Can we determine what the equation of that circle might be?
MaryM
2021-03-12 23:32:17
Hint: We need an equation with an x^2 term and y^2 term in it. Can we find one by combining the equations of the two parabolas?
Hint: We need an equation with an x^2 term and y^2 term in it. Can we find one by combining the equations of the two parabolas?
happyhari
2021-03-12 23:33:17
yes, add them together and then complete the square
yes, add them together and then complete the square
MaryM
2021-03-12 23:33:20
Let's add the two given equations together! Any point that lies on both equations will also lie on the equation given by their sum.
Let's add the two given equations together! Any point that lies on both equations will also lie on the equation given by their sum.
MaryM
2021-03-12 23:33:25
We get x+y = x^2 - k + 2(y-20)^2 - k.
We get x+y = x^2 - k + 2(y-20)^2 - k.
MaryM
2021-03-12 23:33:43
Unfortunately, this ends up not being a circle.
Unfortunately, this ends up not being a circle.
Zhaom
2021-03-12 23:33:50
but we get a 2y^2 term, not a y^2 term
but we get a 2y^2 term, not a y^2 term
Lamboreghini
2021-03-12 23:33:50
wait, that's a 2y^2, which makes it an ellipse
wait, that's a 2y^2, which makes it an ellipse
MaryM
2021-03-12 23:33:57
It's an ellipse, because the x^2 term has a different coefficient than the 2y^2 term.
It's an ellipse, because the x^2 term has a different coefficient than the 2y^2 term.
MaryM
2021-03-12 23:34:08
Let's try again. What can we do instead?
Let's try again. What can we do instead?
Zhaom
2021-03-12 23:34:32
divide the second equation by 2 and add the 2 equations together
divide the second equation by 2 and add the 2 equations together
Lamboreghini
2021-03-12 23:34:32
double the y=x^2-k equation first
double the y=x^2-k equation first
Irving1004
2021-03-12 23:34:32
We can multiply the right one by 1/2
We can multiply the right one by 1/2
ancientwarrior
2021-03-12 23:34:32
divide both sides by 2 for the second equation
divide both sides by 2 for the second equation
Zhaom
2021-03-12 23:34:32
divide the second equation by 2 and then add
divide the second equation by 2 and then add
MaryM
2021-03-12 23:34:36
Add the first equation to half of the second equation.
Add the first equation to half of the second equation.
MaryM
2021-03-12 23:34:40
Now it's y + \dfrac12x = x^2 - k + (y-20)^2 - \dfrac{k}{2}.
Now it's y + \dfrac12x = x^2 - k + (y-20)^2 - \dfrac{k}{2}.
MaryM
2021-03-12 23:34:43
Is that a circle?
Is that a circle?
Lamboreghini
2021-03-12 23:34:55
yep!
yep!
peace09
2021-03-12 23:34:55
I Think S- Yes?
I Think S- Yes?
MaryM
2021-03-12 23:34:57
Yes! It's got an x^2 and a y^2 term.
Yes! It's got an x^2 and a y^2 term.
MaryM
2021-03-12 23:35:01
What's its radius?
What's its radius?
MaryM
2021-03-12 23:35:52
A better question would be "how can we find it's radius?"
A better question would be "how can we find it's radius?"
MaryM
2021-03-12 23:35:59
So, how can we find the radius of this circle?
So, how can we find the radius of this circle?
Zhaom
2021-03-12 23:36:15
complete the square
complete the square
MaryM
2021-03-12 23:36:18
We need to complete the square in x and in y.
We need to complete the square in x and in y.
MaryM
2021-03-12 23:36:23
Let's expand and get all the constants on one side.
Let's expand and get all the constants on one side.
MaryM
2021-03-12 23:36:28
We get \dfrac{3}{2}k - 400 = x^2 - \dfrac12x + y^2 - 41y.
We get \dfrac{3}{2}k - 400 = x^2 - \dfrac12x + y^2 - 41y.
MaryM
2021-03-12 23:36:50
To complete the square in x we need to add \left(\dfrac14\right)^2 = \dfrac{1}{16}.
To complete the square in x we need to add \left(\dfrac14\right)^2 = \dfrac{1}{16}.
MaryM
2021-03-12 23:37:02
And for y we need to add \left(\dfrac{41}{2}\right)^2 = \dfrac{1681}{4}.
And for y we need to add \left(\dfrac{41}{2}\right)^2 = \dfrac{1681}{4}.
MaryM
2021-03-12 23:37:09
So we get \dfrac{3}{2}k - 400 + \dfrac{1}{16} + \dfrac{1681}{4} = \left(x - \dfrac14\right)^2 + \left(y - \dfrac{41}{2}\right)^2.
So we get \dfrac{3}{2}k - 400 + \dfrac{1}{16} + \dfrac{1681}{4} = \left(x - \dfrac14\right)^2 + \left(y - \dfrac{41}{2}\right)^2.
MaryM
2021-03-12 23:37:21
So our equation is \dfrac{3}{2}k + \dfrac{325}{16} = \left(x-\dfrac14\right)^2 + \left(y-\dfrac{41}{2}\right)^2.
So our equation is \dfrac{3}{2}k + \dfrac{325}{16} = \left(x-\dfrac14\right)^2 + \left(y-\dfrac{41}{2}\right)^2.
MaryM
2021-03-12 23:37:29
What the radius then?
What the radius then?
Zhaom
2021-03-12 23:38:02
\sqrt{\frac{3k}{2}+\frac{325}{16}}
\sqrt{\frac{3k}{2}+\frac{325}{16}}
happyhari
2021-03-12 23:38:05
sqrt((3/2)k + (325/16))
sqrt((3/2)k + (325/16))
MaryM
2021-03-12 23:38:10
This is a circle with center \left(\dfrac14,\dfrac{41}{2}\right) and radius \sqrt{\dfrac{3}{2}k + \dfrac{325}{16}}.
This is a circle with center \left(\dfrac14,\dfrac{41}{2}\right) and radius \sqrt{\dfrac{3}{2}k + \dfrac{325}{16}}.
MaryM
2021-03-12 23:38:24
We need the radius to be at most 21.
We need the radius to be at most 21.
functionalmath
2021-03-12 23:38:27
sqrt(3k/2+325/16)<=21
sqrt(3k/2+325/16)<=21
MaryM
2021-03-12 23:38:30
So we need \sqrt{\dfrac{3}{2}k + \dfrac{325}{16}} \le 21.
So we need \sqrt{\dfrac{3}{2}k + \dfrac{325}{16}} \le 21.
MaryM
2021-03-12 23:38:35
Square both sides to get \dfrac{3}{2}k + \dfrac{325}{16} \le 441.
Square both sides to get \dfrac{3}{2}k + \dfrac{325}{16} \le 441.
MaryM
2021-03-12 23:38:40
Then multiply through by 16 to get 24k + 325 \le 7056.
Then multiply through by 16 to get 24k + 325 \le 7056.
MaryM
2021-03-12 23:38:45
So 24k \le 6731 and k \le \dfrac{6731}{24}.
So 24k \le 6731 and k \le \dfrac{6731}{24}.
MaryM
2021-03-12 23:38:55
The largest allowed value of k is \left\lfloor \dfrac{6731}{24} \right\rfloor.
The largest allowed value of k is \left\lfloor \dfrac{6731}{24} \right\rfloor.
Lamboreghini
2021-03-12 23:39:05
we have our upper bound for k!
we have our upper bound for k!
MaryM
2021-03-12 23:39:07
A little long division gives that this is 280.
A little long division gives that this is 280.
MaryM
2021-03-12 23:39:13
So we must have k \le 280 for the circle to have an allowed radius.
So we must have k \le 280 for the circle to have an allowed radius.
MaryM
2021-03-12 23:39:18
Combining this with the fact that k \ge 5 that we found earlier, we see that we must have 5 \le k \le 280.
Combining this with the fact that k \ge 5 that we found earlier, we see that we must have 5 \le k \le 280.
MaryM
2021-03-12 23:39:21
So what's the final answer?
So what's the final answer?
functionalmath
2021-03-12 23:39:33
so 285 is our answer
so 285 is our answer
happyhari
2021-03-12 23:39:33
285\
285\
Zhaom
2021-03-12 23:39:33
280+5=\boxed{285}
280+5=\boxed{285}
functionalmath
2021-03-12 23:39:33
285
285
palaashgang
2021-03-12 23:39:38
5+280 = 285
5+280 = 285
sugar_rush
2021-03-12 23:39:38
285
285
MaryM
2021-03-12 23:39:41
The sum of the smallest and largest k is 5 + 280 = \boxed{285}.
The sum of the smallest and largest k is 5 + 280 = \boxed{285}.
MaryM
2021-03-12 23:39:45
We made it through the AIME I !!
We made it through the AIME I !!
MaryM
2021-03-12 23:39:51
If you'd like to review this transcript or transcripts of any of our previous math jams, you can do so by visiting our past Math Jams transcripts page. Note that the transcript for this Math Jam may take a little while to show up there.
If you'd like to review this transcript or transcripts of any of our previous math jams, you can do so by visiting our past Math Jams transcripts page. Note that the transcript for this Math Jam may take a little while to show up there.
Lamboreghini
2021-03-12 23:40:03
Wow!
Wow!
sugar_rush
2021-03-12 23:40:03
Yay!!
Yay!!
palaashgang
2021-03-12 23:40:03
YAY
YAY
happyhari
2021-03-12 23:40:03
YAY!!
YAY!!
MaryM
2021-03-12 23:40:10
Please join us again for the AIME II Math Jam on Saturday, March 20th, at 6:30 PM Eastern / 3:30 PM Pacific.
Please join us again for the AIME II Math Jam on Saturday, March 20th, at 6:30 PM Eastern / 3:30 PM Pacific.
MaryM
2021-03-12 23:40:14
See you there! Bye!
See you there! Bye!
Copyright © 2025 AoPS Incorporated. This page is copyrighted material. You can view and print this page for your own use, but you cannot share the contents of this file with others.